Ask Our Doctors

Dear Patients,

I created this forum to welcome any questions you have on the topic of infertility, IVF, conception, testing, evaluation, or any related topics. I do my best to answer all questions in less than 24 hours. I know your question is important and, in many cases, I will answer within just a few hours. Thank you for taking the time to trust me with your concern.

– Geoffrey Sher, MD

Fill in the following information and we’ll get back to you.

Name: Katrina S

Dear Dr Sher,

My husband and I (both 35) have a severe male factor as well as lean PCOS. Despite this we had great fertilization and blastocysts rates in our IVF-ICSI with beautiful AA and BB embryos. Meanwhile we had 5 of our embryos transferred, of which 3 were able to implant but none grew beyond HCG 40 mIU/ml. I am lacking the activating KIR receptors (AA haplotype) on my uNK cells and have HLA C1/C2, my husband is C1/C1. To support implantation we apply Granocyte subcutaneously after FET. Our immune-compatibility was checked and we are not “too similar”. I have no inflammation in my endometrium, as the histology stainings were normal, and also no autoimmune autoantibodies (APS, thyroid, rheumatoid arthritis or SLE). My lining is good and the cavity looks good (no chronic/repetitive infections, no endometriosis). We are anxious to keep going with our remaining embryos. What else can we do to overcome repetitive pregnancy loss?

Any advice is very much appreciated. Thank you.

Answer:

When it comes to reproduction, humans are the poorest performers of all mammals. In fact we are so inefficient that up to 75% of fertilized eggs do not produce live births, and up to 30% of pregnancies end up being lost within 10 weeks of conception (in the first trimester). RPL is defined as two (2) or more failed pregnancies. Less than 5% of women will experience two (2) consecutive miscarriages, and only 1% experience three or more.

Pregnancy loss can be classified by the stage of pregnancy when the loss occurs:

  • Early pregnancy loss (first trimester)
  • Late pregnancy loss (after the first trimester)
  • Occult “hidden” and not clinically recognized, (chemical) pregnancy loss (occurs prior to ultrasound confirmation of pregnancy)
  • Early pregnancy losses usually occur sporadically (are not repetitive).

 

In more than 70% of cases the loss is due to embryo aneuploidy (where there are more or less than the normal quota of 46 chromosomes). Conversely, repeated losses (RPL), with isolated exceptions where the cause is structural (e.g., unbalanced translocations), are seldom attributable to numerical chromosomal abnormalities (aneuploidy). In fact, the vast majority of cases of RPL are attributable to non-chromosomal causes such as anatomical uterine abnormalities or Immunologic Implantation Dysfunction (IID).

Since most sporadic early pregnancy losses are induced by chromosomal factors and thus are non-repetitive, having had a single miscarriage the likelihood of a second one occurring is no greater than average. However, once having had two losses the chance of a third one occurring is double (35-40%) and after having had three losses the chance of a fourth miscarriage increases to about 60%. The reason for this is that the more miscarriages a woman has, the greater is the likelihood of this being due to a non-chromosomal (repetitive) cause such as IID. It follows that if numerical chromosomal analysis (karyotyping) of embryonic/fetal products derived from a miscarriage tests karyotypically normal, then by a process of elimination, there would be a strong likelihood of a miscarriage repeating in subsequent pregnancies and one would not have to wait for the disaster to recur before taking action. This is precisely why we strongly advocate that all miscarriage specimens be karyotyped.

There is however one caveat to be taken into consideration. That is that the laboratory performing the karyotyping might unwittingly be testing the mother’s cells rather than that of the conceptus. That is why it is not possible to confidently exclude aneuploidy in cases where karyotyping of products suggests a “chromosomally normal” (euploid) female.

Late pregnancy losses (occurring after completion of the 1st trimester/12th week) occur far less frequently (1%) than early pregnancy losses. They are most commonly due to anatomical abnormalities of the uterus and/or cervix. Weakness of the neck of the cervix rendering it able to act as an effective valve that retains the pregnancy (i.e., cervical incompetence) is in fact one of the commonest causes of late pregnancy loss. So also are developmental (congenital) abnormalities of the uterus (e.g., a uterine septum) and uterine fibroid tumors. In some cases intrauterine growth retardation, premature separation of the placenta (placental abruption), premature rupture of the membranes and premature labor can also causes of late pregnancy loss.

Much progress has been made in understanding the mechanisms involved in RPL. There are two broad categories:

  1. Problems involving the uterine environment in which a normal embryo is prohibited from properly implanting and developing. Possible causes include:
  • Inadequate thickening of the uterine lining
  • Irregularity in the contour of the uterine cavity (polyps, fibroid tumors in the uterine wall, intra-uterine scarring and adenomyosis)
  • Hormonal imbalances (progesterone deficiency or luteal phase defects). This most commonly results in occult RPL.
  • Deficient blood flow to the uterine lining (thin uterine lining).
  • Immunologic implantation dysfunction (IID). A major cause of RPL. Plays a role in 75% of cases where chromosomally normal preimplantation embryos fail to implan
  • Interference of blood supply to the developing conceptus can occur due to a hereditary clotting disorder known as Thrombophili

 

  1. Genetic and/or numerical chromosomal abnormalities(aneuploidy) of the embryo are far away the commonest overall causes of miscarriages. But this only applies to sporadic pregnancy losses (which comprises the majority of all miscarriages. However, recurrent, (consecutive) pregnancy losses are much more likely due to implantation dysfunction than to embryo-related issues, where implantation dysfunction (usually anatomical or immunologic) factors usually underly the problem.
  2. Genetic or Structural chromosomal abnormalities (which only occur in about 1% of cases) can also cause RPL. This is referred to as an unbalanced translocation and they result from part of one chromosome detaching and then fusing with another chromosome. Additionally, genetic defects (unrelated to chromosomal abnormalities) can also affect embryo quality and pregnancy outcome. Damaged sperm DNA can sometimes be diagnosed using the SCSA (see before) which primarily measures the sperm DNA fragmentation index (DFI).

 

IMMUNOLOGIC IMPLANTATION DYSFUNCTIO-IID (see before)

Autoimmune IID: Here an immunologic reaction is produced by the individual to his/her body’s own cellular components. The most common antibodies that form in such situations are APA and antithyroid antibodies

Alloimmune IID, i.e., where there is an immunologic reaction to antigens derived from another member of the same species (i.e. the woman’s immune system reacts to the paternal antigens in the sperm (see above) .

*It is important to recognize that alloimmune (rather than autoimmune) IID is more commonly associated with RPL.

Autoimmune IID is often genetically transmitted. Thus, it should not be surprising to learn that it is more likely to exist in women who have a family (or personal) history of primary autoimmune diseases such as lupus erythematosus (LE), scleroderma or autoimmune hypothyroidism (Hashimoto’s disease), autoimmune hyperthyroidism (Grave’s disease), rheumatoid arthritis, etc. Reactionary (secondary) autoimmunity can occur in conjunction with any medical condition associated with widespread tissue damage. One such gynecologic condition is endometriosis. Since autoimmune IID is usually associated with activated NK and T-cells from the outset, it usually results in such very early destruction of the embryo’s root system that the patient does not even recognize that she is pregnant. Accordingly, the condition usually presents as “unexplained infertility” or “unexplained IVF failure” rather than as a miscarriage.

Alloimmune IID, on the other hand, usually starts off presenting as unexplained miscarriages (often manifesting as RPL). Over time as NK/T cell activation builds and eventually becomes permanently established the patient often goes from RPL to “infertility” due to failed implantation. RPL is more commonly the consequence of alloimmune rather than autoimmune implantation dysfunction.

However, regardless, of whether miscarriage is due to autoimmune or alloimmune implantation dysfunction the final blow to the pregnancy is the result of activated NK cells and CTL in the uterine lining that damage the developing embryo’s “root system” (trophoblast) so that it can no longer sustain the growing conceptus. This having been said, it is important to note that autoimmune IID is readily amenable to reversal through timely, appropriately administered, selective immunotherapy, and alloimmune IID is not. It is much more difficult to treat successfully, even with the use of immunotherapy. In fact, in some cases the only solution will be to revert to selective immunotherapy plus using donor sperm (provided there is no “match” between the donor’s DQ alpha/HLA profile and that of the female recipient) or alternatively to resort to gestational surrogacy.

DIAGNOSING THE CAUSE OF RPL

In the past, women who miscarried were not evaluated thoroughly until they had lost several pregnancies in a row. This was because sporadic miscarriages are most commonly the result of embryo numerical chromosomal irregularities (aneuploidy) and thus not treatable. However, a consecutive series of miscarriages points to a repetitive cause that is non-chromosomal and is potentially remediable. Since RPL is most commonly due to a uterine pathology or immunologic causes that are potentially treatable, it follows that early chromosomal evaluation of products of conception could point to a potentially treatable situation. Thus, we strongly recommend that such testing be done in most cases of miscarriage. Doing so will avoid a great deal of unnecessary heartache for many patients.

Establishing the correct diagnosis is the first step toward determining effective treatment for couples with RPL. It results from a problem within the pregnancy itself or within the uterine environment where the pregnancy implants and grows. Diagnostic tests useful in identifying individuals at greater risk for a problem within the pregnancy itself include:

  • Karyotyping (chromosome analysis) both prospective parents
  • Assessment of the karyotype of products of conception derived from previous miscarriage specimens
  • Ultrasound examination of the uterine cavity after sterile water is injected or sonohysterogram, fluid ultrasound, etc.
  • Hysterosalpingogram (dye X-ray test)
  • Hysteroscopic evaluation of the uterine cavity
  • Full hormonal evaluation (estrogen, progesterone, adrenal steroid hormones, thyroid hormones, FSH/LH, )
  • Immunologic testing to include:
    • Antiphospholipid antibody (APA) panel
    • Antinuclear antibody (ANA) panel
    • Antithyroid antibody panel (i.e., antithyroglobulin and antimicrosomal antibodies)
    • Reproductive immunophenotype
    • Natural killer cell activity (NKa) assay (i.e., K562 target cell test)
    • Alloimmune (DQ alpha/HLA) testing of both the male and female partners

 

TREATMENT OF RPL

Treatment for Anatomic Abnormalities of the Uterus: This involves restoration through removal of local lesions such as fibroids, scar tissue, and endometrial polyps or timely insertion of a cervical cerclage (a stitch placed around the neck of the weakened cervix) or the excision of a uterine septum when indicated.

Treatment of Thin Uterine Lining: A thin uterine lining has been shown to correlate with compromised pregnancy outcome. Often this will be associated with reduced blood flow to the endometrium.  Such decreased blood flow to the uterus can be improved through treatment with sildenafil and possibly aspirin.

Sildenafil (Viagra) Therapy (see above). Viagra has been used successfully to increase uterine blood flow. To date, we have seen significant improvement of the thickness of the uterine lining in about 70% of women treated. Successful pregnancy resulted in almost half of those women who responded to the Viagra. It should be borne in mind that most of these women had previously experienced repeated IVF failures.

Use of Aspirin: This is an anti-prostaglandin that improves blood flow to the endometrium. It is administered at a dosage of 81 mg orally, daily from the beginning of the cycle until ovulation.

Treating Immunologic Implantation Dysfunction with Selective Immunotherapy: Modalities such as IL/IVIg, heparinoids (Lovenox/Clexane), and corticosteroids can be used in select cases depending on autoimmune or alloimmune dysfunction.

The Use of IVF in the Treatment of RPL. In the following circumstances, IVF is the preferred option:

  • When in addition to a history of RPL, another standard indication for IVF (e.g., tubal factor, endometriosis, and male factor infertility) is superimposed.
  • In cases where selective immunotherapy is needed to treat an immunologic implantation dysfunction.

The reason for IVF being a preferred approach in such cases is that in order to be effective, the immunotherapy needs to be initiated well before spontaneous or induced ovulation. Given the fact that in the absence of IVF the anticipated birthrate per cycle of COS with or without IUI is at best about 15%, it follows that short of IVF, to have even a reasonable chance of a live birth, most women with immunologic causes of RPL would need to undergo immunotherapy repeatedly, over consecutive cycles. Conversely, with IVF, the chance of a successful outcome in a single cycle of treatment is several times greater and, because of the attenuated and concentrated time period required for treatment, IVF is far safer and thus represents a more practicable alternative

Since embryo aneuploidy is a common cause of miscarriage, the use of PGS/PGT-A can provide a valuable diagnostic and therapeutic advantage in cases of RPL. PGD requires IVF to provide access to embryos for testing.

There are a few cases of intractable alloimmune dysfunction due to “complete DQ alpha matching where Gestational Surrogacy or use of Donor  Sperm could represent the only viable recourse, other than abandoning treatment altogether and/or resorting to adoption. Other non-immunologic factors such as an intractably thin uterine lining or severe uterine pathology might also warrant that last resort consideration be given to gestational surrogacy.

The good news is that if a couple with RPL is open to all of the diagnostic and treatment options referred to above, a live birthrate of 70%–80% is ultimately achievable.

I strongly recommend that you visit www.SherIVF.com. Then go to my Blog and access the “search bar”. Type in the titles of any/all of the articles listed below, one by one. “Click” and you will immediately be taken to those you select.  Please also take the time to post any questions or comments with the full expectation that I will (as always) respond promptly.

  • The IVF Journey: The importance of “Planning the Trip” Before Taking the Ride”
  • Controlled Ovarian Stimulation (COS) for IVF: Selecting the ideal protocol
  • IVF: Factors Affecting Egg/Embryo “competency” during Controlled Ovarian Stimulation(COS)
  • The Fundamental Requirements For Achieving Optimal IVF Success
  • Ovarian Stimulation for IVF using GnRH Antagonists: Comparing the Agonist/Antagonist Conversion Protocol.(A/ACP) With the “Conventional” Antagonist Approach
  • Ovarian Stimulation in Women Who have Diminished Ovarian Reserve (DOR): Introducing the Agonist/Antagonist Conversion protocol
  • Anti Mullerian Hormone (AMH) Measurement to Assess Ovarian Reserve and Design the Optimal Protocol for Controlled Ovarian Stimulation (COS) in IVF.
  • Human Growth Hormone Administration in IVF: Does it Enhances Egg/Embryo Quality and Outcome?
  • The BCP: Does Launching a Cycle of Controlled Ovarian Stimulation (COS). Coming off the BCP Compromise Response?
  • Blastocyst Embryo Transfers Should be the Standard of Care in IVF
  • IVF: How Many Attempts should be considered before Stopping?
  • “Unexplained” Infertility: Often a matter of the Diagnosis Being Overlooked!
  • IVF Failure and Implantation Dysfunction:
  • The Role of Immunologic Implantation Dysfunction (IID) & Infertility (IID):PART 1-Background
  • Immunologic Implantation Dysfunction (IID) & Infertility (IID):PART 2- Making a Diagnosis
  • Immunologic Dysfunction (IID) & Infertility (IID):PART 3-Treatment
  • Thyroid autoantibodies and Immunologic Implantation Dysfunction (IID)
  • Immunologic Implantation Dysfunction: Importance of Meticulous Evaluation and Strategic Management:(Case Report
  • Intralipid and IVIG therapy: Understanding the Basis for its use in the Treatment of Immunologic Implantation Dysfunction (IID)
  • Intralipid (IL) Administration in IVF: It’s Composition; How it Works; Administration; Side-effects; Reactions and Precautions
  • Natural Killer Cell Activation (NKa) and Immunologic Implantation Dysfunction in IVF: The Controversy!
  • Endometrial Thickness, Uterine Pathology and Immunologic Factors
  • Vaginally Administered Viagra is Often a Highly Effective Treatment to Help Thicken a Thin Uterine Lining
  • Treating Out-of-State and Out-of-Country Patients at Sher-IVF in Las Vegas:
  • A personalized, stepwise approach to IVF
  • How Many Embryos should be transferred: A Critical Decision in IVF.
  • The Role of Nutritional Supplements in Preparing for IVF

 

 

______________________________________________________

ADDENDUM: PLEASE READ!!

INTRODUCING SHER FERTILITY SOLUTIONS (SFS)

Founded in April 2019, Sher Fertility Solutions (SFS) offers online (Skype/FaceTime) consultations to patients from > 40 different countries. All consultations are followed by a detailed written report presenting my personal recommendations for treatment of what often constitute complex Reproductive Issues.

 

If you wish to schedule an online consultation with me, please contact my assistant (Patti Converse) by phone (800-780-7437/702-533-2691), email (concierge@SherIVF.com) or,  enroll online on then home-page of my website (www.SherIVF.com). 

 

PLEASE SPREAD THE WORD ABOUT SFS!

 

Geoff Sher

 

 

 

Name: Brittany F

Hi Doctor,

I am a 30 year old woman with a recent POI diagnosis. I no longer have a natural cycle and at diagnosis in January my FSH was as high as 105. Currently I have been on HRT for 10 months and my FSH is now around 20. During a recent ultrasound I had several antral follicles seen on both ovaries (around 10-12 total, each no greater than 2 mm).

Most other REs I have seen since diagnosis have told me donor egg is my only option, but I know ovulation is possible and have heard of other women with POI having success under the right circumstances. It feels like many doctors don’t want patients like me, but I would love the chance to try to get pregnant with my own eggs before exploring other options.

Do you offer protocols or regimens for women with POI who are trying to conceive with their own eggs?

Answer:

Yes! I do offer the opportunity to try and ovulate on ovarian stimulation. However, you need to be realistic about the fact that in the final analysis, egg donor-IVF might ultimately be your only option.

 

Feel free to call my assistant, Patti at 702-533-2691 and set up an online consultation with me to discuss!~

 

 

 

Name: Victor Lewis L

I need Goeff Sher,s personal email. Dr Victor Lewis M.D. FRCOG united kingdom

Answer:

Hi Victor,

 

Would  you kindly email me at sher@sherivf.com and also send me your residential address and phone # and I will forward you all my contact information.

 

Looking forward to hearing from you!

 

Geoff

Name: Sayora G

Dear Dr Sher,
I have just given birth to baby son ( September 3d). The pregnancy was miracle due to natural conception in December last year. I delivered naturally (not C-section) at 39 weeks 4 days so at full term.
My question is- I had 11 IVFs prior to this miracle pregnancy. I had one transfer which resulted to miscarriage ( trisomy 15) and since then I have PGT all my embryos. Majority of them came aneuploid (20 embryos- frozen them all) and two are euploid. One was day 6 ,3 AB and have not attached. This was in 2020. I have one euploid embryo left- day 5, 3AB.
I want to have second baby and use this embryo, but to have best possible protocol.
Given the fact that I had miracle pregnancy without any complications and easy natural delivery, please advise how quickly can I have my euploid embryo transfer? My son is almost two month now. What protocol would you suggest?

Answer:

I would use both the euploid blastocysts. My suggestion is that once you have stopped breast feeding and are menstruating regularly you proceed to FET.

 

Good luck!

 

 

Iui

Name: Kemisha O

I have low reserve egg quality. I currently did IuI and it failed 😔 also gonal F works fast to help follicles grow. I don’t have menstrual Dr said I have secondary amenorrhea
Is there any hope for me. I can’t afford IVF

Answer:

We would need to talk. Feel free to set up an online consultation by calling my assistant, Patti Converse (702-533-2691)

 

Geoff Sher

Name: Amanda h

Hello,

I had a laproscopy to remove 5cm endometrioma 14 months ago. I have stage 4 endometriosis. The endometrioma was conservatively removed at about 90% to preserve ovarian tissue. Ultrasound now shows the endometrioma is back and measuring 2cm. The ovary is adhered to cervix now as well. I’m looking to do a duo stim. Would sclerotherapy be a good option before ivf or not likely due to the fact the endometrioma was operated on already and the fact that the ovary is stuck to the cervix? Also planning to do prp. Thank you.

Answer:

Sclerotherapy would be an option, but it might not be readily available.

 

Endometriosis is a condition that occurs when the uterine lining (endometrium)  grows not only in the interior of the uterus but in other areas, such as the Fallopian tubes, ovaries and the bowel. Endometriosis is a complex condition where, the lack or relative absence of an overt anatomical barrier to fertility often belies the true extent of reproductive problem(s).

All too often the view is expounded that the severity of endometriosis-related infertility is inevitably directly proportionate to the anatomical severity of the disease itself, thereby implying that endometriosis causes infertility primarily by virtue of creating anatomical barriers to fertilization. This over-simplistic and erroneous view is often used to support the performance of many unnecessary surgeries for the removal of small innocuous endometriotic lesions, on the basis of such “treatment” evoking an improvement in subsequent fertility.

It is indisputable that even the mildest form of endometriosis can compromise fertility. It is equally true that, mild to moderate endometriosis is by no means a cause of absolute “sterility”.

Rather, when compared with normally ovulating women of a similar age who do not have endometriosis, women with mild to moderate endometriosis are about four to six times less likely to have a successful pregnancy. Endometriosis often goes unnoticed for many years. Such patients are frequently, erroneously labeled as having “unexplained infertility”, until the diagnosis is finally clinched through direct visualization of the lesions at the time of laparoscopy or laparotomy. Not surprisingly, many patients with so called “unexplained” infertility, if followed for a number of years, will ultimately reveal endometriosis.

Women who have endometriosis are far more likely to be infertile. There are several reasons for this:

  • First-Ovulation Dysfunction: In about 25 – 30% of cases, endometriosis is associated with ovulation dysfunction. Treatment requires controlled ovarian stimulation (COS). The problem is that the toxic pelvic environment markedly reduces the likelihood that anything other than IVF will enhance pregnancy potential.
  • Second- Toxic Pelvic environment that compromises Fertilization Endometriosis is associated with the presence of toxins in the peritoneal secretions. While it is tempting to assert that endometriosis-related infertility is confined to cases with more severe anatomical disease , that normally ovulating women with mild to moderate endometriosis (where the Fallopian tubes are usually patent and free) should  have no difficulty in conceiving once their anatomical disease is addressed surgically, …nothing could be further from the truth. The natural conception rate for healthy young (<35y) ovulating women who are free of endometriosis, is about 15% per month of trying and 70% per year of actively trying to conceive. Conversely, the conception rate for women of a comparable age who have mild or moderate pelvic endometriosis  is only about 2-4% per month and approximately 40% after 4 The main reason for this difference is that as the  egg travels from the ovary to the fallopian tube, it is exposed to these peritoneal toxins which compromise the fertilization process. And, this “toxic pelvic influence, cannot be eradicated through surgically removing visible endometriotic deposits in the pelvis or through any medication.  The reason that surgical ablation of endometriotic deposits will not improve pregnancy potential is that  for every deposit observed, there are numerous others that are in the process of developing, which at the time might not be visible (because they are translucent) but  still produce toxins. This also explains why surgery to remove visible endometriotic deposits, controlled ovarian stimulation (COS) with or without intrauterine insemination will usually not improve pregnancy potential. Only IVF, through removing eggs before they are exposed to this toxic pelvic environment, fertilizing them in in the IVF laboratory and then transferring the embryo(s) to the uterus represents the only way to enhance pregnancy potential.
  • Third-Pelvic adhesions and Scarring:In its most severe form, endometriosis is associated with scarring and adhesions in the pelvis, resulting in damage to, obstruction or fixation of the fallopian tubes to surrounding structures, thereby preventing the union of sperm and eggs.
  • Fourth-Ovarian Endometriomas, Advanced endometriosis is often associated with ovarian cysts (endometriomas/chocolate cysts) that are filled with altered blood and can be large and multiple. When these are sizable (>1cm) they can activate surrounding ovarian connective tissue causing production of excessive male hormones (androgens) such as testosterone and androstenedione. Excessive ovarian androgens can compromise egg development in the affected ovary (ies) resulting in an increased likelihood of numerical chromosomal abnormalities (aneuploidy) and reduced egg/embryo competency”. In my opinion ovarian endometriomas larger than 1cm need to be removed surgically or though sclerotherapy before embarking on IVF.
  • Fifth- Immunologic Implantation Dysfunction (IID). Endometriosis, regardless of its severity is associated with immunologic implantation dysfunction (IID) linked to activation of uterine natural killer cells (NKa) and cytotoxic lymphocytes (CTL) in about 30% of cases. This is diagnosed by testing the woman’s blood for NKa using the K-562 target cell test or by cytokine analysis, and by doing a blood immunophenotype evaluation. These NKa attack the invading trophoblast cells (developing “root system” of the embryo/early conceptus) as soon as it tries to gain attachment to the uterine wall. In most such cases, this will result in death of the embryo even before the pregnancy is diagnosed and sometimes, in a chemical pregnancy or an early miscarriage. . As such, many women with endometriosis, rather than being infertile, in the strict sense of the word are actually experiencing repeated undetected “mini-miscarriages”.

Advanced Endometriosis: In its most advanced stage, anatomical tubal and ovarian disfiguration is causally linked to the infertility. In such cases, inspection at laparoscopy or laparotomy will usually reveal severe pelvic adhesions, scarring and  endometriomas. However, the quality of life of patients with advanced endometriosis is usually so severely compromised by pain and discomfort, that having a baby is often relatively low on their priority lists. Accordingly, such patients are often more interested in relatively radical medical and surgical treatment options (might preclude a subsequent pregnancy), such as removal of ovaries, fallopian pubis and even the uterus, as a means of alleviating their symptoms.

Moderately Severe Endometriosis. These patients have a modest amount of scarring/ adhesions and endometriotic deposits which are usually detected on the ovaries, Fallopian tubes, bladder surface and on the peritoneal surface, low down  in the pelvis, behind the uterus (in the cul-de-sac). In such cases, the Fallopian tubes are usually opened and functional.

Mild Endometriosis: These are patients who at laparoscopy or laparotomy are found to have no significant distortion of pelvic anatomy are often erroneously labeled as having “unexplained” infertility. To hold that infertility can only be attributed to endometriosis if significant anatomical disease can be identified, is to ignore the fact that, biochemical, hormonal and immunological factors profoundly impact fertility. Failure to recognize this salient fact continues to play havoc with the hopes and dreams of many infertile endometriosis patients.

Treatment:

The following basic concepts apply to management of endometriosis-related infertility:

  1. Controlled Ovulation stimulation (COS) with/without intrauterine insemination (IUI): Toxins in the peritoneal secretions of women with endometriosis exert a negative effect on fertilization potential, regardless of how sperm reach the Fallopian tube(s). This explains why COS with or without IUI will usually not improve the chances of pregnancy (over no treatment at all) in women with endometriosis. IVF is the only way by which to bypass this problem.
  2. Laparoscopy or Laparotomy Surgery aimed at restoring the anatomical integrity of the Fallopian tubes does not counter the negative influence of toxic peritoneal factors that inherently reduce the chances of conception in women with endometriosis many Nor does it address the immunologic implantation dysfunction (IID) often associated with this condition. Pelvic surgery is relatively contraindicated for the treatment of infertility associated with endometriosis, when the woman is more than 35 years of age as such women do not have the time to waste on such less efficacious alternatives. In contrast, younger women who have much more time on their side might consider surgery as a viable option. Approximately 30 -40 percent of women under 35 years of age with endometriosis will conceive within three to four years following corrective pelvic surgery.
  3. Sclerotherapy for ovarian endometriomas (“chocolate” cysts).About 20 years ago I introduced “sclerotherapy”, a relatively non-invasive, safe and effective outpatient method to permanently eliminate endometriomas without surgery being required. Sclerotherapy for ovarian endometriomas involves needle aspiration of the liquid content of the endometriotic cyst, followed by the injection of 5% tetracycline hydrochloride into the cyst cavity. Treatment results in disappearance of the lesion within 6-8 weeks, in more than 75% of cases so treated. Ovarian sclerotherapy can be performed under local anesthesia or under general anesthesia. It has the advantage of being an ambulatory office- based procedure, at low cost, with a low incidence of significant post-procedural pain or complications and the avoidance of the need for laparoscopy or laparotomy.
  4. The role of selective immunotherapyMore than half of women who have endometriosis harbor antiphospholipid antibodies (APA) that can compromise development of the embryo’s root system (trophoblast). In addition and far more serious, is the fact that in about one third of cases endometriosis, regardless of its severity is associated with NKa and cytotoxic uterine lymphocytes (CTL) which can seriously jeopardize implantation. This immunologic implantation dysfunction (IID) is diagnosed by testing the woman’s blood for APA, for NKa (using the K-562 target cell test and/or by testing for cytokine activity) and, for CTL (by a blood immunophenotype). Activated NK cells attack the invading trophoblast cells (developing “root system” of the embryo) as soon as it tries to gain attachment to the uterine wall. In most cases, this results in rejection of the embryo even before the pregnancy is diagnosed and sometimes, in a chemical pregnancy or an early miscarriage. . As such, many women with endometriosis, rather than being infertile, in the strict sense of the word, often actually experience repeated undetected “mini-miscarriages” Women who harbor APA’s often experience improved IVF birth rates when heparinoids (Clexane/Lovenox) are administered from the onset of ovarian stimulation with gonadotropins until the 10th week of pregnancy. Endometriosis-induced NKa is treated with a combination of Intralipid (IL) and steroid  (dexamethasone or prednisone)therapy: Intralipid (IL) is a solution of small lipid droplets suspended in water. When administered intravenously, IL provides essential fatty acids, linoleic acid (LA), an omega-6 fatty acid, alpha-linolenic acid (ALA), an omega-3 fatty acid.IL is made up of 20% soybean oil/fatty acids (comprising linoleic acid, oleic acid, palmitic acid, linolenic acid and stearic acid) , 1.2% egg yolk phospholipids (1.2%), glycerin (2.25%)  and water (76.5%).IL exerts a modulating effect on certain immune cellular mechanisms largely by down-regulating NKa. The therapeutic effect of IL/steroid therapy is likely due to an ability to suppress pro-inflammatory cellular (Type-1) cytokines such as interferon gamma and TNF-alpha. IL/steroids down-regulates NKa within 2-3 weeks of treatment the vast majority of women experiencing immunologic implantation dysfunction. In this regard IL is just as effective as Intravenous Gamma globulin (IVIg) but at a fraction of the cost and with a far lower incidence of side-effects. Its effect lasts for 4-9 weeks when administered in early pregnancy.
  5. The role of IVF: The toxic pelvic environment caused by endometriosis, profoundly reduces natural fertilization potential. As a result normally ovulating infertile women with endometriosis and patent Fallopian tubes are much less likely to conceive naturally, or by using fertility agents alone (with or without intrauterine (IUI) insemination. The only effective way to bypass this adverse pelvic environment is through IVF. I am not suggesting here that all women who have endometriosis require IVF! Rather, I am saying that in cases where the condition is further compromised by an IID associated with NKa and/or for older women(over 35y)  who have  diminished ovarian reserve (DOR) where time is of the essence, it is my opinion that IVF is the treatment of choice.

 

 Call my assistant, Patti Converse (702-533-2691 and set up an online consultation with me.

 

Good luck

Name: Vaughn K

I’ve been trying to conceive a baby with my significant other for a couple months now and have had no luck. We’ve done everything we possibly can and nothing is working. Is it possible that if I smoke nicotine that could be the cause? Would it make my sperm infertil? If not could there be any other causes of not being able to conceive, and if it is because of the nicotine what do you suggest I do?

Answer:

Usually you would wait 1 full year before diagnosing infertility. If not pregnant by then, you would need an infertility work-up with your  local doctor or call my assistant, Patti Converse (702-533-2691) and set up a consultation with me online.

 

I don’t think the use of nicotine is at fault here.

 

Geoff Sher

Name: Denisa K

Dear sir! My name is Denisa, I am 32. In my 29 I gave a birth to a beautiful boy. Got pregnant for the first month trying, verything went well. After labour, there was a very small piece of placenta left in my uterus. I had inflamatory and month after labour there was heavy bleeding and courretage had to be done. My menstruation since that is very poor and my lining grows up to 6,5mm. We have been trying for the second baby for more than 2 years. I had two hysteroscopies done, first to clean the adhesions (mild Asherman) second look to control my uterus. Everything seems nice and cavity is clear. But simply does not grow. I took oestrogen, for the first months it helped endometrium grow up to 8mm, but blocked my ovulation. After months we tried ivf, there were two transfers, but unsuccesful, I reached only 7mm after 10mg oestrogens orally+vaginally. I am very affraid I am the “Marissa example”. Now I would like to ask you abou Tadalafil Mylan. Do you think I can try this? Is it so helpful as Sildenafil? Maybe this treatment could help me a little. Or you see no chance in case like mine? Thank you for your answer! Denisa

Answer:

This is likely due to  post-partum endometritis following retained products of conception. WE should talk. Might I suggest that you call my assistant, Patti Converse (702-533-2691 and set up an online consultation with me to discuss options:

 

It was as far back as 1989, when I first published a study that examined the correlation between the thickness of a woman’s uterine lining (the endometrium), and the subsequent successful implantation of embryos in IVF patients. This study revealed that when the uterine lining measured <8mm in thickness by the day of the “hCG trigger” (in fresh IVF cycles), or at the time of initiating progesterone therapy (in embryo recipient cycles, e.g. frozen embryo transfers-FET, egg donation-IVF etc.) , pregnancy and birth rates were substantially improved. Currently, it is my opinion, that an ideal estrogen-promoted endometrial lining should ideally measure at least 9mm in thickness and that an endometrial lining measuring 8-9mm is “intermediate”. An estrogenic lining of <8mm is in most cases unlikely to yield a viable pregnancy.

 

A “poor” uterine lining is usually the result of the innermost layer of endometrium (the basal or germinal endometrium from which endometrium grows) ) not being able to respond to estrogen by propagating an outer, “functional” layer thick enough  to support optimal embryo implantation and development of a healthy placenta (placentation). The “functional” layer ultimately comprises 2/3 of the full endometrial thickness and is the layer that sheds with menstruation in the event that no pregnancy occurs.

 

The main causes of a “poor” uterine lining are:

 

  1. Damage to the basal endometrium as a result of:
    1. Inflammation of the endometrium (endometritis) most commonly resulting from infected products left over following abortion, miscarriage or birth
    2. Surgical trauma due to traumatic uterine scraping, (i.e. due to an over-aggressive D & C)
  2. Insensitivity of the basal endometrium to estrogen due to:
    1. Prolonged , over-use/misuse of clomiphene citrate
    2. Prenatal exposure to diethylstilbestrol (DES).  This is a drug that was given to pregnant women in the 1960’s to help prevent miscarriage
  3. Over-exposure of the uterine lining to ovarian male hormones (mainly testosterone): Older women, women with diminished ovarian reserve (poor responders) and women with polycystic ovarian syndrome -PCOS tend to have raised LH biological activity.. This causes the connective tissue in the ovary (stroma/theca) to overproduce testosterone. The effect can be further exaggerated when certain methods for ovarian stimulation such as agonist (Lupron/Buserelin) “flare” protocols and high dosages of menotropins such as Menopur are used in such cases.
  4. Reduced blood flow to the basal endometrium:

Examples include;

    1. Multiple uterine fibroids – especially when these are present under the endometrium (submucosal)
  1. Uterine adenomyosis (excessive, abnormal invasion of the uterine muscle by endometrial glands).

 

 

“The Viagra Connection”

 

Eighteen years ago years ago, after reporting on the benefit of vaginal Sildenafil (Viagra) for to women who had implantation dysfunction due to thin endometrial linings I was proud to announce the birth of the world’s first “Viagra baby.” Since the introduction of this form of treatment, thousands of women with thin uterine linings have been reported treated and many have gone on to have babies after repeated prior IVF failure.

 

For those of you who aren’t familiar with the use of Viagra in IVF, allow me to provide some context. It was in the 90’s that Sildenafil (brand named Viagra) started gaining popularity as a treatment for erectile dysfunction.  The mechanism by which it acted was through increasing penile blood flow through increasing nitric oxide activity. This prompted me to investigate whether Viagra administered vaginally, might similarly improve uterine blood flow and in the process cause more estrogen to be delivered to the basal endometrium and thereby increase endometrial thickening. We found that when Viagra was administered vaginally it did just that! However oral administration was without any significant benefit in this regard.  We enlisted the services of a compound pharmacy to produce vaginal Viagra suppositories. Initially, four (4) women with chronic histories of poor endometrial development and failure to conceive following several advanced fertility treatments were evaluated for a period of 4-6 weeks and then underwent IVF with concomitant Viagra therapy. Viagra suppositories were administered four times daily for 8-11 days and were discontinued 5-7 days prior to embryo transfer in all cases.

 

Our findings clearly demonstrated that vaginal Viagra produced a rapid and profound improvement in uterine blood flow and that was followed by enhanced endometrial development in all four cases. Three (3) of the four women subsequently conceived. I expanded the trial in 2002 and became the first to report on the administration of vaginal Viagra to 105 women with repeated IVF failure due to persistently thin endometrial linings. All of the women had experienced at least two (2) prior IVF failures attributed to intractably thin uterine linings. About 70% of these women responded to treatment with Viagra suppositories with a marked improvement in endometrial thickness. Forty five percent (45%) achieved live births following a single cycle of IVF treatment with Viagra The miscarriage rate was 9%. None of the women who had failed to show an improvement in endometrial thickness following Viagra treatment achieved viable pregnancies.

 

Following vaginal administration, Viagra is rapidly absorbed and quickly reaches the uterine blood system in high concentrations. Thereupon it dilutes out as it is absorbed into the systemic circulation. This probably explains why treatment is virtually devoid of systemic side effects

 

It is important to recognize that Viagra will NOT be effective in improving endometrial thickness in all cases. In fact, about 30%-40% of women treated fail to show any improvement. This is because in certain cases of thin uterine linings, the basal endometrium will have been permanently damaged and left unresponsive to estrogen. This happens in cases of severe endometrial damage due mainly to post-pregnancy endometritis (inflammation), chronic granulomatous inflammation due to uterine tuberculosis (hardly ever seen in the United States) and following extensive surgical injury to the basal endometrium (as sometimes occurs following over-zealous D&C’s).

 

Combining vaginal Viagra Therapy with oral Terbutaline;

In my practice I sometimes recommend combining Viagra administration with 5mg of oral terbutaline. The Viagra relaxes the muscle walls of uterine spiral arteries that feed the basal (germinal) layer of the endometrium while Terbutaline, relaxes the uterine muscle through which these spiral arteries pass. The combination of these two medications interacts synergistically to maximally enhance blood flow through the uterus, thereby improving estrogen delivery to the endometrial lining. The only drawback in using Terbutaline is that some women experience agitation, tremors and palpitations. In such cases the terbutaline should be discontinued. Terbutaline should also not be used women who have cardiac disease or in those who have an irregular heartbeat.

 

About 75% of women with thin uterine linings see a positive response to treatment within 2-3 days. The ones that do not respond well to this treatment are those who have severely damaged inner (basal/germinal) endometrial linings, such that no improvement in uterine blood flow can coax an improved response. Such cases are most commonly the result of prior pregnancy-related endometrial inflammation (endometritis) that sometimes occurs post abortally or following infected vaginal and/or cesarean delivery.

 

Viagra therapy has proven to be a god send to thousands of woman who because of a thin uterine lining would otherwise never have been able to successfully complete the journey “from infertility to family”.

 

 

___________________________________________________

ADDENDUM: PLEASE READ!!

INTRODUCING SHER FERTILITY SOLUTIONS (SFS)

Founded in April 2019, Sher Fertility Solutions (SFS) offers online (Skype/FaceTime) consultations to patients from > 40 different countries. All consultations are followed by a detailed written report presenting my personal recommendations for treatment of what often constitute complex Reproductive Issues.

 

Patients are encouraged to share the information I provide, with their treating Physicians and/or to avail themselves of my personal hands-on services, provided through batched IVF cycles that I conduct every 3 months at Los Angeles IVF (LAIVF) Clinic, Century City, Los Angeles, CA.

 

If you wish to schedule an online consultation with me, please contact my assistant (Patti Converse) by phone (800-780-7437/702-533-2691), email (concierge@SherIVF.com) or,  enroll online on then home-page of my website (www.SherIVF.com). 

 

PLEASE SPREAD THE WORD ABOUT SFS!

 

Geoff Sher

 

Name: Christine K

Thoughts on priming with estrace and testosterone?

Answer:

Estrogen priming for cases of severely diminished ovarian reserve is in my opinion worthy of consideration. Rarely is testosterone priming beneficial!

 

Geoff Sher

Name: Anne-Marie D

Hi Dr Sher,

I’ve had one natural pregnancy which ended at 19 weeks. I had a D&C for this. Since then I’ve had two 3D saline sonograms, both of which have shown a completely normal uterine cavity.

Since my D&C, I’ve had two failed embyro transfers with Day 6 good quality euploid embryos (though one of these embryos was twice biopsied and thawed), the first resulted in a chemical pregnancy age 35 and the second didn’t implant age 36.

I’m wondering whether I should also have a hysteroscopy to completely rule out any potential scarring caused by my D&C which may be causing our embryos to not implant.

Is a 3D saline sonogram as reliable at detecting uterine abnormalities as a hysteroscopy?

 

 

Answer:

You certainly are likely to have a secondary anatomical or immunologic implantation dysfunction…We should talk. I suggest you call my assistant, Patti Converse and set up an online consultation with me.

Implantation dysfunction is unfortunately often overlooked as an important cause of IVF failure. This is especially relevant in cases of unexplained IVF failure, recurrent pregnancy loss (RPL), and in women with underlying endo-uterine surface lesions, thickness of the uterine lining (endometrium) and /or immunologic factors.

IVF success rates have been improving over the last decade. The average live birth rate per embryo transfer in the U.S.A for women under 40y using their own eggs is currently better than 1:3 women. However, there is still a wide variation from program to program for IVF live birth rates, ranging from 20% to near 50%. Based upon these statistics, most women undergoing IVF in the United States require two or more attempts to have a baby. IVF practitioners in the United States commonly attribute the wide dichotomy in IVF success rates to variability in expertise of the various embryology laboratories. This is far from accurate. In fact, other factors such as wide variations in patient selection and the failure to develop individualized protocols for ovarian stimulation or to address those infectious, anatomical, and immunologic factors that influence embryo implantation are at least equally important.

About 80% of IVF failures are due to “embryo incompetency” that is largely due to aneuploidy      usually related to advancing age of the woman and is further influenced by other factors such as the protocol selected for ovarian stimulation, diminished ovarian reserve (DOR), and severe male factor infertility. However, in about 20% of dysfunctional cases embryo implantation is the cause of failure.

This section will focus on implantation dysfunction and IVF failure due to:

ANATOMICAL ENDO-UTERINE SURFACE LESIONS

            It has long been suspected that anatomical defects of the uterus might result in infertility. While the presence of uterine fibroids, in general, are unlikely to cause infertility, an association between their presence and infertility has been observed in cases where the myomas distort the uterine cavity or protrude through the endometrial lining.  Even small fibroids that lie immediately under the endometrium (submucous fibroids) and protrude into the uterine cavity have the potential to lower embryo implantation.  Multiple fibroids in the uterine wall (intramural fibroids) that encroach upon the uterine cavity can sometimes so compromise blood flow that estrogen delivery is impaired, and the endometrium is unable to thicken properly. This can usually be diagnosed by ultrasound examination during the proliferative phase of the menstrual cycle.  It is likely that any surface lesion in the uterine cavity, whether submucous fibroids, intrauterine adhesions a small endometrial or a placental polyp, has the potential to interfere with implantation by producing a local inflammatory response, not too dissimilar in nature from that which is caused by an intrauterine contraceptive device (IUD).       

            Clearly, since even small uterine lesions have the potential to adversely affect implantation, the high cost (financial, physical, and emotional) associated with IVF and related procedures, justifies the routine performance of diagnostic procedures such as an HSG, hysterosonogram (fluid ultrasound examination), or hysteroscopy prior to initiating IVF.  Identifiable uterine lesions that have the potential of impairing implantation usually require surgical intervention.  In most cases, dilatation and curettage (D & C) or hysteroscopic resection will suffice. Some cases might require the performance of a laparotomy.  Such intervention will often result in subsequent improvement of the endometrial response.      

Sonohysterography [Fluid ultrasonography (FUS)]: Fluid ultrasonography is a procedure whereby a sterile solution of saline is injected via a catheter through the cervix and into the uterine cavity. The fluid-distended cavity is examined by vaginal ultrasound for any irregularities that might point to surface lesions such as polyps, fibroid tumors, scarring, or a uterine septum. If performed by an expert, a FUS is highly effective in recognizing even the smallest lesion and can replace hysteroscopy under such circumstances. FUS is less expensive, less traumatic, and equally as effective as hysteroscopy. The only disadvantage lies in the fact that if a lesion is detected, it may require the subsequent performance of hysteroscopy to treat the problem anyway.

Hysteroscopy: Diagnostic hysteroscopy is an office procedure that is performed under intravenous sedation, general anesthesia, or paracervical block with minimal discomfort to the patient. This procedure involves the insertion of a thin, lighted, telescope like instrument known as a hysteroscope through the vagina and cervix into the uterus to fully examine the uterine cavity. The uterus is first distended with normal saline, which is passed through a sleeve adjacent to the hysteroscope. As is the case with FUS, diagnostic hysteroscopy facilitates examination of the inside of the uterus under direct vision for defects that might interfere with implantation. We have observed that approximately one in eight candidates for IVF have lesions that require attention prior to undergoing IVF in order to optimize the chances of a successful outcome. We strongly recommend that all patients undergo therapeutic surgery (usually by hysteroscopy) to correct the pathology prior to IVF.  Depending on the severity and nature of the pathology, therapeutic hysteroscopy may require general anesthesia and, in such cases, should be performed in an outpatient surgical facility or conventional operating room where facilities are available for laparotomy, a procedure in which an incision is made in the abdomen to expose the abdominal contents for diagnosis, or for surgery should this be required.       

THICKNESS OF THE UTERINE LINING (ENDOMETRIUM):

As far back as in 1989 we first reported on the finding that ultrasound assessment of the late proliferative phase endometrium can identify those candidates who are least likely to conceive. We noted that the ideal thickness of the endometrium at the time of ovulation or egg retrieval is >8 mm and that thinner linings are associated with decreased implantation rates.

More than 30 years ago we first showed that in normal and “stimulated” cycles, pre-ovulatory endometrial thickness and ultrasound appearance is predictive of embryo implantation (pregnancy) potential following ET. With conventional IVF and with FET, endometrial lining at the time of the “trigger shot” or with the initiation of progesterone needs to preferably be at least 8 mm in sagittal thickness with a triple line (trilaminar) appearance. Anything less than an 8mm endometrial thickness       is associated with a reduction in live birth rate per ET. An 8-9mm thickness represents a transitional measurement…a “gray zone”.  Hitherto, attempts to augment endometrial growth in women with poor endometrial linings by bolstering circulating estrogen blood levels (through the administration of increased doses of fertility drugs, aspirin administration and by supplementary estrogen therapy) yielded disappointing results.

            A “poor” uterine lining is usually the result of the innermost layer of endometrium (the basal or germinal endometrium from which endometrium grows) not being able to respond to estrogen by propagating an outer, “functional” layer thick enough to support optimal embryo implantation and development of a healthy placenta (placentation). The “functional” layer ultimately comprises 2/3 of the full endometrial thickness and is the layer that sheds with menstruation if no pregnancy occurs.

The main causes of a “poor” uterine lining are:

  • Damage to the basal endometrium because of:
  • Inflammation of the endometrium (endometritis) most commonly resulting from infected products left over following abortion, miscarriage, or birth
  • Surgical trauma due to traumatic uterine scraping, (i.e. due to an over-aggressive D & C)
  • Insensitivity of the basal endometrium to estrogen due to:
  • Prolonged, over-use/misuse of clomiphene citrate
  • Prenatal exposure to diethylstilbestrol (DES). This is a drug that was given to pregnant women in the 1960’s to help prevent miscarriage
  • Over-exposure of the uterine lining to ovarian male hormones (mainly testosterone): Older women, women with diminished ovarian reserve (poor responders) and women with polycystic ovarian syndrome -PCOS tend to have raised LH biological activity. This causes the connective tissue in the ovary (stroma/theca) to overproduce testosterone. The effect may be further exaggerated when certain methods for ovarian stimulation such as “flare” protocols and high dosages of Menopur are used in such cases.
  • Reduced blood flow to the basal endometrium: Examples include.
  • Multiple uterine fibroids – especially when these are present under the endometrium (submucosal)
  • Uterine adenomyosis (excessive, abnormal invasion of the uterine muscle by endometrial glands).

Vaginal Viagra: About 35 years ago, after reporting on the benefit of administering vaginal Sildenafil (Viagra) to women who had implantation dysfunction due to thin endometrial linings we announced the birth of the world’s first “Viagra baby.”  Viagra administered vaginally, but not orally, in affected women improves uterine blood flow causing more estrogen to be delivered to the basal endometrium and increasing the endometrial thickening.  Following vaginal administration, Viagra is rapidly absorbed and quickly reaches the uterine blood system in high concentrations. Thereupon it dilutes out as it is absorbed into the systemic circulation. This probably explains why treatment is virtually devoid of systemic side effects.  It is important to recognize that Viagra will NOT be effective in improving endometrial thickness in all cases. In fact, about one third of women treated fail to show any improvement. This is because in certain cases of thin uterine linings, the basal endometrium will have been permanently damaged and left unresponsive to estrogen. This happens in cases of severe endometrial damage due mainly to post-pregnancy endometritis (inflammation), chronic granulomatous inflammation due to uterine tuberculosis (hardly ever seen in the United States) and following extensive surgical injury to the basal endometrium (as sometimes occurs following over-zealous D&C’s).

  • Immunologic factors: These also play a role in IVF failure (see “Immunologic factors and Implantation” …see below.

IMMUNOLOGIC IMPLANTATION DYSFUNCTION (IID)

            Currently, with few exceptions, practitioners of assisted reproduction tend to attribute “unexplained and/or repeated” IVF failure(s), almost exclusively to poor embryo quality, advocating adjusted protocols for ovarian stimulation and/or gamete and embryo preparation as a potential remedy. The idea, having failed IVF, that all it takes to ultimately succeed is to keep trying the same recipe is over-simplistic.

            The implantation process begins six or seven days after fertilization of the egg. At this time, specialized embryonic cells (i.e., trophoblasts), that later become the placenta begin growing into the uterine lining. When the trophoblast and the uterine lining meet, they, along with immune cells in the lining, become involved in a “cross talk” through mutual exchange of hormone-like substances called cytokines. Because of this complex immunologic interplay, the uterus can foster the embryo’s successful growth. Thus, from the earliest stage, the trophoblast establishes the very foundation for the nutritional, hormonal and respiratory interchange between mother and baby.  In this manner, the interactive process of implantation is not only central to survival in early pregnancy but also to the quality of life after birth.

There is an ever growing realization, recognition, and acceptance of the fact that uterine immunologic dysfunction can lead to immunologic implantation dysfunction (IID) with “unexplained” infertility, IVF failure, and recurrent pregnancy loss (RPL).

DIAGNOSIS 

Because immunologic problems may lead to implantation failure, it is important to properly evaluate women with risk factors such as:

  • Unexplained or recurrent IVF failures
  • Unexplained infertility or a family history of autoimmune diseases (e.g., rheumatoid arthritis, lupus erythematosus and hypothyroidism).
  • Recurrent Pregnancy Loss (RPL)
  • Endometriosis
  • A personal or family history of autoimmune conditions, e.g., Rheumatoid Arthritis, Lupus erythematosus, autoimmune hypothyroidism (Hashimoto’s disease) etc.

            Considering its importance, it is not surprising that the failure of a properly functioning immunologic interaction during implantation has been implicated as a cause of recurrent miscarriage, late pregnancy fetal loss, IVF failure and infertility. A partial list of immunologic factors that may be involved in these situations includes:

  • Antiphospholipid antibodies (APA)
  • Antithyroid antibodies (ATA/AMA)
  • Activated natural killer cells (NKa)

ACTIVATED NATURAL KILLER CELLS (NKa):

Following ovulation and during early pregnancy, NK cells and T-cells comprise more than 80% of the lymphocyte-immune cells that frequent the uterine lining. These lymphocytes (white blood cells) journey from the bone marrow to the uterus and under hormonal regulation, proliferate there. After exposure to progesterone (due to induced /spontaneous exogenous administration), they begin to produce TH-1 and TH-2 cytokines. TH-2 cytokines are humoral in nature and induce the trophoblast (“root system of the embryo”) to permeate the uterine lining while TH-I cytokines induce a process referred to as apoptosis (cell suicide) thereby confining placental development to the inner part of the uterus. Optimal placental development (placentation) mandates that there be a balance between TH1 and TH-2 cytokines. Most of the cytokine production originates from NK cells (rather than from cytotoxic T-cells/Lymphocytes (CTL)). Excessive production/release of TH-1 cytokines, is toxic to the trophoblast and to endometrial cells, leading to programmed death/suicide (apoptosis) and subsequently to IID.

Functional NK cells reach a maximal concentration in the endometrium by about t day 6-7 days after exposure to progesterone …. This timing corresponds with when the embryo implants into the uterine lining (endometrium).

It is important to bear in mind that measurement of the concentration of blood NK cells has little or no relevance when it comes to assessing NK cell activation (NKa). Rather, it is the NK cell activation that matters. In fact, there are certain conditions (such as with endometriosis) where the NK cell blood concentration is below normal, but NK cell activation is markedly increased.

There are several methods by which NK cell activation (cytotoxicity) can be assessed in the laboratory. Methods such as immunohistochemical assessment of uterine NK cells and/or through measurement of uterine or blood TH-1 cytokines. However, the K-562 target cell blood test still remains the gold standard. With this test, NK cells, isolated from the woman’s blood using Flow Cytometry are incubated in the presence of specific “target cells”. The percentage (%) of “target cells” killed is then quantified. More than 12% killing suggests a level of NK cell activation that usually requires treatment.

Currently, there are less than a half dozen Reproductive Immunology Reference Laboratories in the U.S.A that are capable of performing the K-562 target cell test reliably.

There exists a pervasive but blatant misconception on the part of many, that the addition of IL or IVIg to a concentration of NK cells could have an immediate down-regulatory effect on NK cell activity. Neither IVIg nor IL is capable of significantly suppressing already activated “functional NK cells”. They are believed to work through “regulating” NK cell progenitors which only thereupon will start to propagate down-regulated NK cells. Thus, testing for a therapeutic effect would require that the IL/IVIg infusion be done about 14 days prior to ovulation or progesterone administration…  in order to allow for a sufficient number of normal (non-activated) “functional” NK cell” to be present at the implantation site when the embryos are transferred.

Failure to recognize this reality has, in our opinion, established an erroneous demand by practicing IVF doctors, that Reproductive Immunology Reference Laboratories report on NK cell activity before and again, immediately following laboratory exposure to IVIg and/or IL in different concentrations. Allegedly, this is to allow the treating physician to report back to their patient(s) on whether an IL or IVIG infusion will be effective in downregulating their Nka.  But, since already activated NK cells (NKa) cannot be deactivated in the laboratory, effective NKa down-regulation can only be adequately accomplished through deactivation of NK cell “progenitors /parental” NK cells in order to allow them thereupon, to s propagate normal “functional” NK cells and his takes about 10-14 days, such practice would be of little clinical benefit. This is because even if blood were to be drawn 10 -14 days after IL/IVIg treatment it would require at least an additional 10 -14days to receive results from the laboratory, by which time it would be far too late to be of practical advantage.

ANTIPHOSPHOLIPID ANTIBODIES:

 Many women who experience “unexplained” IVF failure, women with RPL, those with a personal or family history of autoimmune diseases such as lupus erythematosus, rheumatoid arthritis, scleroderma, and dermatomyositis (etc.)  as well as women who have endometriosis (“silent” or overt) test positive for APAs. More than 30 years ago, we were the first to propose that women who test positive for APA’s be treated with a mini-dose heparin to improve IVF implantation and thus birth rates. This approach was based upon research that suggested that heparin repels APAs from the surface of the trophoblast (the embryo’s “root system) thereby reducing its ant-implantation effects.  We subsequently demonstrated that such therapy only improved IVF outcome in women whose APAs were directed against two specific IgG and/or IgM phospholipids [i.e., phosphatidylethanolamine (PE) and phosphatidylserine (PS)].  More recently low dosage heparin therapy has been supplanted using longer acting low molecular weight heparinoids such as Lovenox and Clexane.   It is very possible that APAs alone do not cause IID but that their presence might help to identify a population at risk due to concomitant activation of uterine natural killer cells (Nka) which through excessive TH-1 cytokine production causes in IID: This is supported by the following observations:

  • The presence of female APAs in cases of male factor cases appears to bear no relationship to IID.
  • Only APA positive women who also test positive for abnormal NK activity appear to benefit from selective immunotherapy with intralipid/IVIg/ steroids.
  • Most APA positive women who have increased NK cell activity also harbor IgG or IgM phosphatidylethanolamine (PE) and phosphatidylserine (PS) antibodies.

ANTITHYROID ANTIBODIES: (ATA).

 A clear relationship has been established between ATA and reproductive failure (especially recurrent miscarriage and infertility).

Between 2% and 5% of women of the childbearing age have reduced thyroid hormone activity (hypothyroidism). Women with hypothyroidism often manifest with reproductive failure i.e., infertility, unexplained (often repeated) IVF failure, or recurrent pregnancy loss (RPL). The condition is 5-10 times more common in women than in men. In most cases hypothyroidism is caused by damage to the thyroid gland resulting from of thyroid autoimmunity (Hashimoto’s disease) caused by damage done to the thyroid gland by antithyroglobulin and antimicrosomal auto-antibodies. 

The increased prevalence of hypothyroidism and thyroid autoimmunity (TAI) in women is likely the result of a combination of genetic factors, estrogen-related effects, and chromosome X abnormalities.  This having been said, there is significantly increased incidence of thyroid antibodies in non-pregnant women with a history of infertility and recurrent pregnancy loss and thyroid antibodies can be present asymptomatically in women without them manifesting with overt clinical or endocrinologic evidence of thyroid disease. In addition, these antibodies may persist in women who have suffered from hyper- or hypothyroidism even after normalization of their thyroid function by appropriate pharmacological treatment. The manifestations of reproductive dysfunction thus seem to be linked more to the presence of thyroid autoimmunity (TAI) than to clinical existence of hypothyroidism and treatment of the latter does not routinely result in a subsequent improvement in reproductive performance.

It follows, that if antithyroid autoantibodies are associated with reproductive dysfunction they may serve as useful markers for predicting poor outcome in patients undergoing assisted reproductive technologies.

Some years back, I reported on the fact that 47% of women who harbor thyroid autoantibodies, regardless of the absence or presence of clinical hypothyroidism, have activated uterine natural killer cells (NKa) cells and cytotoxic lymphocytes (CTL) and that such women often present with reproductive dysfunction. We demonstrated that appropriate immunotherapy with IVIG or intralipid (IL) and steroids, subsequently often results in a significant improvement in reproductive performance in such cases.

The fact that almost 50% of women who harbor antithyroid antibodies do not have activated CTL/NK cells suggests that it is NOT the antithyroid antibodies themselves that cause reproductive dysfunction. The activation of CTL and NK cells that occurs in half of the cases with TAI is probably an epiphenomenon with the associated reproductive dysfunction being due to CTL/NK cell activation that damages the early “root system” (trophoblast) of the implanting embryo. We have shown that treatment of those women who have thyroid antibodies + NKa/CTL using IL/steroids, improves subsequent reproductive performance while women with thyroid antibodies who do not harbor NKa/CTL do not require or benefit from such treatment

TEATMENT OF IID:

The mainstay of treatment involves the selective use of:

  • Intralipid (IL) infusion
  • IVIg therapy
  • Corticosteroids (Prednisone/dexamethasone)
  • Heparinoids (Lovenox/Clexane)

Intralipid (IL) Therapy:  IL is a suspension of soybean lipid droplets in water and is primarily used as source of parenteral nutrition. When administered intravenously, IL provides essential fatty acids, linoleic acid (LA), an omega-6 fatty acid, and alpha-linolenic acid (ALA), an omega-3 fatty acid.     

It is thought that fatty acids within the emulsion serve as ligands that activate peroxisome proliferator-activated receptors (PPARs) expressed by the NK cells. This is believed to decrease NK cell cytotoxic activity, and thereby enhance implantation A growing number of IVF programs, including ours, perform egg retrieval under conscious sedation using Propofol, a short acting hypnotic agent.

            Whatever the exact mechanism of action might be, Intralipid acts primarily to suppress NK cell over-production of TH-I cytokines. It exerts a modulating effect on certain immune cellular mechanisms largely by down-regulating cytotoxic /activated natural killer cells (NKa). This effect is enhanced through the concomitant administration of corticosteroids such as dexamethasone, prednisolone and prednisone which augment immune modulation of T cells. The combined effect of IL + steroid therapy suppresses pro-inflammatory cellular TH1 cytokines such as interferon gamma and TNF-alpha that are produced in excess by activated NK cells and cytotoxic lymphocytes/T-cells (CTL).   IL will, in about 80% of cases, successfully down-regulate activated natural killer cells (NKa) over a period of 2-3 weeks. It is likely to be just as effective as IVIg in this respect but at a fraction of the cost and with a far lower incidence of side-effects. Its effect lasts for ~ 4-6 weeks when administered in early pregnancy.

Intralipid is a suspension of soybean lipid droplets in water and is primarily used as source of parenteral nutrition. When administered intravenously, IL provides essential fatty acids, linoleic acid (LA), an omega-6 fatty acid, and alpha-linolenic acid (ALA), an omega-3 fatty acid.     

It is thought that fatty acids within the emulsion serve as ligands that activate peroxisome proliferator-activated receptors (PPARs) expressed by the NK cells. This is believed to decrease NK cell cytotoxic activity, and thereby enhance implantation A growing number of IVF programs, including ours, perform egg retrieval under conscious sedation using Propofol, a short acting hypnotic agent.

            Whatever the exact mechanism of action might be, Intralipid acts primarily to suppress NK cell over-production of TH-I cytokines. It exerts a modulating effect on certain immune cellular mechanisms largely by down-regulating cytotoxic /activated natural killer cells (NKa). This effect is enhanced through the concomitant administration of corticosteroids such as dexamethasone, prednisolone and prednisone which augment immune modulation of T cells. The combined effect of IL + steroid therapy suppresses pro-inflammatory cellular TH1 cytokines such as interferon gamma and TNF-alpha that are produced in excess by activated NK cells and cytotoxic lymphocytes/T-cells (CTL).   IL will, in about 80% of cases, successfully down-regulate activated natural killer cells (NKa) over a period of 2-3 weeks. It is likely to be just as effective as IVIg in this respect but at a fraction of the cost and with a far lower incidence of side-effects. Its effect lasts for ~ 4-6 weeks when administered in early pregnancy.

            Can laboratory testing be used to assess for an immediate effect of IL on Nka suppression?  Since the downregulation of NKa through IL (or IVIg) therapy can take several weeks to become measurable, it follows that there is really no benefit in trying to assess the potential efficacy of such treatment by retesting NKa in the laboratory after adding IL (or IVIg) to the cells being tested.

IVIg Therapy:  Until about a decade ago, the only effective and available way (in the US) to down-regulate activated NK cells was through the intravenous administration of a blood product known as immunoglobulin-G (IVIg). The fear (albeit unfounded) that the administration of this product might lead to the transmission of viral infections such as HIV and hepatitis C, plus the high cost of IVIG along with the fact that significant side effects occurred about 20% of the time, led to bad press and bad publicity for the entire field of reproductive immunology. It was easier for RE’s to simply say “I don’t believe IVIg works” and thereby avoid risk and bad publicity. But the thousands of women who had babies because of NK cell activity being downregulated through its use, attests to IVIg’s efficacy. But those of us who felt morally obligated to many desperate patients who would not conceive without receiving IVIg were facing an uphill battle. The bad press caused by fear mongering took its toll and spawned a malicious controversy. It was only through the introduction of IL less (about 15-20 years ago ), that the tide began to turn in favor of those patients who required low cost, safe and effective immunotherapy to resolve their IID.

 Corticosteroid Therapy (e.g., Prednisone, and Dexamethasone): Corticosteroid therapy has become a mainstay in the treatment of most women undergoing IVF. It is believed by most to enhance implantation due to an overall immunomodulatory effect. Corticosteroids reduce TH-1 cytokine production by CTL. When given in combination with IL or IVIG they augment the implantation process. The prednisone or dexamethasone therapy must commence (along with IL/IVIg) 10-14 days prior to egg retrieval and continue until pregnancy is discounted or until the 10th week of pregnancy.

 Heparinoid Therapy: There is compelling evidence that the subcutaneous administration of low molecular heparin (Clexane, Lovenox) once daily, (starting with the onset of ovarian stimulation) can improve IVF birthrate in women who test positive for APAs and might prevent later pregnancy loss when used to treat certain thrombophilias (e.g., homozygous MTHFR mutation)

What About Baby Aspirin? In our opinion, aspirin has little (if any) value when it comes to IID, and besides, it could even reduce the chance of success. The reason for this is that aspirin thins the blood and increases the potential to bleed. This effect can last for up to a week and could complicate an egg retrieval procedure or result in “concealed” intrauterine bleeding at the time of embryo transfer, thereby potentially compromising IVF success.

TH-1 Cytokine Blockers (Enbrel, Humira): TH-1 cytokine blockers, (Enbrel and Humira) are in our opinion relatively ineffective in the IVF setting. There has to date been no convincing data to support their use. However, these blockers could have a role in the treatment of a threatened miscarriage thought to be due to CTL/NK activation, but not for IVF. The reason is that the very initial phase of implantation requires a cellular response involving TH-1 cytokines. To block them completely (rather than simply restore a TH-1:TH-2 balance as occurs with IL therapy) so very early on could compromise rather than benefit implantation.

Leukocyte Immunization Therapy (LIT): The subcutaneous injection of the male partner’s lymphocytes to the mother is thought to enhance the ability for the mother’s decidua (uterus) to recognize the DQ alpha matching embryo as “self” or “friend” and thereby avert its rejection. LIT has been shown to up-regulate Treg cells and thus down-regulate NK cell activation thereby improving decidual TH-1:TH-2 balance. Thus, there could be a therapeutic benefit from such therapy. However, the same benefit can be achieved through the use of IL plus corticosteroids. Besides, IL is much less expensive, and the use of LIT is prohibited by law in the U.S.A.

There are two categories of immunologic implantation dysfunction (IID) linked to NK cell activation (NKa).

Autoimmune Implantation Dysfunction: Here, the woman will often have a personal or family history of autoimmune conditions such as Rheumatoid arthritis, Lupus Erythematosus, and thyroid autoimmune activity (e.g., Hashimoto’s disease) etc. Autoimmune as well as in about one third of cases of endometriosis, regardless of severity.  Autoimmune sometimes also occurs in the absence of a personal or family history of autoimmune disease.

When it comes to treating  NKa in  IVF cases complicated by autoimmune implantation dysfunction,  the combination of daily oral dexamethasone commencing with the onset of ovarian stimulation and continuing until the 10th week of pregnancy, combined with an initial infusion of IL (100ml, 20% Il dissolved in 500cc of saline solution, 10-14  days prior to PGT-normal embryo transfer and repeated once more (only), as  soon as the blood pregnancy test is positive), the anticipated chance of a viable pregnancy occurring within 2 completed IVF attempts (including fresh + frozen ET’s)  in women under 39Y (who have normal ovarian reserve)  is approximately  65%.

Alloimmune Implantation Dysfunction: Here, NK cell activation results from uterine exposure to an embryo derived through fertilization by a spermatozoon that shares certain genotypic (HLA/DQ alpha) similarities with that of the embryo recipient.

Partial DQ alpha/HLA match:  Couples who upon genotyping are shown to share only one DQ alpha/HLA gene are labeled as having a “partial match”. The detection of a “partial match” in association with NKa puts the couple at a considerable disadvantage with regard to IVF outcome. It should be emphasized however, that in the absence of associated Nka, DQ alpha/HLA matching whether “partial” or “total (see below) will NOT cause an IID. Since we presently have no way of determining which embryo carries a matching paternal DQ alpha gene, it follows that each embryo transferred will have about half the chance of propagating a viable pregnancy. Treatment of a partial DQ alpha/HLA match (+ Nka) involves the same IL, infusion as for autoimmune-Nka with one important caveat, namely that here we prescribe oral prednisone as adjunct therapy (rather than dexamethasone) and the IL infusion is repeated every 2-4 weeks following the diagnosis of pregnancy and continued until the 24th week of gestation. Additionally, (as alluded to elsewhere) in such cases we transfer a single (1) embryo at a time. This is because, the likelihood is that one out of two embryos will “match” and we are fearful that if we transfer >1 embryo, and one transferred embryos “matches” it could cause further activation of uterine NK cells and so prejudice the implantation of all transferred embryos. Here it should be emphasized that if associated with Nka, a matching embryo will still be at risk of rejection even in the presence of Intralipid (or IVIg) therapy.

Total (complete) DQ alpha Match:   Here the husband’s DQ alpha genotype matches both of that of his partner’s. While this occurs very infrequently, a total alloimmune (DQ alpha) match with accompanying Nka, means that the chance of a viable pregnancy resulting in a live birth at term, is unfortunately greatly diminished.  Several instances in our experience have required the use of a gestational surrogate.

It is indeed unfortunate that so many patients are being denied the ability to go from “infertility to family” simply because (for whatever reason) so many reproductive specialists refuse to embrace the role of immunologic factors in the genesis of intractable reproductive dysfunction. Hopefully this will change, and the sooner the better.

______________________________________________________________________

I urge you to  visit my website at  www.SherIVF.com. Then go to my Blog and access the “search bar”. Type in the titles of any/all of the articles listed below, one by one. “Click” and you will immediately be taken to those you select.  Please also take the time to post any questions or comments with the full expectation that I will (as always) respond promptly.

 

  • A Fresh Look at the Indications for IVF
  • The IVF Journey: The importance of “Planning the Trip” Before Taking the Ride”
  • Controlled Ovarian Stimulation (COS) for IVF: Selecting the ideal protocol
  • IVF: Factors Affecting Egg/Embryo “competency” during Controlled Ovarian Stimulation(COS)
  • The Fundamental Requirements For Achieving Optimal IVF Success
  • Use of GnRH Antagonists (Ganirelix/Cetrotide/Orgalutron) in IVF-Ovarian Stimulation Protocols.
  • Human Growth Hormone Administration in IVF: Does it Enhances Egg/Embryo Quality and Outcome?
  • IVF and the use of Supplementary Human Growth Hormone (HGH) : Is it Worth Trying and who needs it?
  • The BCP: Does Launching a Cycle of Controlled Ovarian Stimulation (COS). Coming off the BCP Compromise Response?
  • Blastocyst Embryo Transfers Should be the Standard of Care in IVF
  • Anti Mullerian Hormone (AMH) Measurement to Assess Ovarian Reserve and Design the Optimal Protocol for Controlled Ovarian Stimulation (COS) in IVF.
  • IVF: Approach to Selecting the Best Embryos for Transfer to the Uterus.
  • Fresh versus Frozen Embryo Transfers (FET) Enhance IVF Outcome
  • Frozen Embryo Transfer (FET): A Rational Approach to Hormonal Preparation and How new Methodology is Impacting IVF.
  • Genetically Testing Embryos for IVF
  • Staggered IVF
  • Staggered IVF with PGS- Selection of “Competent” Embryos Greatly Enhances the Utility & Efficiency of IVF.
  • Preimplantation Genetic Testing (PGS) in IVF: It should be Used Selectively and NOT be Routine.
  • IVF: Selecting the Best Quality Embryos to Transfer
  • Preimplantation Genetic Sampling (PGS) Using: Next Generation Gene Sequencing (NGS): Method of Choice.
  • PGS in IVF: Are Some Chromosomally abnormal Embryos Capable of Resulting in Normal Babies and Being Wrongly Discarded?
  • PGS and Assessment of Egg/Embryo “competency”: How Method, Timing and Methodology Could Affect Reliability
  • Endometrial Receptivity Array (ERA): Is There an actual “There, There”?
  • IVF Failure and Implantation Dysfunction:
  • Diagnosing and Treating Immunologic Implantation Dysfunction (IID)
  • The Role of Immunologic Implantation Dysfunction (IID) & Infertility (IID):PART 1-Background
  • Immunologic Implantation Dysfunction (IID) & Infertility (IID):PART 2- Making a Diagnosis
  • Immunologic Dysfunction (IID) & Infertility (IID):PART 3-Treatment
  • Thyroid autoantibodies and Immunologic Implantation Dysfunction (IID)
  • Immunologic Implantation Dysfunction: Importance of Meticulous Evaluation and Strategic Management:(Case Report
  • Intralipid and IVIG therapy: Understanding the Basis for its use in the Treatment of Immunologic Implantation Dysfunction (IID)
  • Intralipid (IL) Administration in IVF: It’s Composition; How it Works; Administration; Side-effects; Reactions and Precautions
  • Natural Killer Cell Activation (NKa) and Immunologic Implantation Dysfunction in IVF: The Controversy!
  • Endometrial Thickness, Uterine Pathology and Immunologic Factors
  • Vaginally Administered Viagra is Often a Highly Effective Treatment to Help Thicken a Thin Uterine Lining
  • A Thin Uterine Lining: Vaginal Viagra is Often the Answer (update)
  • Cervical Ureaplasma Urealyticum Infection: How can it Affect IUI/IVF Outcome?
  • The Role of Nutritional Supplements in Preparing for IVF
  • The Basic Infertility Work-Up
  • Defining and Addressing an Abnormal Luteal Phase
  • Male Factor Infertility
  • Routine Fertilization by Intracytoplasmic Sperm Injection (ICSI): An Argument in Favor
  • Hormonal Treatment of Male Infertility
  • Hormonal Treatment of Male Infertility
  • Antisperm Antibodies, Infertility and the Role of IVF with Intracytoplasmic Sperm Injection (ICSI)
  • Endometriosis and Infertily
  • Endometriosis and Immunologic Implantation Dysfunction (IID) and IVF
  • Endometriosis and Infertility: Why IVF Rather than IUI or Surgery Should be the Treatment of Choice.
  • Endometriosis and Infertility: The Influence of Age and Severity on Treatment Options
  • Early -Endometriosis-related Infertility: Ovulation Induction (with or without Intrauterine Insemination) and Reproductive Surgery  Versus IVF
  • Deciding Between Intrauterine Insemination (IUI) and In Vitro Fertilization (IVF).
  • Intrauterine Insemination (IUI): Who Needs it & who Does Not: Pro’s & Con’s!IUI-Reflecting upon its Use and Misuse: Time for a Serious “Reality Check
  • Mode of Action, Indications, Benefits, Limitations and Contraindications for its ue
  • Clomiphene Induction of Ovulation: Its Use and Misuse!

 

 

 

Name: Ola K

Dear Doctor Sher,

After many years of trying and 13 chemical pregnancies finally I did some immune tests this month:
(Partner) 01.02 02.01
(Me) 01.02 01.02

19 17.9 (H) (2.00 – 12.00)
19 +cells 5+ 11.3 (H) (5.00 – 10.00)
TNF-a: IL-10 (CD3+CD4+) 16.6 (13.2 – 30.6)
IFN-g: IL-10 (CD3+CD4+) 4.7 (L) (5.8 – 20.5)
LAD weak positive
ANA weak positive
High Ant. TG_ Hashimoto

I see that there is a partial match 50% risk 😔With these results can we say that we found reason/s for my recurrent pregnancy losses?
Do you think in this case IVF NGS, prednisone, IL, clexane and LIT might work? LIT is only once or needs to be repeated after some time?

Donor eggs in this case will make a difference?

Thank you very much for your kind support
Ola

Answer:

WE really do need to talk! Please call Patti Converse (my assistant, at 702-533-2691 and set up an online consultation with me.

You have a partial DQ alpha match with your husband. This is only significant if in addition, you have activated NK cells (which needs to be tested). Blood cytokines are not as helpful.AND if you are NKa+_,  use of an egg donor will not help, in my opinion!

Currently, with few exceptions, practitioners of assisted reproduction tend to attribute “unexplained and/or repeated” IVF failure(s), almost exclusively to poor embryo quality, advocating adjusted protocols for ovarian stimulation and/or gamete and embryo preparation as a potential remedy. The idea, having failed IVF, that all it takes to ultimately succeed is to keep trying the same recipe is over-simplistic.

The implantation process begins six or seven days after fertilization of the egg. At this time, specialized embryonic cells (i.e., trophoblasts), that later become the placenta begin growing into the uterine lining. When the trophoblast and the uterine lining meet, they, along with immune cells in the lining, become involved in a “cross talk” through mutual exchange of hormone-like substances called cytokines. Because of this complex immunologic interplay, the uterus can foster the embryo’s successful growth. Thus, from the earliest stage, the trophoblast establishes the very foundation for the nutritional, hormonal and respiratory interchange between mother and baby.  In this manner, the interactive process of implantation is not only central to survival in early pregnancy but also to the quality of life after birth.

There is an ever growing realization, recognition, and acceptance of the fact that uterine immunologic dysfunction can lead to immunologic implantation dysfunction (IID) with “unexplained” infertility, IVF failure, and recurrent pregnancy loss (RPL).

DIAGNOSIS 

Because immunologic problems may lead to implantation failure, it is important to properly evaluate women with risk factors such as:

  • Unexplained or recurrent IVF failures
  • Unexplained infertility or a family history of autoimmune diseases (e.g., rheumatoid arthritis, lupus erythematosus and hypothyroidism).
  • Recurrent Pregnancy Loss (RPL)
  • Endometriosis
  • A personal or family history of autoimmune conditions, e.g., Rheumatoid Arthritis, Lupus erythematosus, autoimmune hypothyroidism (Hashimoto’s disease) etc.

Considering its importance, it is not surprising that the failure of a properly functioning immunologic interaction during implantation has been implicated as a cause of recurrent miscarriage, late pregnancy fetal loss, IVF failure and infertility. A partial list of immunologic factors that may be involved in these situations includes:

  • Antiphospholipid antibodies (APA)
  • Antithyroid antibodies (ATA/AMA)
  • Activated natural killer cells (NKa)

ACTIVATED NATURAL KILLER CELLS (NKa):

Following ovulation and during early pregnancy, NK cells and T-cells comprise more than 80% of the lymphocyte-immune cells that frequent the uterine lining. These lymphocytes (white blood cells) journey from the bone marrow to the uterus and under hormonal regulation, proliferate there. After exposure to progesterone (due to induced /spontaneous exogenous administration), they begin to produce TH-1 and TH-2 cytokines. TH-2 cytokines are humoral in nature and induce the trophoblast (“root system of the embryo”) to permeate the uterine lining while TH-I cytokines induce a process referred to as apoptosis (cell suicide) thereby confining placental development to the inner part of the uterus. Optimal placental development (placentation) mandates that there be a balance between TH1 and TH-2 cytokines. Most of the cytokine production originates from NK cells (rather than from cytotoxic T-cells/Lymphocytes (CTL)). Excessive production/release of TH-1 cytokines, is toxic to the trophoblast and to endometrial cells, leading to programmed death/suicide (apoptosis) and subsequently to IID.

Functional NK cells reach a maximal concentration in the endometrium by about t day 6-7 days after exposure to progesterone …. This timing corresponds with when the embryo implants into the uterine lining (endometrium).

It is important to bear in mind that measurement of the concentration of blood NK cells has little or no relevance when it comes to assessing NK cell activation (NKa). Rather, it is the NK cell activation that matters. In fact, there are certain conditions (such as with endometriosis) where the NK cell blood concentration is below normal, but NK cell activation is markedly increased.

There are several methods by which NK cell activation (cytotoxicity) can be assessed in the laboratory. Methods such as immunohistochemical assessment of uterine NK cells and/or through measurement of uterine or blood TH-1 cytokines. However, the K-562 target cell blood test still remains the gold standard. With this test, NK cells, isolated from the woman’s blood using Flow Cytometry are incubated in the presence of specific “target cells”. The percentage (%) of “target cells” killed is then quantified. More than 12% killing suggests a level of NK cell activation that usually requires treatment.

Currently, there are less than a half dozen Reproductive Immunology Reference Laboratories in the U.S.A that are capable of performing the K-562 target cell test reliably.

There exists a pervasive but blatant misconception on the part of many, that the addition of IL or IVIg to a concentration of NK cells could have an immediate down-regulatory effect on NK cell activity. Neither IVIg nor IL is capable of significantly suppressing already activated “functional NK cells”. They are believed to work through “regulating” NK cell progenitors which only thereupon will start to propagate down-regulated NK cells. Thus, testing for a therapeutic effect would require that the IL/IVIg infusion be done about 14 days prior to ovulation or progesterone administration…  in order to allow for a sufficient number of normal (non-activated) “functional” NK cell” to be present at the implantation site when the embryos are transferred.

Failure to recognize this reality has, in our opinion, established an erroneous demand by practicing IVF doctors, that Reproductive Immunology Reference Laboratories report on NK cell activity before and again, immediately following laboratory exposure to IVIg and/or IL in different concentrations. Allegedly, this is to allow the treating physician to report back to their patient(s) on whether an  IL or IVIG infusion will be effective in down-regulating their Nka.  But, since already activated NK cells (NKa) cannot be deactivated in the laboratory, effective NKa down-regulation can only be adequately accomplished through deactivation of NK cell “progenitors /parental” NK cells in order too allow them thereupon, to s propagate normal “functional” NK cells and his takes about 10-14 days, such practice  would be of little clinical benefit. This is because even if blood were to be drawn 10 -14 days after IL/IVIg treatment it would require at least  an additional 10 -14days to receive results from the laboratory, by which time it would be far too late to be of practical advantage.

ANTIPHOSPHOLIPID ANTIBODIES:

 Many women who experience “unexplained” IVF failure, women with RPL, those with a personal or family history of autoimmune diseases such as lupus erythematosus, rheumatoid arthritis, scleroderma, and dermatomyositis (etc.)  as well as women who have endometriosis (“silent” or overt) test positive for APAs. More than 30 years ago, we were the first to propose that women who test positive for APA’s be treated with a mini-dose heparin to improve IVF implantation and thus birth rates. This approach was based upon research that suggested that heparin repels APAs from the surface of the trophoblast (the embryo’s “root system) thereby reducing its ant-implantation effects.  We subsequently demonstrated that such therapy only improved IVF outcome in women whose APAs were directed against two specific IgG and/or IgM phospholipids [i.e., phosphatidylethanolamine (PE) and phosphatidylserine (PS)].  More recently low dosage heparin therapy has been supplanted using longer acting low molecular weight heparinoids such as Lovenox and Clexane.   It is very possible that APAs alone do not cause IID but that their presence might help to identify a population at risk due to concomitant activation of uterine natural killer cells (Nka) which through excessive TH-1 cytokine production causes in IID: This is supported by the following observations:

  • The presence of female APAs in cases of male factor cases appears to bear no relationship to IID.
  • Only APA positive women who also test positive for abnormal NK activity appear to benefit from selective immunotherapy with intralipid/IVIg/ steroids.
  • Most APA positive women who have increased NK cell activity also harbor IgG or IgM phosphatidylethanolamine (PE) and phosphatidylserine (PS) antibodies.

ANTITHYROID ANTIBODIES: (ATA).

 A clear relationship has been established between ATA and reproductive failure (especially recurrent miscarriage and infertility).

Between 2% and 5% of women of the childbearing age have reduced thyroid hormone activity (hypothyroidism). Women with hypothyroidism often manifest with reproductive failure i.e., infertility, unexplained (often repeated) IVF failure, or recurrent pregnancy loss (RPL). The condition is 5-10 times more common in women than in men. In most cases hypothyroidism is caused by damage to the thyroid gland resulting from of thyroid autoimmunity (Hashimoto’s disease) caused by damage done to the thyroid gland by antithyroglobulin and antimicrosomal auto-antibodies. 

The increased prevalence of hypothyroidism and thyroid autoimmunity (TAI) in women is likely the result of a combination of genetic factors, estrogen-related effects, and chromosome X abnormalities.  This having been said, there is significantly increased incidence of thyroid antibodies in non-pregnant women with a history of infertility and recurrent pregnancy loss and thyroid antibodies can be present asymptomatically in women without them manifesting with overt clinical or endocrinologic evidence of thyroid disease. In addition, these antibodies may persist in women who have suffered from hyper- or hypothyroidism even after normalization of their thyroid function by appropriate pharmacological treatment. The manifestations of reproductive dysfunction thus seem to be linked more to the presence of thyroid autoimmunity (TAI) than to clinical existence of hypothyroidism and treatment of the latter does not routinely result in a subsequent improvement in reproductive performance.

It follows, that if antithyroid autoantibodies are associated with reproductive dysfunction they may serve as useful markers for predicting poor outcome in patients undergoing assisted reproductive technologies.

Some years back, I reported on the fact that 47% of women who harbor thyroid autoantibodies, regardless of the absence or presence of clinical hypothyroidism, have activated uterine natural killer cells (NKa) cells and cytotoxic lymphocytes (CTL) and that such women often present with reproductive dysfunction. We demonstrated that appropriate immunotherapy with IVIG or intralipid (IL) and steroids, subsequently often results in a significant improvement in reproductive performance in such cases.

The fact that almost 50% of women who harbor antithyroid antibodies do not have activated CTL/NK cells suggests that it is NOT the antithyroid antibodies themselves that cause reproductive dysfunction. The activation of CTL and NK cells that occurs in half of the cases with TAI is probably an epiphenomenon with the associated reproductive dysfunction being due to CTL/NK cell activation that damages the early “root system” (trophoblast) of the implanting embryo. We have shown that treatment of those women who have thyroid antibodies + NKa/CTL using IL/steroids, improves subsequent reproductive performance while women with thyroid antibodies who do not harbor NKa/CTL do not require or benefit from such treatment

TEATMENT OF IID:

The mainstay of treatment involves the selective use of :

  • Intralipid (IL) infusion
  • IVIg therapy
  • Corticosteroids (Prednisone/dexamethasone)
  • Heparinoids (Lovenox/Clexane)

Intralipid (IL) Therapy:  IL is a suspension of soybean lipid droplets in water and is primarily used as source of parenteral nutrition. When administered intravenously, IL provides essential fatty acids, linoleic acid (LA), an omega-6 fatty acid, and alpha-linolenic acid (ALA), an omega-3 fatty acid.     

It is thought that fatty acids within the emulsion serve as ligands that activate peroxisome proliferator-activated receptors (PPARs) expressed by the NK cells. This is believed to decrease NK cell cytotoxic activity, and thereby enhance implantation A growing number of IVF programs, including ours, perform egg retrieval under conscious sedation using Propofol, a short acting hypnotic agent.

            Whatever the exact mechanism of action might be, Intralipid acts primarily to suppress NK cell over-production of TH-I cytokines. It exerts a modulating effect on certain immune cellular mechanisms largely by down-regulating cytotoxic /activated natural killer cells (NKa). This effect is enhanced through the concomitant administration of corticosteroids such as dexamethasone, prednisolone and prednisone which augment immune modulation of T cells. The combined effect of IL + steroid therapy suppresses pro-inflammatory cellular TH1 cytokines such as interferon gamma and TNF-alpha that are produced in excess by activated NK cells and cytotoxic lymphocytes/T-cells (CTL).   IL will, in about 80% of cases, successfully down-regulate activated natural killer cells (NKa) over a period of 2-3 weeks. It is likely to be just as effective as IVIg in this respect but at a fraction of the cost and with a far lower incidence of side-effects. Its effect lasts for ~ 4-6 weeks when administered in early pregnancy.

Intralipid is a suspension of soybean lipid droplets in water and is primarily used as source of parenteral nutrition. When administered intravenously, IL provides essential fatty acids, linoleic acid (LA), an omega-6 fatty acid, and alpha-linolenic acid (ALA), an omega-3 fatty acid.     

It is thought that fatty acids within the emulsion serve as ligands that activate peroxisome proliferator-activated receptors (PPARs) expressed by the NK cells. This is believed to decrease NK cell cytotoxic activity, and thereby enhance implantation A growing number of IVF programs, including ours, perform egg retrieval under conscious sedation using Propofol, a short acting hypnotic agent.

            Whatever the exact mechanism of action might be, Intralipid acts primarily to suppress NK cell over-production of TH-I cytokines. It exerts a modulating effect on certain immune cellular mechanisms largely by down-regulating cytotoxic /activated natural killer cells (NKa). This effect is enhanced through the concomitant administration of corticosteroids such as dexamethasone, prednisolone and prednisone which augment immune modulation of T cells. The combined effect of IL + steroid therapy suppresses pro-inflammatory cellular TH1 cytokines such as interferon gamma and TNF-alpha that are produced in excess by activated NK cells and cytotoxic lymphocytes/T-cells (CTL).   IL will, in about 80% of cases, successfully down-regulate activated natural killer cells (NKa) over a period of 2-3 weeks. It is likely to be just as effective as IVIg in this respect but at a fraction of the cost and with a far lower incidence of side-effects. Its effect lasts for ~ 4-6 weeks when administered in early pregnancy.

            Can laboratory testing be used to assess for an immediate effect of IL on Nka suppression?  Since the downregulation of NKa through IL (or IVIg) therapy can take several weeks to become measurable, it follows that there is really no benefit in trying to assess the potential efficacy of such treatment by retesting NKa in the laboratory after adding IL (or IVIg) to the cells being tested.

IVIg Therapy:  Until about a decade ago, the only effective and available way (in the US) to down-regulate activated NK cells was through the intravenous administration of a blood product known as immunoglobulin-G (IVIg). The fear (albeit unfounded) that the administration of this product might lead to the transmission of viral infections such as HIV and hepatitis C, plus the high cost of IVIG along with the fact that significant side effects occurred about 20% of the time, led to bad press and bad publicity for the entire field of reproductive immunology. It was easier for RE’s to simply say “I don’t believe IVIg works” and thereby avoid risk and bad publicity. But the thousands of women who had babies because of NK cell activity being downregulated through its use, attests to IVIg’s efficacy. But those of us who felt morally obligated to many desperate patients who would not conceive without receiving IVIg were facing an uphill battle. The bad press caused by fear mongering took its toll and spawned a malicious controversy. It was only through the introduction of IL less (about 15-20 years ago ), that the tide began to turn in favor of those patients who required low cost, safe and effective immunotherapy to resolve their IID.

 Corticosteroid Therapy (e.g., Prednisone, and Dexamethasone): Corticosteroid therapy has become a mainstay in the treatment of most women undergoing IVF. It is believed by most to enhance implantation due to an overall immunomodulatory effect. Corticosteroids reduce TH-1 cytokine production by CTL. When given in combination with IL or IVIG they augment the implantation process. The prednisone or dexamethasone therapy must commence (along with IL/IVIg)  10-14 days prior to egg retrieval and continue until pregnancy is discounted or until the 10th week of pregnancy.

 Heparinoid Therapy: There is compelling evidence that the subcutaneous administration of low molecular heparin (Clexane, Lovenox) once daily, (starting with the onset of ovarian stimulation) can improve IVF birthrate in women who test positive for APAs and might prevent later pregnancy loss when used to treat certain thrombophilias (e.g. homozygous MTHFR mutation)

What About Baby Aspirin? In our opinion, aspirin has little (if any) value when it comes to IID, and besides, it could even reduce the chance of success. The reason for this is that aspirin thins the blood and increases the potential to bleed. This effect can last for up to a week and could complicate an egg retrieval procedure or result in “concealed” intrauterine bleeding at the time of embryo transfer, thereby potentially compromising IVF success.

TH-1 Cytokine Blockers (Enbrel, Humira): TH-1 cytokine blockers, (Enbrel and Humira) are in our opinion relatively ineffective in the IVF setting. There has to date been no convincing data to support their use. However, these blockers could have a role in the treatment of a threatened miscarriage thought to be due to CTL/NK activation, but not for IVF. The reason is that the very initial phase of implantation requires a cellular response involving TH-1 cytokines. To block them completely (rather than simply restore a TH-1:TH-2 balance as occurs with IL therapy) so very early on could compromise rather than benefit implantation.

Leukocyte Immunization Therapy (LIT): The subcutaneous injection of the male partner’s lymphocytes to the mother is thought to enhance the ability for the mother’s decidua (uterus) to recognize the DQ alpha matching embryo as “self” or “friend” and thereby avert its rejection. LIT has been shown to up-regulate Treg cells and thus down-regulate NK cell activation thereby improving decidual TH-1:TH-2 balance. Thus, there could be a therapeutic benefit from such therapy. However, the same benefit can be achieved through the use of IL plus corticosteroids. Besides, IL is much less expensive, and the use of LIT is prohibited by law in the U.S.A.

There are two categories of immunologic implantation dysfunction (IID) linked to NK cell activation (NKa).

Autoimmune Implantation Dysfunction: Here, the woman will often have a personal or family history of autoimmune conditions such as Rheumatoid arthritis, Lupus Erythematosus, and thyroid autoimmune activity (e.g., Hashimoto’s disease) etc. Autoimmune as well as in about one third of cases of endometriosis, regardless of severity.  Autoimmune sometimes also occurs in the absence of a personal or family history of autoimmune disease.

When it comes to treating  NKa in  IVF cases complicated by autoimmune implantation dysfunction,  the combination of daily oral dexamethasone commencing with the onset of ovarian stimulation and continuing until the 10th week of pregnancy, combined with an initial infusion of IL (100ml, 20% Il dissolved in 500cc of saline solution, 10-14  days prior to PGT-normal embryo transfer and repeated once more (only), as  soon as the blood pregnancy test is positive), the anticipated chance of a viable pregnancy occurring within 2 completed IVF attempts (including fresh + frozen ET’s)  in women under 39Y (who have normal ovarian reserve)  is approximately  65%.

Alloimmune Implantation Dysfunction: Here, NK cell activation results from uterine exposure to an embryo derived through fertilization by a spermatozoon that shares certain genotypic (HLA/DQ alpha) similarities  with that of the embryo recipient.

Partial DQ alpha/HLA match:  Couples who upon genotyping are shown to share only one DQ alpha/HLA gene are labeled as having a “partial match”. The detection of a “partial match” in association with NKa puts the couple at a considerable disadvantage with regard to IVF outcome. It should be emphasized however, that in the absence of associated Nka, DQ alpha/HLA matching whether “partial” or “total (see below) will NOT cause an IID. Since we presently have no way of determining which embryo carries a matching paternal DQ alpha gene, it follows that each embryo transferred will have about half the chance of propagating a viable pregnancy. Treatment of a partial DQ alpha/HLA match (+ Nka) involves the same IL, infusion as for autoimmune-Nka with one important caveat, namely that here we prescribe oral prednisone as adjunct therapy (rather than dexamethasone) and the IL infusion is repeated every 2-4 weeks following the diagnosis of pregnancy and continued until the 24th week of gestation. Additionally, (as alluded to elsewhere) in such cases we transfer a single (1) embryo at a time. This is because, the likelihood is that one out of two embryos will “match” and we are fearful that if we transfer >1 embryo, and one transferred embryos “matches” it could cause further activation of uterine NK cells and so prejudice the implantation of all transferred embryos. Here it should be emphasized that if associated with Nka, a matching embryo will still be at risk of rejection even in the presence of Intralipid (or IVIg) therapy.

Total (complete) DQ alpha Match:   Here the husband’s DQ alpha genotype matches both of that of his partner’s. While this occurs very infrequently, a total alloimmune (DQ alpha) match with accompanying Nka, means that the chance of a viable pregnancy resulting in a live birth at term, is unfortunately greatly diminished.  Several instances in our experience have required the use of a gestational surrogate.

It is indeed unfortunate that so many patients are being denied the ability to go from “infertility to family” simply because (for whatever reason) so many reproductive specialists refuse to embrace the role of immunologic factors in the genesis of intractable reproductive dysfunction. Hopefully this will change, and the sooner the better.

I strongly recommend that you visit www.DrGeoffreySherIVF.com. Then go to my Blog and access the “search bar”. Type in the titles of any/all of the articles listed below, one by one. “Click” and you will immediately be taken to those you select.  Please also take the time to post any questions or comments with the full expectation that I will (as always) respond promptly.

 

  • The IVF Journey: The importance of “Planning the Trip” Before Taking the Ride”
  • Controlled Ovarian Stimulation (COS) for IVF: Selecting the ideal protocol
  • IVF: Factors Affecting Egg/Embryo “competency” during Controlled Ovarian Stimulation (COS)
  • The Fundamental Requirements for Achieving Optimal IVF Success
  • Use of GnRH Antagonists (Ganirelix/Cetrotide/Orgalutron) in IVF-Ovarian Stimulation Protocols.
  • The Role of Immunologic Implantation Dysfunction (IID) & Infertility (IID): PART 1-Background
  • Immunologic Implantation Dysfunction (IID) & Infertility (IID): PART 2- Making a Diagnosis
  • Immunologic Dysfunction (IID) & Infertility (IID): PART 3-Treatment
  • Thyroid autoantibodies and Immunologic Implantation Dysfunction (IID) Why did my IVF Fail
  • Recurrent Pregnancy Loss (RPL): Why do I keep losing my PregnanciesGenetically Testing Embryos for IVF
  • Staggered IVF
  • Staggered IVF with PGS- Selection of “Competent” Embryos Greatly Enhances the Utility & Efficiency of IVF.
  • Embryo Banking/Stockpiling: Slows the “Biological Clock” and offers a Selective Alternative to IVF-Egg Donation
  • Preimplantation Genetic Testing (PGS) in IVF: It should be Used Selectively and NOT be Routine.
  • IVF: Selecting the Best Quality Embryos to Transfer
  • Preimplantation Genetic Sampling (PGS) Using: Next Generation Gene Sequencing (NGS): Method of Choice.
  • PGS in IVF: Are Some Chromosomally abnormal Embryos Capable of Resulting in Normal Babies and Being Wrongly Discarded?
  • Immunologic Implantation Dysfunction: Importance of Meticulous Evaluation and Strategic Management 🙁 Case Report)
  • Intralipid and IVIG therapy: Understanding the Basis for its use in the Treatment of Immunologic Implantation Dysfunction (IID)
  • Intralipid (IL) Administration in IVF: It’s Composition; how it Works; Administration; Side-effects; Reactions and Precautions
  • Natural Killer Cell Activation (NKa) and Immunologic Implantation Dysfunction in IVF: The Controversy!
  • Natural Killer Cell Activation (NKa) and Immunologic Implantation Dysfunction in IVF: The Controversy!

 

 

 

Name: Anne-Marie D

Hi Dr Sher,

I’ve just had a failed medicated FET. I have now stopped all medication and am waiting for my period to start. I want to do my next transfer as quickly as possible and I’m keen to try a modified natural FET. I have two questions on this:

1. Can I go straight into a modified natural FET cycle when I start my period or do I need to wait to have a natural period first?

2. Is it possible to take a course of the birth control pill when my next period arrives so I can start a modified natural cycle more quickly than having to wait for a natural period?

Answer:

The answer to both questions is yes! However, I am not a proponent of natural cycle FET. See below:

Until less than a decade ago, most women undergoing IVF would have embryos transferred to the uterus in the same cycle that the egg retrieval was performed (“Fresh” Embryo Transfer). This was because embryo cryopreservation (freezing) was a hazardous undertaking.  In fact, it resulted in about 30% not surviving the freezing process and those that did, having about one half the potential of “fresh embryos to implant and propagate a viable pregnancy. The main reason for the high attrition rate associated with embryo cryopreservation is that the “conventional” freezing” process that was done slowly and this resulted in ice forming within the embryo’s cells, damaging or destroying them. The introduction of an ultra-rapid cryopreservation process (vitrification) freezes the embryos so rapidly as to avoid ice crystals from developing. As a result, >90% survive the freeze/thaw process in as good a condition as they were prior to being frozen and thus without being compromised in their ability to propagate a viable pregnancy.

Recently, there have been several articles that have appeared in the literature suggest that an altered hormonal environment may be the reason for this effect.  There have also been reports showing that when singletons (pregnancy with one baby) conceived naturally are compared to singletons conceived through a “fresh” embryo transfers they tend to have a greater chance of low birth weight/prematurity. This difference was not observed in babies born following FET.  Hence, there is a suspicion that the altered hormonal environment during the fresh cycle may be the causative factor.

Available evidence suggests that FET (of pre-vitrified blastocysts) is at least as successful as is the transfer of “fresh” embryos and might even have the edge. The reason for this is certainly unlikely to have anything to do with the freezing process itself. It more than likely  has to do with two factors:

  1. An ever increasing percentage of FET’s involve the transfer of PGS-tested, fully karyotyped, euploid blastocysts that have a greater potential to propagate viable pregnancies, than is the case with “fresh” ET’s where the embryos have rarely undergone prior PGS selection for “competency”…and,
  2. With targeted hormone replacement therapy for FET, one is far better able to better to optimally prepare the endometrium for healthy implantation than is the case where embryos are transferre3d following ovarian stimulation with fertility drugs.

There are additional factors other than method used for embryo cryopreservation that influence outcome following FET. These include

  • An emerging trend towards selective transferring only advanced (day 5-6) embryos (blastocysts).
  •  (PGS) to allow for the selective transfer of genetic competent (euploid) embryos
  • Addressing underlying causes of implantation dysfunction (anatomical and immunologic uterine factors) and
  •  Exclusive use of ultrasound guidance for delivery of embryos transferred to the uterus.

Against this background, the use of FET has several decided advantages:

  • The ability to cryostore surplus embryos left over after fresh embryo transfer
  • The ability to safely hold embryos over for subsequent transfer in a later frozen embryo transfer (FET) cycle (i.e. Staggered IVF) in cases where:
    1. Additional time is needed to perform preimplantation Genetic testing for embryo competency.
    2. In cases where ovarian hyperstimulation increases the risk of life-endangering complications associated with critically severe ovarian hyperstimulation syndrome (OHSS).
    3. To bank (stockpile) embryos for selective transfer of karyotypically normal embryos in older women or those who are diminished ovarian reserve
    4. The ability to store embryos in cases of IVF with third party parenting (Egg Donation; Gestational Surrogacy and Embryo donation) and so improve convenience for those couples seeking such services.

Preimplantation Genetic Sampling with FET:

The introduction of preimplantation genetic sampling (PGS) to karyotyping of embryos for selective transfer of the most “competent” embryos, requires in most cases that the tested blastocysts be vitribanked while awaiting test results and then transferred to the uterus at a later date. Many IVF programs have advocated the routine use of PGS in IVF purported to improve IVF outcome. But PGS should in my opinion should only be used selectively. I do not believe that it is needed for all women undergoing IVF. First there is the significant additional cost involved and second it will not benefit everyone undergoing IVF, in my opinion.

While PGS is a good approach for older women and those with diminished ovarian reserve (DOR) and also for woman who experience recurrent pregnancy loss (RPL) or “unexplained” recurrent IVF failure recent data suggests that it will not improve IVF success rates in  women under 36Y  who have normal ovarian reserve, who represent the majority of women seeking IVF treatment. Nor is it needed in women (regardless of their age) undergoing IVF with eggs donated by a younger donor.  This is because in such women about 1:2/3 of their eggs/embryos are usually chromosomally normal, and in most cases will upon fertilization produce multiple blastocysts per IVF attempt, anyway. Thus in such cases the transfer of 2 blastocysts will likely yield the same outcome regardless of whether the embryos had been subjected to PGS or not. The routine use of

It is another matter when it comes to women who have diminished ovarian reserve and/or DOR contemplating embryo banking and for women with unexplained recurrent IVF failure, recurrent pregnancy loss and women with alloimmune implantation dysfunction who regardless of their age or ovarian reserve require PGS for diagnostic reasons.

Embryo Banking: Some IVF centers are doing embryo banking cycles with Preimplantation Genetic Screening (PGS).  With Embryo Banking” several IVF cycles are performed sequentially (usually about 2 months apart), up to the egg retrieval stage. The eggs are fertilized and the resulting advanced embryos are biopsied. The biopsy specimens are held over until enough 4-8 blastocysts have been vitribanked, thus providing a reasonable likelihood that one or more will turn out to be PGS-normal. At this point the biopsy specimens (derived all banking cycles) are sent for PGS testing at one time (a significant cost-saver), the chromosomally normal blastocysts are identified and the women are scheduled for timed FET procedures….. with a good prospect of  a markedly improved chance of success as well as a reduced risk of miscarriage.

Standard (proposed) Regimen for preparing the uterus for frozen embryo transfer FET) is as follows:

 

 The recipient’s cycle is initiated with an oral contraceptive-OC (e.g. Marvelon/Lo-Estrin; Lo-Ovral etc) for at least 10 days. This is later overlapped with 0.5 mg. (10 units) Lupron/Lucrin (or Superfact/Buserelin) daily for 3 days. Thereupon the OC is withdrawn and daily 0.25 mg (5 units) of Lupron/Lucrin/Superfact injections are continued. Menstruation will usually ensue within 1 week. At this point, an ultrasound examination is performed to exclude ovarian cyst(s) and a blood estradiol measurement is taken (it needs to be <70pg/ml) until daily progesterone administration is initiated some time later. The daily Lupron/Lucrin/Superfact is continued until the initiation of progesterone therapy (see below).

 

Four milligram (4mg) Estradiol valerate (Delestrogen) IM is injected SC, twice weekly (on Tuesday and Friday), commencing within a few days of Lupron/Lucrin/Superfact-induced menstruation. Blood is drawn on Monday and Thursday for measurement of blood [E2].  This allows for planned adjustment of the E2V dosage scheduled for the next day. The objective is to achieve a plasma E2 concentration of 500-1,000pg/ml and an endometrial lining of >8mm, as assessed by ultrasound examination done after 10 days of estrogen exposure i.e. a day after the 3rd dosage of Delestrogen..  The twice weekly, final (adjusted) dosage of E2V is continued until pregnancy is discounted by blood testing or an ultrasound examination. Dexamethasone 0.75 mg is taken orally, daily with the start of the Lupron/Lucrin/Superfact. Oral folic acid (1 mg) is taken daily commencing with the first E2V injection and is continued throughout gestation. Patients also receive Ciprofloxin 500mg BID orally starting with the initiation of Progesterone therapy and continuing for 10 days.

 

Luteal support commences 6 days prior to the ET, with intramuscular progesterone in oil (PIO) at an initial dose of 50 mg (P4-Day 1).  Starting on progesterone administration-Day 2, PIO is increased to 100 mg daily continuing until the 10th week of pregnancy, or until a blood pregnancy test/negative ultrasound (after the 6-7th gestational week), discounts a viable pregnancy.

 

Also, commencing on the day following the ET, the patient inserts one (1) vaginal progesterone suppository (100 mg) in the morning + 2mg E2V vaginal suppository (in the evening) and this is continued until the 10th week of pregnancy or until pregnancy is discounted by blood testing or by an ultrasound examination after the 6-7th gestational week. Dexamethasone o.75mg is continued to the 10th week of pregnancy (tailed off from the 8th to 10th week) or as soon as pregnancy is ruled out. With the obvious exception of the fact that embryo recipients do not receive an hCG injections, luteal phase and early pregnancy hormonal support and immuno-suppression is otherwise the same as for conventional IVF patients.  Blood pregnancy tests are performed 13 days and 15 days after the first P4 injection was given.

  

Note: One (1) vaginal application of Crinone 8% is administered on the 1st day (referred to as luteal phase day 0 – LPO). On LP Day 1, they will commence the administration of Crinone 8% twice daily (AM and PM) until the day of embryo transfer.  Withhold Crinone on the morning of the embryo transfer and resume Crinone administration in the PM.   Crinone twice daily is resumed from the day after embryo transfer. Contingent upon positive blood pregnancy tests, and subsequently upon the ultrasound confirmation of a viable pregnancy, administration of Crinone twice daily are continued until the 10th week of pregnancy.

 

Regime for Thawing and Transferring Cryopreserved Embryos/Morulae/Blastocysts:

 

Patients undergoing ET with cryopreserved embryos/morulas/blastocysts will have their embryos thawed and transferred by the following regimen.

 

  Day 2 (P4) Day 4 (P4) Day 5 (P4) Day 6 (P4)
PN Thaw     ET
Day 3 Embryo   Thaw   ET
Blastocysts frozen on day 5 post-ER       Thaw in AM

ET in PM

Blastocysts frozen on day 6, post-ER       Thaw in AM

ET in PM

 

 

______________________________________________________

ADDENDUM: PLEASE READ!!

INTRODUCING SHER FERTILITY SOLUTIONS (SFS)

Founded in April 2019, Sher Fertility Solutions (SFS) offers online (Skype/FaceTime) consultations to patients from > 40 different countries. All consultations are followed by a detailed written report presenting my personal recommendations for treatment of what often constitute complex Reproductive Issues.

 

Patients are encouraged to share the information I provide, with their treating Physicians and/or to avail themselves of my personal hands-on services, provided through batched IVF cycles that I conduct every 3 months at Los Angeles IVF (LAIVF) Clinic, Century City, Los Angeles, CA.

 

If you wish to schedule an online consultation with me, please contact my assistant (Patti Converse) by phone (800-780-7437/702-533-2691), email (concierge@SherIVF.com) or,  enroll online on then home-page of my website (www.SherIVF.com). 

 

PLEASE SPREAD THE WORD ABOUT SFS!

 

Geoff Sher

 

 

Name: Kodi F

I am 41.5 with DOR. I’m desperately looking for advice on what would be the right protocol. I don’t believe BC helps me. I had 2 months cancelled when on it for cysts. 1 estrogen producing and the other was not so aspirated and started with Protocol #2. I’ve also tried a lupron micro dose flare. I’m trying to do my own research and be my own advocate as I desperately want a child of my own. I’m also on a bunch of supplements in addition to levothyroxine and metformin and do acupuncture.

5/20/22
AMH .63
AFC between 8-15 depending on month
FSH 5.5
EST 56.6

Protocol 1/Egg Retrieval #1 March 2021 (1 month shy of 40)
29 days BC
3 days after stopping BC
stim day 1 (2/28/21)Follistim 225/Menopur 225 evenings
Dexamethasone 1mg by mouth at bedtime
3/4/21 Day 5 – Add Ganirelix
3/9/21 Day 10 – Azithromycin and 4mg Lupron trigger
3/12/21 Retrieval

Retrieved 6 (think there were close to 12, some empty)
Mature 4
Fertilized 4
Blasts 2
PGT-A 2 (both FETs failed, so started seeing a RI)

Protocol 2 – Cancelled
Antagonist with Growth Hormone
7/29 – 8/9 – Nuvaring
8/14-17 (stim day 1-4) – AM – 3 vials Menopur + 1mg Dexamethasone (stop when trigger)
PM – 3 vials Menopur + .35 Omnitrope

8/18-22 (stim day 5-9) AM AM – 3 vials Menopur + 1mg Dexamethasone (stop when trigger)
Add in Ganirelix before 6:30am daily
PM – 3 vials Menopur + .35 Omnitrope
Cancelled on 8/22 (stim day 9)

8/18
Progesterone-.70
LH-2.2
EST-137.3
Left – 9, 5, 3mm
Right – 16, 5, 4mm

8/20
Progesterone-.47
LH-1.1
EST-241.2
Left – 12, 8mm
Right – 20mm

8/22 (I chose to cancel cycle)
Progesterone-.46
LH-2.2
EST-383.6
Left – 16, 11mm
Right – 22, 6mm

Protocol 3 – Failed
Micro Dose Flare (Lupron) with Growth Hormone
9/6-9/26 BC (20 days)
10/1 clomid 100mg in evening for 5x nights
Lupron 20 every 12 hours and 1mg dexamethasone every morning until trigger
10/2-10/9 (stim day 1-8) AM- 2 vials Menopur + .35 Omnitrope + Lupron 20 + Dexa
PM – 300IU Gonal-F + Lupron 20 + Clomid 100mg

10/6 Labs/Ultrasound
Left – 11, 6, 5, 5mm
Right – 16, 12, 11, 11, 9, 8, 8, 7, 6, 4mm
LH – 10.1
Progesterone .53
Est – 426.4

10/8
Left – 12, 12, 12, 11, 7mm
Right – 13, 10mm
LH – 3.7
Progesterone .76
Est – 650.9

10/10
Left – 16, 11, 9, 7mm
Right – 29mm, 25mm, 21mm
LH – 4.1
Progesterone .73
Est – 1017.8

10/10 (stim day 9) Pregnyl Trigger 10,000 (stop melatonin)

10/11 Post Trigger Labs:
Progesterone – 2.5
LH – 1.4

10/12 Retrieval
Retrieved 6
Mature 2
Fertilized 0

Answer:

I cannot provide you with a specific protocol via this venue. I can only do so for an active patient of mine. You would need to contact my assistant, Patti Converse at 702-533-26912 and set up an online consultation with me for this. However, below please find the broad principles I apply when preparing an individualized protocol.

The importance of the IVF stimulation protocol on egg/embryo quality cannot be overstated. This factor seems often to be overlooked or discounted by t IVF practitioners who use a “one-size-fits-all” approach to ovarian stimulation. My experience is that the use of individualized/customized COS protocols can greatly improve IVF outcome. While no one can influence underlying genetics or turn back the clock on a woman’s age, any competent IVF specialist should be able to tailor the protocol for COS to meet the individual needs of the patient.

Gonadotropins (LH and FSH), whether produced by the pituitary gland or administered by way of fertility drugs, have different “targeted” sites of action in the ovary. FSH targets cells that line the inner wall of the follicle (granulosa cells) and also form the cumulus cells that bind the egg to the inner surface of the follicle. Granulosa cells are responsible for estrogen production.

LH, on the other hand, targets the ovarian connective tissue (stroma/theca) that surrounds ovarian follicles resulting in the production of male hormones such as testosterone (predominantly), androstenedione and DHEA. These androgens are then transported to the granulosa cells of the adjacent follicles in a “bucket brigade fashion”. There FSH converts testosterone to estradiol, causing granulosa cells to multiply (proliferate) and produce estradiol, follicles to  grows and eggs to develop (ovogenesis) It follows that  ovarian androgens (mainly testosterone) is absolutely indispensable to follicle/ egg growth and development.

However, the emphasis is on a “normal” amount of testosterone. Over-exposure of the follicle to testosterone can in my opinion,  compromise egg development and lead to an increased likelihood of chromosomal irregularities (aneuploid) following LH/hCG-induced egg maturational division (meiosis) and compromise embryo “competency/quality.

Ovarian androgens can also reach the uterine lining where they sometimes will compromise estrogen receptor -induced endometrial growth and development.

A significant percentage of  older women and those who have diminished ovarian reserve (DOR) have increased LH activity is increased. Such women either over-produce LH and/or the LH produced is far more biologically active. Chronically increased LH activity leads to overgrowth of ovarian connective tissue (stroma/theca). This condition, which is often referred to as Stromal Hyperplasia or hyperthecosis can result in  excessive ovarian androgen/testosterone production and poorer egg-embryo quality/competency, Similarly, women with polycystic ovarian syndrome (PCOS), also characteristically have Stromal hyperplasia/hyperthecosis due to chronically increased LH activity. Thus they too often manifest with increased ovarian androgen production. It is therefore not surprising that “poor egg/embryo quality” is often also a feature of PCOS.

In my opinion, the over-administration of LH-containing menotropins such as Menopur, [which is comprised of roughly equal amount of FSH and   hCG ,which acts similar to LH)], to older women, women with DOR and those who have PCOS can also lead to reduced egg/embryo competency . Similarly, drugs such as clomiphene or Letrozole that cause the pituitary gland to release excessive amounts of LH, are also potentially harmful to egg development and in my opinion, are best omitted from IVF COS protocols. This is especially the case when it comes to older women and those with DOR, who in my opinion should preferably be stimulated using FSH-dominant products such as Follistim, Puregon, Fostimon and Gonal-F. 

Gonadotropin releasing hormone agonists (GnRHa): GnRHa such as Lupron, Buserelin, Superfact, Gonopeptyl etc. are often used to launch ovarian stimulation cycles. They act by causing an initial outpouring followed by a depletion of pituitary gonadotropins. This results in LH levels falling to low concentrations, within 4-7 days, thereby establishing a relatively “LH-free environment”. When GnRHa are administered for about 7 days prior to initiating gonadotropin stimulation (“long” pituitary down-regulation”), the LH depletion that will exist when COS is initiated, will usually be protective of subsequent egg development. In contrast, when the GnRHa administration commences along with the initiation of gonadotropin therapy, there will be a resultant immediate surge in the release of pituitary LH with  the potential to increase ovarian testosterone to egg-compromising levels , from the outset of COS. This, in my opinion could be particularly harmful when undertaken in older women and those who have DOR.

GnRH-antagonists such as Ganirelix, Cetrotide and Orgalutron, on the other hand, act very rapidly (within hours) to block pituitary LH release. The purpose in using GnRH antagonists is to prevent the release of LH during COS. In contrast, the LH-lowering effect of GnRH agonists develops over a number of days.

GnRH antagonists are traditionally given, starting after  5th -7th day of gonadotropin stimulation. However, when this is done in older women and those (regardless of age) who have DOR, LH-suppression might be reached too late to prevent the deleterious effect of excessive ovarian androgen production on egg development in the early stage of ovarian stimulation. This is why, it is my preference to administer GnRH-antagonists, starting at the initiation of gonadotropin administration.

Preferred Protocols for Controlled Ovarian Stimulation (COS):

  • Long GnRH Agonist Protocols: The most prescribed protocol for agonist/gonadotropin administration is the so-called “long protocol”. An agonist (usually, Lupron) is given either in a natural cycle, starting 5-7 days prior to menstruation or is overlapped with the BCP for two days whereupon the latter is stopped and the Lupron, continued until menstruation ensues. The agonist precipitates a rapid rise in FSH and LH level, which is rapidly followed by a precipitous decline in the blood level of both, to near zero. This is followed by uterine withdrawal bleeding (menstruation) within 5-7 days of starting the agonist treatment, whereupon gonadotropin treatment is initiated (preferably within 7-10 days of the onset of menses) while daily Lupron injections continue, to ensure a relatively “low LH- environment”. Gonadotropin administration continues until the hCG trigger.
  • Short (“Flare”) GnRH-agonist (GnRHa) Protocol: Another GnRHa usage for COS is the so called “(micro) flare protocol”. This involves initiating gonadotropin therapy commensurate with initiation of gonadotropin administration. The supposed objective is to deliberately allow Lupron to elicit an initial surge (“flare”) in pituitary FSH release in order to augment FSH administration by increased FSH production. Unfortunately, this “springboard effect” constitutes “a double-edged sword”. While it indeed increases the release of FSH, it at the same time causes a surge in LH release. The latter can evoke excessive ovarian stromal/thecal androgen production which could potentially compromise egg quality, especially when it comes to older women and women with DOR. I am of the opinion that by evoking an exaggerated ovarian androgen response, such “(micro) flare protocols” can harm egg/embryo quality and reduce IVF success rates, especially when it comes to COS in older women, and in women with diminished ovarian reserve. Accordingly, I do not prescribe such protocols to my IVF patients
  • Long-Agonist/Antagonist Conversion Protocol (A/ACP):With a few (notable) exceptions I preferentially advocate this protocol for many of my patients. With the A/ACP, as with the long protocol (see above) the woman again prepares to launch her stimulation cycle by taking a BCP for at least ten days before overlapping with an agonist such as Lupron. However, when about 5-7 days later her menstruation starts, she supplants the agonist with a with 250 mcg) of an antagonist (e.g. Ganirelix, Orgalutron or Cetrotide). Within a few days of this switch-over, gonadotropin stimulation is commenced. Both the antagonist and the gonadotropins are then continued until the hCG trigger. The purpose in switching from agonist to antagonist is to intentionally allow only a very small amount of the woman’s own pituitary LH to enter her blood and reach her ovaries, while at the same time preventing a large amount of LH from reaching her ovaries. This is because while a small amount of LH is essential to promote and optimize FSH-induced follicular growth and egg maturation, a large concentration of LH can trigger over-production of ovarian stromal testosterone, with an adverse effect of follicle/egg/embryo quality. Moreover, since testosterone also down-regulates estrogen receptors in the endometrium, an excess of testosterone can also have an adverse effect on endometrial growth. Also, since agonists might suppress some ovarian response to the gonadotropin stimulation, antagonists do not do so. It is for this reason that the A/ACP is so well suited to older women and those with some degree of diminished ovarian reserve.
  • Agonist/antagonist conversion protocol with estrogen priming:Patients start their treatment cycle on a combined (monophasic) birth control pill-BCP (e.g., Marvelon, Desogen, Orthonovum 135; Low-Estrin…etc.)  for at least 8-10 days (depending on individual circumstances), before commencing controlled ovarian stimulation for IVF. With this approach, a GnRH agonist (e.g. Lupron/Superfact/Buserelin/Decapeptyl etc.) is continued until menstruation ensues (usually 5-7 days after commencement of the GnRH-agonist). At this point, the GnRH-agonist is SUPPLANTED with 250mcg GnRH antagonist (e.g. Ganirelix/Cetrotide, Orgalutron) and daily estradiol(E2) “priming” commences using either E2 skin-patches or intramuscular estradiol valerate (Delestrogen) injections, twice weekly while continuing the administration of the GnRH antagonist. Seven (7) days after commencing the E2 skin patches or intramuscular Delestrogen, daily injections of recombinant FSH-(e.g., Follistim/Gonal-F/Puregon)  + menotropin (e.g., Menopur)  therapy begins.. This is continued at a modified dosage, along with E2 patches or Delestrogen injections) until the “hCG trigger”. The egg retrieval is performed 36 hours later.

There are a few potential drawback to the use of the A/ACP. We have learned that prolonged use of a GnRH antagonist throughout the ovarian stimulation process can compromise the predictive value of serial plasma E2 measurements to evaluate follicle growth and development. It appears that when the antagonist is given throughout stimulation, the blood E2 levels tend to be significantly lower than when the agonist alone is used or where antagonist treatment is only commenced 5-7 days into the ovarian stimulation process. The reason for this is presently unclear. Accordingly, when the A/ACP is employed, we rely more on follicle size monitoring than on serial blood E2 trends to assess progress.

Also, younger women (under 30 years) and women with absent, irregular or dysfunctional ovulation, and those with polycystic ovarian syndrome are at risk of developing life-threatening Severe Ovarian Hyperstimulation Syndrome (OHSS). The prediction of this condition requires daily access to accurate blood E2 levels. Accordingly, we currently tend to refrain from prescribing the A/ACP in such cases, preferring instead use the “standard long-protocol” approach.

  • Short-GnRH antagonist protocols:The use of GnRH antagonists as currently prescribed in ovarian stimulation cycles (i.e. the administration of 250mcg daily starting on the 6th or 7th day of stimulation with gonadotropins) may be problematic, especially in women over 39 yrs., women with diminished ovarian reserve (i.e. “poor responders” to gonadotropins), and women with PCOS. Such women tend to have higher levels of LH to start with and as such the initiation of LH suppression with GnRH antagonists so late in the cycle (usually on day 6-7) of stimulation fails to suppress LH early enough to avoid compromising egg development. This can adversely influence egg/embryo quality and endometrial development. As is the case with the “microflare” approach (see above) the use of GnRH antagonist protocols in younger women who have normal ovarian reserve, is acceptable. Again, for reasons of caution, and because I see no benefit in doing so, I personally never prescribe this approach for my patients. Presumably, the reason for the suggested mid-follicular initiation of high dose GnRH antagonist is to prevent the occurrence of the so called “premature LH surge”, which is known to be associated with “follicular exhaustion” and poor egg/embryo quality. However the term “premature LH surge” is a misnomer and the concept of this being a “terminal event” or an isolated insult is erroneous. In fact, the event is the culmination (end point) of the progressive escalation in LH (“a staircase effect”) which results in increasing ovarian stromal activation with commensurate growing androgen production. Trying to improve ovarian response and protect against follicular exhaustion by administering GnRH antagonists during the final few days of ovarian stimulation is like trying to prevent a shipwreck by removing the tip of an iceberg.
  • Short-GnRH-agonist (“micro-flare”) protocols:Another approach to COH is by way of so-called “microflare protocols”. This involves initiating gonadotropin therapy simultaneously with the administration of GnRH agonist. The intent is to deliberately allow Lupron to affect an initial surge (“flare”) in pituitary FSH release to augment ovarian response to the gonadotropin medication. Unfortunately, this approach represents “a double-edged sword” as the resulting increased release of FSH is likely to be accompanied by a concomitant (excessive) rise in LH levels that could evoke excessive production of male hormone by the ovarian stroma. The latter in turn could potentially compromise egg quality, especially in women over 39 years of age, women with diminished ovarian reserve, and in women with polycystic ovarian syndrome (PCOS) – all of whose ovaries have increased sensitivity to LH. In this way, “microflare protocols” can potentially hinder egg/embryo development and reduce IVF success rates. While microflare protocols usually are not harmful in younger women and those with normal ovarian reserve, I personally avoid this approach altogether for safety’s sake. The follicles/eggs of women on GnRH-agonist “micro-flare protocols” can be exposed to exaggerated agonist-induced LH release, (the “flare effect”) while the follicles/eggs of women, who receive GnRH antagonists starting 6-8 days following the initiation of stimulation with gonadotropins can likewise be exposed to pituitary LH-induced ovarian male hormones (especially testosterone). While this is not necessarily problematic in younger women and those with adequate ovarian reserve (“normal responders”) it could be decidedly prejudicial in “poor responders” and older women where there is increased follicle and egg vulnerability to high local male hormone levels.
  • The “Trigger Shot”- A Critical Decision:The egg goes through maturational division (meiosis) during the 36-hour period that precedes ovulation or retrieval. The efficiency of this process will determine the outcome of reproduction. It follows that when it comes to ovulation induction, aside from selecting a suitable protocol for COS one of the most important decisions the clinician has to make involves choosing and implementing with logic and precision, the “trigger shot” by which to facilitate meiosis.
    • Urinary versus recombinant hCG:Until quite recently, the standard method used to initiate the “trigger shot” was through the administration of 10,000 units of hCGu. More recently, a recombinant form of hCGr (Ovidrel) was introduced and marketed in 250 mcg doses. But clinical experience strongly suggests that 250 mcg of Ovidrel is most likely not equivalent in biological potency to 10,000 units of hCG. It probably at best only has 60%of the potency of a 10,000U dose of hCGu and as such might not be sufficient to fully promote meiosis, especially in cases where the woman has numerous follicles. For this reason, I firmly believe that when hCGr is selected as the “trigger shot” the dosage should be doubled to 500 mcg, at which dosage it will probably have an equivalent effect on promoting meiosis as would 10,000 units of hCGu.
    • The dosage of hCG used: Some clinicians, when faced with a risk of OHSS developing will deliberately elect to reduce the dosage of hCG administered as a trigger in the hope that by doing so, the risk of developing critical OHSS will be lowered. It is my opinion that such an approach is not optimal because a low dose of hCG (e.g., 5000 units hCGu or 25omcg hCGr) is likely inadequate to optimize the efficiency of meiosis, particularly when it comes to cases such as this where there are numerous follicles. In my opinion a far better approach is to use a method that I first described in 1989, known as “prolonged coasting”
    • Use of hCG versus a GnRHa(e.g., Lupron/Buserelin/Superfact) as the trigger shot: It has been suggested that the use of an “agonist ( Lupron) trigger” in women at risk of developing severe ovarian hyperstimulation syndrome (OHSS) could potentially reduce the risk of the condition becoming critical and thereby placing the woman at risk of developing life-endangering complications. It is for this reason that many RE’s prefer to trigger meiosis in this way (using an agonist-Lupron) rather than through the use of hCG. The agonist promptly causes the woman’s pituitary gland to expunge a large amount of LH over a short period of time and it is this LH “surge” that triggers meiosis. The problem with this approach, in my opinion, is that it is hard to predict how much LH will be released in by the pituitary gland of a given patient receiving an agonist trigger shot, especially if the woman was down-regulated using an agonist, or in cases where an antagonist was used to block pituitary LH release. For this reason, I personally prefer to use hCGu for the trigger, even in cases of ovarian hyperstimulation, with one important proviso…that she underwent “prolonged coasting” in order to reduce the risk of critical OHSS prior to the 10,000 unit hCGu “trigger”.
    • Combined use of hCG +GnRHa; This approach is preferable to the use of a GnRHa, alone. However, in my opinion is inferior to the appropriate and correct use of hCG, alone.
    • The timing of the trigger shot to initiate meiosis:This should coincide with the majority of ovarian follicles being >15 mm in mean diameter with several follicles having reached 18-22 mm. Follicles of larger than 22 mm will usually harbor overdeveloped eggs which in turn will usually fail to produce good quality eggs. Conversely, follicles less than 15 mm will usually harbor underdeveloped eggs that are more likely to be aneuploid and incompetent following the “trigger”.

Severe Ovarian Hyperstimulation Syndrome (OHSS) and prolonged Coasting”

OHSS is a life-endangering condition that usually occurs in women undergoing COS where the blood E2 level rises to above 4,000pg/ml. The risk escalates to greater than 80% in cases where the E2 level rises above 6,000pg/ml. It rarely occurs in normally ovulating women or older (>39Y) women and is more commonly encountered in:

  • Young women (under 30y) who have a high ovarian reserve(based upon basal FSH and AMH.
  • Women with polycystic Ovarian Syndrome (PCOS)
  • Non-PCOS women who do not ovulate spontaneously

The treating physician should be alerted to the possibility of hyperstimulation when encountering a woman who develops >25 ovarian follicles of 14mm-16mm in mean diameter, in association with a blood E2 level of above 2,5000pg/ml prior to the hCG “trigger”.

OHSS is a self-limiting condition. Its development is linked to the effect of hCG and thus does not occur until the “hCG trigger” is administered. In fact, there is virtually no risk of OHSS until the hCG “trigger” is administered.

Prolonged Coasting” is a procedure I introduced in 1991. It involves abruptly stopping gonadotropin therapy while continuing to administer the GnRH agonist (e.g. Lupron, Buserelin) deferring the hCG “trigger” until the woman is out of risk (as evidenced by a fall in plasma estradiol level to below 2,500pg/ml).

It is important that “prolonged coasting” be initiated as soon as two or more follicles have attained a greater diameter than 18mm with at least 50% of the remaining follicles having attained 14-16mm. To start the process of “prolonged coasting” any earlier or any later, while it would still protect against the development of OHSS, would almost certainly result in compromised egg and embryo quality with ultimate failure of the IVF cycle. Simply stated, the precise timing of initiating the process is critical. Proper implementation of PC will almost always prevent OHSS without seriously compromising egg/embryo quality.

Use of the Birth Control Pill (BCP) to launch IVF-COS.

In natural (unstimulated) as well as in cycles stimulated with fertility drugs, the ability of follicles to properly respond to FSH stimulation is dependent on their having developed FSH-responsive receptors. Pre-antral follicles (PAF) do not have such primed FSH receptors and thus cannot respond properly to FSH stimulation with gonadotropins. The acquisition of FSH receptor responsivity requires that the pre-antral follicles be exposed to FSH, for a number of days (5-7) during which time they attain “FSH-responsivity” and are now known as antral follicles (AF). These AF’s are now able to respond properly to stimulation with administered FSH-gonadotropins. In regular menstrual cycles, the rising FSH output from the pituitary gland insures that PAFs convert tor AF’s. The BCP (as well as prolonged administration of estrogen/progesterone) suppresses FSH. This suppression needs to be countered by artificially causing blood FSH levels to rise in order to cause PAF to AF conversion prior to COS commencing, otherwise pre-antral-to –antral follicle conversion will not take place in an orderly fashion, the duration of ovarian stimulation will be prolonged and both follicle and egg development may be compromised. GnRH agonists cause an immediate surge in release of FSH by the pituitary gland thus causing conversion from PAF to SAF. This is why women who take a BCP to launch a cycle of COS need to have an overlap of the BCP with an agonist. By overlapping the BCP with an agonist for a few days prior to menstruation the early recruited follicles are able to complete their developmental drive to the AF stage and as such, be ready to respond appropriately to optimal ovarian stimulation. Using this approach, the timing of the initiation of the IVF treatment cycle can readily and safely be regulated and controlled by varying the length of time that the woman is on the BCP.

Since optimizing follicular response to COS requires that prior to stimulation with gonadotropins, FSH-induced conversion from PAF to AF’s first be completed and the BCP suppresses FSH, it follows when it comes to women launching COS coming off a BCP something needs to be done to cause a rise in FSH for 5-7 days prior to menstruation heralding the cycle of CO S. This is where overlapping the BCP with a GnRHa comes in. The agonist causes FSH to be released by the pituitary gland and if overlapped with the BCP for several days and this will (within 2-5 days) facilitate PAF to AF conversion…. in time to start COS with the onset of menstruation. Initiating ovarian stimulation in women taking a BCP, without doing this is suboptimal.

I strongly recommend that you visit www.SherIVF.com. Then go to my Blog and access the “search bar”. Type in the titles of any/all of the articles listed below, one by one. “Click” and you will immediately be taken to those you select.  Please also take the time to post any questions or comments with the full expectation that I will (as always) respond promptly.

  • The IVF Journey: The importance of “Planning the Trip” Before Taking the Ride”
  • Controlled Ovarian Stimulation (COS) for IVF: Selecting the ideal protocol
  • The Fundamental Requirements For Achieving Optimal IVF Success
  • Use of GnRH Antagonists (Ganirelix/Cetrotide/Orgalutron) in IVF-Ovarian Stimulation Protocols.
  • Anti Mullerian Hormone (AMH) Measurement to Assess Ovarian Reserve and Design the Optimal Protocol for Controlled Ovarian Stimulation (COS) in IVF.
  • The “Biological Clock” and how it should Influence the Selection and Design of Ovarian Stimulation Protocols for IVF.
  • A Rational Basis for selecting Controlled Ovarian Stimulation (COS) protocols in women with Diminished Ovarian Reserve (DOR)
  • Diagnosing and Treating Infertility due to Diminished Ovarian Reserve (DOR)
  • Ovarian Stimulation in Women Who have Diminished Ovarian Reserve (DOR): Introducing the Agonist/Antagonist Conversion protocol
  • Controlled Ovarian Stimulation (COS) in Older women and Women who have Diminished Ovarian Reserve (DOR): A Rational Basis for Selecting a Stimulation Protocol
  • Optimizing Response to Ovarian Stimulation in Women with Compromised Ovarian Response to Ovarian Stimulation: A Personal Approach.
  • Egg Maturation in IVF: How Egg “Immaturity”, “Post-maturity” and “Dysmaturity” Influence IVF Outcome:
  • Commonly Asked Question in IVF: “Why Did so Few of my Eggs Fertilize and, so Many Fail to Reach Blastocyst?”
  • Human Growth Hormone Administration in IVF: Does it Enhances Egg/Embryo Quality and Outcome?
  • The BCP: Does Launching a Cycle of Controlled Ovarian Stimulation (COS). Coming off the BCP Compromise Response?
  • Staggered IVF
  • Staggered IVF with PGS- Selection of “Competent” Embryos Greatly Enhances the Utility & Efficiency of IVF.
  • Staggered IVF: An Excellent Option When. Advancing Age and Diminished Ovarian Reserve (DOR) Reduces IVF Success Rate
  • Embryo Banking/Stockpiling: Slows the “Biological Clock” and offers a Selective Alternative to IVF-Egg Donation
  • Preimplantation Genetic Testing (PGS) in IVF: It should be Used Selectively and NOT be Routine.
  • IVF: Selecting the Best Quality Embryos to Transfer
  • Preimplantation Genetic Sampling (PGS) Using: Next Generation Gene Sequencing (NGS): Method of Choice.
  • PGS in IVF: Are Some Chromosomally abnormal Embryos Capable of Resulting in Normal Babies and Being Wrongly Discarded?
  • PGS and Assessment of Egg/Embryo “competency”: How Method, Timing and Methodology Could Affect Reliability
  • IVF outcome: How Does Advancing Age and Diminished Ovarian Reserve (DOR) Affect Egg/Embryo “Competency” and How Should the Problem be addressed.

 

 

______________________________________________________

ADDENDUM: PLEASE READ!!

INTRODUCING SHER FERTILITY SOLUTIONS (SFS)

Founded in April 2019, Sher Fertility Solutions (SFS) offers online (Skype/FaceTime) consultations to patients from > 40 different countries. All consultations are followed by a detailed written report presenting my personal recommendations for treatment of what often constitute complex Reproductive Issues.

 

If you wish to schedule an online consultation with me, please contact my assistant (Patti Converse) by phone (800-780-7437/702-533-2691), email (concierge@SherIVF.com) or,  enroll online on then home-page of my website (www.SherIVF.com). 

 

PLEASE SPREAD THE WORD ABOUT SFS!

 

Geoff Sher

 

 

 

 

IVF

Name: Nagibe C

Hi,
I’m 40 years old, 3 failed IUI during the last year and consulting for IVF. I’m looking for a clinic that allows me 2 embryo transfer at once for a chance of twins. is this possible at your clinic?

Answer:

I would absolutely consider this favorably!

 

I suggest you call my assistant, Patti Converse at 702-533-2591 and set up an online consultation with me to discuss.

 

Geoff Sher

Name: Kristin C

Hi Dr.
I am 39 and have PCOS. I went for my day 3 AFC and I was informed by the nurse that I have 20 follicles on my left ovary and another 20 on my right. The strange thing is 7 years ago when I conceived my daughter, my numbers were less at 15 and 13 per ovary.

My doctor is starting me on letrozole with ovidrel and timed intercourse this month. With such high numbers for my AFC, is it likely that if I do conceive it will be a miscarriage’s due to poor egg quality? I had two miscarriages from natural pregnancies in the past 9 months. And I ovulate like clockwork between day 14 -16 unmedicated.

Answer:

Quite frankly, at 39y ,with PCOS, I would favor IVF over ovarian stimulation with or without IUI.

Polycystic ovary syndrome (PCOS) is a common hormonal system disorder among women affecting between 5% and 10% of women of reproductive age worldwide.  Women with PCOS may have enlarged ovaries that contain small collections of fluid — called follicles — located in each ovary as seen during an ultrasound. The condition is characterized by abnormal ovarian function (irregular or absent periods, abnormal or absent ovulation and infertility), androgenicity (increased body hair or hirsutism, acne) and increased body weight –body mass index or BMI. The ovaries of women with PCOS characteristically contain multiple micro-cysts often arranged like a “string of pearls” immediately below the ovarian surface (capsule) interspersed by an overgrowth of ovarian connective tissue (stroma).

PCOS is one of the most common causes of menstrual irregularities, infertility, and hirsutism. Despite an enormous effort to define its cause, the etiology of PCOS remains unclear, and there is no definite cure at this time. PCOS is clearly a heterogeneous disorder which often has a familial (genetic) basis. Infertility associated with PCOS has been attributed to numerous factors, including dysfunctional gonadotropin pituitary secretion, peripheral insulin resistance, elevated adrenal and/or ovarian androgen (male hormone) levels, and dysfunction of several growth factors. Women with this condition are often obese and insulin resistant. The compensatory hyperinsulinemia further stimulates ovarian androgen production which may be detrimental to egg maturation and there is a clear link between the degree of insulin resistance and anovulation. PCOS is also a significant long-term health risk for women, thus necessitating vigilance through regular annual examinations (non-insulin dependent diabetes mellitus, hypertension, hypercholesterolemia, cardiovascular disease and endometrial cancer). Whereas PCOS-related infertility is usually manageable through the use of fertility drugs, lifestyle changes (diet and exercise) remain a mainstay of long-term therapy. More recently, ovulation rates, circulating androgens, pregnancy rates and perhaps even first-trimester miscarriage rates have been shown to improve when insulin sensitizers like metformin are used to correct the underlying insulin resistance.

Most patients with PCOS are young and have excellent pregnancy rates with oral clomiphene. Those that require more aggressive treatments with injectable medications probably represent a subgroup of PCOS patients with severe ovarian dysfunction. These women often have explosive response to gonadotropins which can result in serious complications like Severe Ovarian Hyperstimulation Syndrome (OHSS…see below) and high order multiple births. In those women, the ability to perform “prolonged coasting” (see below) and selectively transfer fewer embryos during IVF offers a clear advantage over standard gonadotropin injections.

Egg quality in PCOS

The potential for a woman’s eggs to undergo orderly maturation, successful fertilization and subsequent progression to “good quality embryos” is in large part genetically determined. However, the expression of such potential is profoundly susceptible to numerous influences, especially intra-ovarian hormonal changes during the pre-ovulatory phase of the cycle. Proper follicular stimulation as well as precise timing of egg maturation with LH (Luteinizing Hormone) or hCG (human chorionic gonadotropin) is crucial to optimal egg maturation, fertilization and ultimately embryo quality. Both pituitary gonadotropins, LH and FSH (follicle stimulating hormone) play a pivotal but different role in follicular development. The action of FSH is mainly directed toward granulosa cell (cells lining the inside of the follicle) proliferation and estrogen production (E2). LH, on the other hand, acts primarily on the ovarian stroma (the connective tissue that surrounds the follicle) to produce androgens. While small amounts of ovarian androgens, such as testosterone, enhance egg and follicle development, over-exposure to them can have a deleterious effect. Furthermore, excessive ovarian androgens can also compromise estrogen-induced endometrial growth and development.

Suppressing pituitary secretion of LH with gonadotropin releasing hormone (GnRH) agonists such as Lupron®, is particularly useful in PCOS. In that condition, serum LH levels are elevated, leading to stromal overgrowth, follicular arrests (so-called cysts) and high levels of androgens synthesis. It is therefore not surprising that these follicles often yield poorly developed (“immature”) eggs” at the time of egg retrieval (ET) and that “poor egg/embryo quality”, inadequate endometrial development and high miscarriage rates are common features of this condition. However, contrary to popular belief, this is not due to an intrinsic deficit in “egg quality”. Stimulation protocols geared toward optimizing follicle and egg development and avoiding over exposure to androgens correct these problems ad result in pregnancy rates similar to those of non-PCOS women. Whereas the overuse of LH-containing preparations such as Menopur® and Luveris® further aggravates this effect. In conclusion, to maximize ultimate oocyte maturation, we strongly recommend against the exclusive use of such products in PCOS patients, preferring FSH-dominant products such as Folistim®, Gonal F® or Bravelle® over a period of at least 9 days following pituitary suppression with Lupron®.

PCOS women often have a family history of diabetes and demonstrable insulin resistance (evidenced by high blood insulin levels and an abnormal 2-hour glucose tolerance test).This underlying Diabetes mellitus tendency could play a role in the development of PCOS and contribute to the development of obesity, an abnormal blood lipid profile, and a predisposition to coronary vascular disease. Women with PCOS are slightly more at risk of developing uterine, ovarian and possibly also breast cancer in later life and accordingly should be evaluated for these conditions on a more frequent basis than would ordinarily be recommended to non-PCOS women.

Most women with PCOS either do not ovulate at all or they ovulate irregularly. As a consequence, thereof they in addition usually experience delayed, absent or irregular menstruation. In addition, an inordinate percentage of the eggs produced by PCOS women following ovulation induction, tend to be chromosomally abnormal (aneuploid). Rather than being due to an intrinsic egg defect being inherent in PCOS women, the poor egg quality more than likely the result of over-exposure to male hormones (predominantly, testosterone) produced by the ovarian stroma. These two factors (ovulation dysfunction and poor egg quality) are the main reasons for the poor reproductive performance (infertility and an increased miscarriage rate) in PCOS women.

PCOS patients are at an inordinate risk of severely over-responding fertility drugs, both oral varieties (e.g., Clomiphene, Serophene & Femara) and especially the injectables (e.g., Follistim, Puregon, Gonal F, Menopur and Bravelle) by forming large numbers ovarian follicles. This can lead to life endangering complications associated with sever ovarian hyperstimulation (OHSS). In addition PCOS women receiving fertility drugs often experience multiple ovulations putting them at severe risk (40%+)  of high order multiple pregnancy (i.e. triplets or greater) with often devastating consequences.

VARIETIES OF POLYCYSTIC OVARIAN SYNDROME:

1) Hypothalamic-pituitary-PCOS: This is the commonest form of PCOS and is  often genetically transmitted and is characteristically  associated with a blood concentration of Luteinizing Hormone (LH) that is uncharacteristically much higher than  the Follicle Stimulating Hormone (FSH) level (FSH is normally higher than the LH concentration) as well as high-normal or  blood androgen ( male) hormone  concentrations (e.g. androstenedione, testosterone and dehydroepiandrosterone -DHEA).Hypothalamic-pituitary-ovarian  PCOS is also often associated with insulin resistance and in about 40%-50% of the cases.

2) Adrenal PCOS: Here the excess of male hormones are derived from overactive adrenal glands rather than from the ovaries. Blood levels of testosterone and/or androstenedione raised but here, but here, the blood level of dehydroepiandrosterone (DHEAS) is also raised, clinching the diagnosis.

3) Severe pelvic adhesive disease secondary to severe endometriosis, chronic pelvic inflammatory disease and/or extensive pelvic surgery: Women who have this type of PCOS tend to less likely to hyperstimulate in response to ovulation induction. Their. DHEAS is also is not raised.

 

TREATMENT OF INFERTILITY DUE TO ASSOCIATED OVULATION DYSFUNCTION:

Hypothalamic-pituitary-/ovarian PCOS: Ovulation induction with fertility drugs such as clomiphene citrate, Letrozole (Femara) or gonadotropins, with or without intrauterine insemination (IUI) is often highly successful in establishing pregnancies in PCOS women. However, IVF is fast becoming a treatment of choice (see below).

 

 

In about 40% of cases, 3-6 months of oral Metformin (Glucophage) treatment results in a significant reduction of insulin resistance, lowering of blood androgen levels, an improvement in ovulatory function, and/or some amelioration of androgenous symptoms and signs.

Surgical treatment by “ovarian drilling” of the many small ovarian cysts lying immediately below the envelopment (capsule) of the ovaries, is often used, but is less successful than alternative non-surgical treatment and is only temporarily effective. The older form of surgical treatment, using ovarian wedge resection is rarely used any longer as it can produce severe pelvic adhesion formation.

Adrenal PCOS is treated with steroids such as prednisone or dexamethasone which over a period of several weeks will suppress adrenal androgen production, allowing regular ovulation to take place spontaneously. This is often combined with clomiphene, Letrozole and/or gonadotropin therapy to initiate ovulation.

PCOS attributable to Pelvic Adhesive Disease is one variety which often is associated with compromised ovarian reserve, a raised FSH blood level and ovarian resistance to fertility drugs. In many such cases, high dosage of gonadotropins (FSH-dominant) with “estrogen priming” will often elicit an ovarian response necessary for successful ovulation induction and/or IVF. Neither steroids nor Metformin are helpful in the vast majority of such cases.

PCOS women undergoing ovulation induction usually release multiple eggs following the hCG trigger and are thus at inordinate risk of twin or higher order multiple pregnancies. They are also at risk of developing OHSS.  Many now believe that IVF should be regarded as a primary and preferential treatment for PCOS. The reason is that it is only through this approach that the number of embryos reaching the uterus can be controlled and in this manner the risk of high-order multiples can be minimized and it is only in the course of IVF  treatment that a novel treatment method  known as “prolonged coasting” ( see below) which prevents OHSS, can be implemented

SEVERE OVARIAN HYPERSTIMULATION SYNDROME (OHSS):

As indicated above, there is an inordinate propensity for women with PCOS to hyper-respond to gonadotropin fertility drugs and in the process produce large numbers of ovarian follicles. If left unchecked this can lead to OHSS, a potentially life endangering condition. The onset of OHSS is signaled by the development of a large number of ovarian follicles (usually more than 25 in number). This is accompanied by rapidly rising plasma estradiol (E2) levels, often exceeding 3000pg/ml within 7 or 9 days of stimulation, often rapidly peaking above 6,000 pg/ml prior to hCG administration. When this happens, the risk of OHSS developing is above 80%.

Symptoms and signs of OHSS include: abdominal distention due to fluid collection (ascites), fluid in the chest cavity (hydrothorax), rapid weight gain (of a pound or more per day) due to tissue fluid retention, abdominal pain, lower back ache, nausea, diarrhea, vomiting, visual disturbances such as blurred vision and spots in front of the eyes (scotomata), a rapidly declining urine output, cardiovascular collapse and failure of blood to clot which sometimes results in severe bruising (echymosis) and frank bleeding.  These symptoms and signs may appear before pregnancy can be diagnosed. If pregnancy occurs, the condition is likely to worsen progressively over a period of 3-5 weeks whereupon it rapidly resolves spontaneously over a few days. If no pregnancy occurs, the symptoms and signs all disappear spontaneously within 10-12 days of the hCG injection.

When increasing fluid collection in the abdominal cavity (ascites) starts to compromise breathing raising the head of the bed rose slightly by placing a 4–6-inch block at the base of each head post and using a few additional pillows, will sometimes help ameliorate the problem. In cases where this does not help or symptoms become severe, all or most of the fluid can readily and safely be drained through t transvaginal sterile needle aspiration (vaginal paracentesis-performed once or sometimes twice a week) can be performed once or twice weekly. The problem will usually self corrects within 10-12 days of the hCG shot if pregnancy does not occur or, by the 8th week of pregnancy.

Urine output should be monitored daily to see if it drops below about 500ml a day (about two cups and a half). A chest X-ray, to evaluate for fluid collection in the chest and around the heart should be done weekly along with blood tests for hematocrit, BUN, electrolytes, creatinine, platelet count and fibrin split products (FSP). If indicated on the basis of a deteriorating clinical situation, hospitalization might be needed for close observation and if necessary, to provide intensive care.

In all case of OHSS, the ovaries will invariably be considerably enlarged. This is irrelevant to the final outcome, unless ovarian torsion (twisting of the ovary on its axis), an extremely rare complication occurs. The latter would usually require surgical emergency surgical intervention.

 

 

It is important to know that symptoms and signs of OHSS are severely aggravated by rising hCG levels. Thus such patients should not receive additional hCG injections. 

Does PCOS cause poor egg/embryo quality? It is an undeniable fact that women with PCOS undergoing IVF are commonly found to have poorly developed (“dysmorphic”) eggs, with reduced fertilization potential and yielding “poor quality embryos”. However, in the author’s opinion (which admittedly runs contrary to popular opinion), this is unlikely to be due to an intrinsic deficit in egg quality. Rather, it more likely relates to   intra-ovarian hormonal changes brought about by hyperstimulation and which compromise egg development.  This effect, in the author’s opinion, can often be significantly reduced through implementation of an individualized or customized   ovarian stimulation protocols that minimize exposure of the developing follicles and eggs to excessive LH-induced ovarian androgens. This can be best achieved by limiting the use of LH-containing gonadotropins such as Menopur through selective institution of “prolonged coasting” (see below).

In the past, the onset of OHSS, heralded by the presence of large numbers of developing ovarian follicles and rapidly rising plasma estradiol levels often led the treating physician to prematurely administer hCG in an attempt to abruptly arrest the process and prevent escalation of risk to the patient. However, the premature administration of hCG, while abruptly arresting further proliferation of estrogen producing granulosa cells in the follicles, unfortunately also prematurely arrests egg development. Since the ability of an egg to achieve optimal maturation upon hCG triggering is largely predicated upon it having achieved prior optimal development, the untimely administration of hCG which triggers meiosis, probably increases the risk of numerical chromosomal abnormalities (aneuploidy) of the egg. This in turn would lead to reduced fertilization potential, poor egg/embryo quality and low embryo implantation potential.

In women with PCOS the connective tissue that surrounding the follicles (ovarian stroma) is often characteristically overgrown (stromal hyperplasia). It is the stroma that produces androgens (mainly testosterone) in response to LH. It is this, coupled with the fact that PCOS women also often have elevated blood LH concentrations (see above) results in the excessive production of androgen hormones, which is so characteristic in PCOS. While excessive exposure of developing eggs to ovarian androgens compromises follicle and egg growth it also impairs endometrial response to estrogen, which could explain the common finding of poor endometrial thickening in many PCOS women undergoing IVF.

The obvious remedy for these adverse effects on egg and endometrial development is to employ stimulation protocols that limit ovarian over-exposure to LH and allowing the time necessary for the follicles/eggs to develop optimally, prior to administering hCG through the judicious implementation of   “Prolonged coasting” (PC).

 

 

“PROLONGED COASTING”:

In the early 90’s we were the first to report on “prolonged coasting” (PC), a novel approach that protects egg quality while preventing the development of OHSS. PC has since, gained widespread acceptance as a method of choice for preventing OHSS and has established itself as the “standard of care”. It involves withholding gonadotropin therapy while continuing the administration of the GnRHa and waiting until the plasma estradiol concentration drops below 2,500 pg/ml. Thereupon hCG is administered. In such cases, regardless of the number of developed follicles or the number of eggs retrieved, these women rarely, if ever develop OHSS. It has been reported that while PC virtually eliminates the risk of life-endangering complications associated with OHSS, there are reports in the literature that “the price to pay with PC” is often a poorer fertilization rate and   reduced embryo implantation potential, compromising the pregnancy”. It is the author’s opinion an experience in the development of PC that egg/embryo quality deficit likely has  little to do with the process of PC, itself and can be  explained as follows:  When  PC is initiated too early, follicle growth and development may cease (as evidenced by the estradiol level plateauing or falling immediately, rather than showing an initial continued increase), and when  PC is started  too late, the follicles will often become cystic, measuring >21mm by the time the estradiol level falls below the safe threshold of 2500pg/ml, and so harbor dysmorphic  eggs. Thus precise timing of the initiation of PC is critical. It should in pact be initiated preemptively in all cases when there are more than 25 follicles and the plasma estradiol reaches or exceeds 2,500pg/ml in association, provided that at least 50% of the follicles measuring 14-16mm in mean diameter. Not a day sooner or a day later. If PC is initiated with precise timing, it will usually be followed by a further progressive rise in the estradiol concentration. After a few days, the estradiol level will plateau and then it will start to fall (often rapidly). The temptation to trigger with hCG before the estradiol level falls below 3000picogtrams per milliliter must be resisted …even if the level falls below 1,000pg/ml by the time hCG is given.

Since when using agonist (Cetrotide/Ganirelix/Orgalutron) pituitary suppression throughout the stimulation phase with gonadotropins, the plasma estradiol level often under expressed follicle growth, this method of pituitary blockade should not be used in cases (such as with PCOS) where PC might be required.

 

Please go to my Blog and access the “search bar”. Type in the titles of any/all of the articles listed below, one by one. “Click” and you will immediately be taken to those you select.  Please also take the time to post any questions or comments with the full expectation that I will (as always) respond promptly.

  • A Fresh Look at the Indications for IVF
  • The IVF Journey: The importance of “Planning the Trip” Before Taking the Ride”
  • Controlled Ovarian Stimulation (COS) for IVF: Selecting the ideal protocol
  • IVF: Factors Affecting Egg/Embryo “competency” during Controlled Ovarian Stimulation(COS)
  • The Fundamental Requirements For Achieving Optimal IVF Success
  • Use of GnRH Antagonists (Ganirelix/Cetrotide/Orgalutron) in IVF-Ovarian Stimulation Protocols.
  • Human Growth Hormone Administration in IVF: Does it Enhances Egg/Embryo Quality and Outcome?
  • IVF and the use of Supplementary Human Growth Hormone (HGH) : Is it Worth Trying and who needs it?
  • The BCP: Does Launching a Cycle of Controlled Ovarian Stimulation (COS). Coming off the BCP Compromise Response?
  • Blastocyst Embryo Transfers Should be the Standard of Care in IVF
  • Anti Mullerian Hormone (AMH) Measurement to Assess Ovarian Reserve and Design the Optimal Protocol for Controlled Ovarian Stimulation (COS) in IVF.
  • Embryo Transfer: The “Holy Grail in IVF.
  • IVF: Approach to Selecting the Best Embryos for Transfer to the Uterus.
  • Fresh versus Frozen Embryo Transfers (FET) Enhance IVF Outcome
  • Frozen Embryo Transfer (FET): A Rational Approach to Hormonal Preparation and How new Methodology is Impacting IVF.
  • Genetically Testing Embryos for IVF
  • Staggered IVF
  • Preimplantation Genetic Testing (PGS) in IVF: It should be Used Selectively and NOT be Routine.
  • IVF: Selecting the Best Quality Embryos to Transfer
  • Preimplantation Genetic Sampling (PGS) Using: Next Generation Gene Sequencing (NGS): Method of Choice.
  • PGS in IVF: Are Some Chromosomally abnormal Embryos Capable of Resulting in Normal Babies and Being Wrongly Discarded?
  • Sher Fertility Solutions (SFS): An Exciting New Chapter….
  • Should IVF Treatment Cycles be provided uninterrupted or be Conducted in 7-12 Pre-scheduled “Batches” per Year
  • A personalized, stepwise approach to IVF
  • How Many Embryos should be transferred: A Critical Decision in IVF.
  • Avoiding High Order Multiple Pregnancies (Triplets or Greater) with IVF
  • The Role of Nutritional Supplements in Preparing for IVF
  • Ovarian Hyperstimulation Syndrome (OHS): Its Evolution & Reducing itsIncumbent Risks
  • Taking A Fresh Look at Ovarian Hyperstimulation Syndrome (OHSS), its Presentation, Prevention and Management
  • Preventing Severe Ovarian Hyperstimulation Syndrome (OHSS) with “Prolonged Coasting”
  • IVF Outcome in Patients with Polycystic Ovarian Syndrome (PCOS): Minimizing the Risk of Severe Ovarian Hyperstimulation Syndrome (OHSS) and optimizing Egg/Embryo Quality.
  • Understanding Polycystic Ovarian Syndrome (PCOS) and the Need to Customize Ovarian Stimulation Protocols.
  • IVF & Polycystic Ovarian Syndrome (PCOS): Reducing the Risk of Severe Ovarian Hyperstimulation Syndrome (OHSS), Improving Egg Quality and Optimizing Outcome.

 

 

______________________________________________________

ADDENDUM: PLEASE READ!!

INTRODUCING SHER FERTILITY SOLUTIONS (SFS)

Founded in April 2019, Sher Fertility Solutions (SFS) offers online (Skype/FaceTime) consultations to patients from > 40 different countries. All consultations are followed by a detailed written report presenting my personal recommendations for treatment of what often constitute complex Reproductive Issues.

 

If you wish to schedule an online consultation with me, please contact my assistant (Patti Converse) by phone (800-780-7437/702-533-2691), email (concierge@SherIVF.com) or,  enroll online on then home-page of my website (www.SherIVF.com). 

 

PLEASE SPREAD THE WORD ABOUT SFS!

 

Geoff Sher

 

 

 

 

Name: Shira M

Dear Doctor, I would like to ask for your opinion of ERA specifically in women with adenomyosis. I had an ERA which showed that I would need one extra day of progesterone (so transfer after 6 days). However, a new doctor came at my clinic and he does all transfers after 5 days of progesterone. After that, I had three transfers: one at 5.5 days which worked but resulted in a late loss, and two at 5 days with no implantation (not even a chemical). What do you think in the context of adenomyosis? Thank you very much in advance.

Answer:

Herewith my response on both issues;

A: ADENOMYOSIS:

Adenomyosis is a condition where endometrial glands develop outside the uterine lining (endometrium), within the muscular wall of the uterus (myometrium). Definitive diagnosis of adenomyosis is difficult to make. The condition should be suspected when a premenopausal woman (usually>25 years of age) presents with pelvic pain, heavy painful periods, pain with deep penetration during intercourse, “unexplained infertility” or repeated miscarriages and thereupon, when on digital pelvic examination she is found to have an often smoothly enlarged (bulky) soft tender uterus. Previously, a definitive diagnosis was only possible after a woman had her uterus removed (hysterectomy) and it this was inspected under a microscope. However the use of uterine magnetic resonance imaging (MRI) now permits reliable diagnosis. Ultrasound examination of the uterus on the other hand , while not permitting definitive diagnosis, is a very helpful tool in raising a suspicion of the existence of adenomyosis.

 

Criteria used to make a diagnosis of adenomyosis on transvaginal ultrasound:

 

  • Smooth generalized enlargement of the uterus.
  • Asymmetrical thickening of one side of the (myometrium) as compared to another side.
  • Thickening (>12mm) of the junctional zone between the endometrium and myometrium with increased blood flow.
  • Absence of a clear line of demarcation between the endometrium and the myometrium
  • Cysts in the myometrium
  • One or more non discrete (not encapsulated) tumors (adenomyomas) in the myometrium.

 

Since there is no proven independent relationship between adenomyosis and egg/embryo quality any associated reproductive dysfunction (infertility/miscarriages) might be attributable to an implantation dysfunction. It is tempting to postulate that this is brought about by adenomyosis-related anatomical pathology at the endometrial-myometrial junction. However, many women with adenomyosis, do go on to have children without difficulty. Given that 30%-70% of women who have adenomyosis also have endometriosis…. a known cause of infertility, it is my opinion that infertility caused by adenomyosis is likely linked to endometriosis where infertility is at least in part due to a toxic pelvic environment that compromises egg fertilization potential and/or due to an immunologic implantation dysfunction (IID) linked to activation of uterine natural killer cells (NKa). Thus, in my opinion all women who are suspected of having adenomyosis-related reproductive dysfunction (infertility/miscarriages) should be investigated for endometriosis and for IID. The latter, if confirmed would make them candidates for selective immunotherapy (using intralipid/steroid/heparin) in combination with IVF.

 

 

 

Surgery: Conservative surgery to address adenomyosis-related infertility involves excision of portions of the uterus with focal or nodular adenomyosis and/or excision of uterine adenomyomas. It is very challenging and difficult to perform because adenomyosis does not have distinct borders that distinguish normal uterine tissue from the lesions. In addition, surgical treatment for adenomyosis-related reproductive dysfunction is of questionable value and of course is  not an option for diffuse adenomyosis.

 

Medical treatment: There are three approaches.

  • GnRH agonists (Buserelin/Lupron) which is thought to work by lowering estrogen levels.
  • Aromatase inhibitors such as Letrozole have also been tried with limited success
  • Inhibitors of angiogenesis: The junctional zone in women with adenomyosis may grow blood vessels more readily that other women (i.e. angiogenesis). A hormone known as VEGF can drive this process. It is against this background that it has been postulated that use of drugs that reduce the action of VEGF and thereby counter blood vessel proliferation in the uterus could have a therapeutic benefit. While worth trying in some cases, thus far such treatment has been rather disappointing
  • Immunotherapy to counter IID: The use of therapies such as Intralipid (or IVIG)/steroids/heparin in combination with IVF might well hold promise in those women with adenomyosis who have NKa.

Fortunately, not all women with adenomyosis are infertile. For those who are, treatment presents a real problem. Even when IVF is used and the woman conceives, there is still a significant risk of miscarriage. Since the condition does not compromise egg/embryo quality, women with adenomyosis-related intractable reproductive dysfunction who fail to benefit from all options referred to above…(including IVF) might as a last resort consider  Gestational surro resort consider  Gestational surrogacy.

B: ERA

 

The blastocyst and the endometrium are in a constant state of cross-talk. In order for successful implantation to take place, the blastocyst must be at the appropriate stage of development, and needs to signal a well synchronized endometrium to ‘accept it”. This dialogue between embryo and endometrium involves growth factors, cytokines, immunologic accommodations, cell adhesion molecules, and transcription factors. These are all mostly genetically driven but are also heavily influenced by numerous physiologic, pathophysiologic, hormonal and molecular mechanisms capable of profoundly affecting the receptivity of the secretory endometrium to the overtures made by the embryo, to implant. 

Embryo implantation takes place 6-9 days after ovulation. This period is commonly referred to as the “window of implantation (WOI)”. In the past it was believed that as long as the embryo reached the uterus in this 4 day time frame, its chance of implanting would not be affected.

In 2013, after evaluating 238 genes in the secretory endometrium and applying bioformatics, Ruiz-Alonzo, et all introduced the Endometrial Receptivity Array (ERA) . Using this test, they categorized mid-secretory endometria into 4 categories: “a) proliferative, b) pre-receptive, c) receptive or d) post-receptive”. They claimed that women with pre-receptive or post-receptive endometria were more likely to experience failed implantation post-embryo transfer (ET).

It was in large part this research which suggested that the concept of a relatively “wide” (4day) WOI, was flawed, that an optimal WOI is likely much narrower and could be a critical factor in determining the success or failure of implantation post-ET. Ruiz-Alonzo also  reported that about 25% of women with recurrent IVF failure (RIF), have pre, or post-receptive endometria. They presented data suggesting that  viable IVF pregnancy rates could be enhanced,

by deferring FET by about 24 hours in women who had pre-receptive endometria and bringing ET forward by the same amount of time, in women with post-receptive endometria,

 

There is no doubt that ERA testing has opened the door to an intriguing arena for research. Presently however, available data is inconclusive. Here, following  recent studies are 2 dissenting opinions regarding  the value for ERA:

  • Basil and Casper (2018) state: “Performing the ERA test in a mock cycle prior to a FET does not seem to improve the ongoing pregnancy rate in good prognosis patients. Further large prospective studies are needed to elucidate the role of ERA testing in both good prognosis patients and in patients with recurrent implantation failure”
  • Churchill and Comstock (2017)  conclude:” In our preliminary observations, the non-receptive ERA group had similar live birth rates compared to the receptive ERA group. It appears the majority of the pregnancies conceived in the non-receptive group occurred during ovulatory cycles and thus a non-receptive ERA in a medicated cycle likely does not have prognostic value for ovulatory cycles. Larger studies are needed to assess the prognostic value of ERA testing in the gen-eral infertility population.”

There are additional negatives that relate to the considerable emotional and financial cost of doing ERA testing:

  1. First, the process costs $600-$1000 to undertake
  2. , Second, it requires that the patient undergo egg retrieval, vitrify (cryobank) all blastocysts, res for 1 or more cycles to allow their hormonal equilibrium to restore, do an ERA biopsy to determine the synchronicity of the endometrium, wait a few weeks for the results of the test and thereupon engage in undertaking an additional natural or hormonal preparation cycle for timed FET. This represents a significant time lapse, emotional cost and additional expense.

Presently, ERA testing is only advocated for women who have experienced several IVF failures. However, some authorities are beginning to advocate that it become routine for women undergoing all IVF.

The  additional financial cost inherent in the performance of the ERA test ($600-$1000), the considerable time delay in getting results, the fact that awaiting results of testing and preparing the patient for FET, of necessity extends the completion of the IVF/ET process by at least a few months, all serve to increase the emotional and financial hardship confronting patients undergoing ERA. Such considerations, coupled with the current absence of conclusive data that confirm efficacy, are arguments against the widespread use of ERA . In my opinion, ERA testing should presently be considered as being  one additional diagnostic and be confined to women with “unexplained” RIF.

Gold standard statistical analyses require that all confounding variables be controlled while examining the effect of altering the one under assessment. There is an obvious interplay of numerous, ever changing variables involved in outcome following ET,  e.g. embryo competency, anatomical configuration of the uterus and the contour of the endometrial cavity, endometrial thickness, immunologic and molecular factors as well as the very important  effect of technical skill/expertise in performing the  ET procedure …(to mention but a few). It follows that it is virtually impossible to draw reliable conclusions from IVF-related randomized controlled studies that use outcome as the end-point. This applies equally to results reported following “ gold standard”  testing on the efficacy of ERA and, is one of the main reasons why I question the reliability of reported data (positive or negative).

The fact is that IVF (and related technologies) constitute neither a “pure science” nor a “pure art”. Rather they represent an “art-science blend”, where scientific principles applied to longitudinal experience and technical expertise coalesce to produce a biomedical product that will invariably differ (to a greater or lesser degree) from one set of clinical circumstances to another.

Since, the ultimate goal of applied Assisted Reproductive Medicine is to safely achieve the birth of a viable and healthy baby, the tools we apply, that are aimed at achieving this end-point, are honed through the adaptation of scientific principles and concepts, experience and expertise, examined and tested longitudinally over time. Needless to say, the entire IVF/ET process is of necessity subject to change and adaptation as new scientific and technical developments emerge.

This absolutely applies to the ERA as well!

______________________________________________________

ADDENDUM: PLEASE READ!!

INTRODUCING SHER FERTILITY SOLUTIONS (SFS)

Founded in April 2019, Sher Fertility Solutions (SFS) offers online (Skype/FaceTime) consultations to patients from > 40 different countries. All consultations are followed by a detailed written report presenting my personal recommendations for treatment of what often constitute complex Reproductive Issues.

 If you wish to schedule an online consultation with me, please contact my assistant (Patti Converse) by phone (800-780-7437/702-533-2691), email (concierge@SherIVF.com) or,  enroll online on then home-page of my website (www.SherIVF.com). 

 

Name: Svitlana M

Hello Doctor Sher,
I have a question about my last analyses. I got the resulting biopsy.
CD3 (Polyclonal Rabbit, Dako) – an average of 20 positive cells per 1 v.p.z. X40;
CD4 (Clone4B12, Thermo scientific) – an average of 28 positives were detected cells per 1 v.p.z. X40;
CD8(cloneSP16, Thermo scientific) – an average of 7 positives were detected cells for 1 v.p.z. X40;
CD56/NCAM-1,Ab-4(56C04, Thermo scientific) – detected on average 28 of positive cells per 1 v.p.z. X40;
CD25 (Clone IL2R.1, Diagnostic Biosystems) – no positive cells were detected;
Can it be the reason for infertility?
I also have endometriosis.
I will be grateful for your answer.

Answer:

In my opinion, these tests cannot provide reasonable insight into whether or not there is an underlying immunologic implantation dysfunction.(IID). Might I recommend that you contact my assistant, Patti Converse at 702-533-2691 and set up an online consultation with me to discuss~!

Implantation dysfunction is unfortunately often overlooked as an important cause of IVF failure. This is especially relevant in cases of unexplained IVF failure, recurrent pregnancy loss (RPL), and in women with underlying endo-uterine surface lesions, thickness of the uterine lining (endometrium) and /or immunologic factors.

IVF success rates have been improving over the last decade. The average live birth rate per embryo transfer in the U.S.A for women under 40y using their own eggs is currently better than 1:3 women. However, there is still a wide variation from program to program for IVF live birth rates, ranging from 20% to near 50%. Based upon these statistics, most women undergoing IVF in the United States require two or more attempts to have a baby. IVF practitioners in the United States commonly attribute the wide dichotomy in IVF success rates to variability in expertise of the various embryology laboratories. This is far from accurate. In fact, other factors such as wide variations in patient selection and the failure to develop individualized protocols for ovarian stimulation or to address those infectious, anatomical, and immunologic factors that influence embryo implantation are at least equally important.

About 80% of IVF failures are due to “embryo incompetency” that is largely due to aneuploidy      usually related to advancing age of the woman and is further influenced by other factors such as the protocol selected for ovarian stimulation, diminished ovarian reserve (DOR), and severe male factor infertility. However, in about 20% of dysfunctional cases embryo implantation is the cause of failure.

This section will focus on implantation dysfunction and IVF failure due to:

ANATOMICAL ENDO-UTERINE SURFACE LESIONS

            It has long been suspected that anatomical defects of the uterus might result in infertility. While the presence of uterine fibroids, in general, are unlikely to cause infertility, an association between their presence and infertility has been observed in cases where the myomas distort the uterine cavity or protrude through the endometrial lining.  Even small fibroids that lie immediately under the endometrium (submucous fibroids) and protrude into the uterine cavity have the potential to lower embryo implantation.  Multiple fibroids in the uterine wall (intramural fibroids) that encroach upon the uterine cavity can sometimes so compromise blood flow that estrogen delivery is impaired, and the endometrium is unable to thicken properly. This can usually be diagnosed by ultrasound examination during the proliferative phase of the menstrual cycle.  It is likely that any surface lesion in the uterine cavity, whether submucous fibroids, intrauterine adhesions a small endometrial or a placental polyp, has the potential to interfere with implantation by producing a local inflammatory response, not too dissimilar in nature from that which is caused by an intrauterine contraceptive device (IUD).       

            Clearly, since even small uterine lesions have the potential to adversely affect implantation, the high cost (financial, physical, and emotional) associated with IVF and related procedures, justifies the routine performance of diagnostic procedures such as an HSG, hysterosonogram (fluid ultrasound examination), or hysteroscopy prior to initiating IVF.  Identifiable uterine lesions that have the potential of impairing implantation usually require surgical intervention.  In most cases, dilatation and curettage (D & C) or hysteroscopic resection will suffice. Some cases might require the performance of a laparotomy.  Such intervention will often result in subsequent improvement of the endometrial response.      

Sonohysterography [Fluid ultrasonography (FUS)]: Fluid ultrasonography is a procedure whereby a sterile solution of saline is injected via a catheter through the cervix and into the uterine cavity. The fluid-distended cavity is examined by vaginal ultrasound for any irregularities that might point to surface lesions such as polyps, fibroid tumors, scarring, or a uterine septum. If performed by an expert, a FUS is highly effective in recognizing even the smallest lesion and can replace hysteroscopy under such circumstances. FUS is less expensive, less traumatic, and equally as effective as hysteroscopy. The only disadvantage lies in the fact that if a lesion is detected, it may require the subsequent performance of hysteroscopy to treat the problem anyway.

Hysteroscopy: Diagnostic hysteroscopy is an office procedure that is performed under intravenous sedation, general anesthesia, or paracervical block with minimal discomfort to the patient. This procedure involves the insertion of a thin, lighted, telescope like instrument known as a hysteroscope through the vagina and cervix into the uterus to fully examine the uterine cavity. The uterus is first distended with normal saline, which is passed through a sleeve adjacent to the hysteroscope. As is the case with FUS, diagnostic hysteroscopy facilitates examination of the inside of the uterus under direct vision for defects that might interfere with implantation. We have observed that approximately one in eight candidates for IVF have lesions that require attention prior to undergoing IVF in order to optimize the chances of a successful outcome. We strongly recommend that all patients undergo therapeutic surgery (usually by hysteroscopy) to correct the pathology prior to IVF.  Depending on the severity and nature of the pathology, therapeutic hysteroscopy may require general anesthesia and, in such cases, should be performed in an outpatient surgical facility or conventional operating room where facilities are available for laparotomy, a procedure in which an incision is made in the abdomen to expose the abdominal contents for diagnosis, or for surgery should this be required.       

THICKNESS OF THE UTERINE LINING (ENDOMETRIUM):

As far back as in 1989 we first reported on the finding that ultrasound assessment of the late proliferative phase endometrium can identify those candidates who are least likely to conceive. We noted that the ideal thickness of the endometrium at the time of ovulation or egg retrieval is >8 mm and that thinner linings are associated with decreased implantation rates.

More than 30 years ago we first showed that in normal and “stimulated” cycles, pre-ovulatory endometrial thickness and ultrasound appearance is predictive of embryo implantation (pregnancy) potential following ET. With conventional IVF and with FET, endometrial lining at the time of the “trigger shot” or with the initiation of progesterone needs to preferably be at least 8 mm in sagittal thickness with a triple line (trilaminar) appearance. Anything less than an 8mm endometrial thickness       is associated with a reduction in live birth rate per ET. An 8-9mm thickness represents a transitional measurement…a “gray zone”.  Hitherto, attempts to augment endometrial growth in women with poor endometrial linings by bolstering circulating estrogen blood levels (through the administration of increased doses of fertility drugs, aspirin administration and by supplementary estrogen therapy) yielded disappointing results.

            A “poor” uterine lining is usually the result of the innermost layer of endometrium (the basal or germinal endometrium from which endometrium grows) not being able to respond to estrogen by propagating an outer, “functional” layer thick enough to support optimal embryo implantation and development of a healthy placenta (placentation). The “functional” layer ultimately comprises 2/3 of the full endometrial thickness and is the layer that sheds with menstruation if no pregnancy occurs.

The main causes of a “poor” uterine lining are:

  • Damage to the basal endometrium because of:
  • Inflammation of the endometrium (endometritis) most commonly resulting from infected products left over following abortion, miscarriage, or birth
  • Surgical trauma due to traumatic uterine scraping, (i.e. due to an over-aggressive D & C)
  • Insensitivity of the basal endometrium to estrogen due to:
  • Prolonged, over-use/misuse of clomiphene citrate
  • Prenatal exposure to diethylstilbestrol (DES). This is a drug that was given to pregnant women in the 1960’s to help prevent miscarriage
  • Over-exposure of the uterine lining to ovarian male hormones (mainly testosterone): Older women, women with diminished ovarian reserve (poor responders) and women with polycystic ovarian syndrome -PCOS tend to have raised LH biological activity. This causes the connective tissue in the ovary (stroma/theca) to overproduce testosterone. The effect may be further exaggerated when certain methods for ovarian stimulation such as “flare” protocols and high dosages of Menopur are used in such cases.
  • Reduced blood flow to the basal endometrium: Examples include.
  • Multiple uterine fibroids – especially when these are present under the endometrium (submucosal)
  • Uterine adenomyosis (excessive, abnormal invasion of the uterine muscle by endometrial glands).

Vaginal Viagra: About 35 years ago, after reporting on the benefit of administering vaginal Sildenafil (Viagra) to women who had implantation dysfunction due to thin endometrial linings we announced the birth of the world’s first “Viagra baby.”  Viagra administered vaginally, but not orally, in affected women improves uterine blood flow causing more estrogen to be delivered to the basal endometrium and increasing the endometrial thickening.  Following vaginal administration, Viagra is rapidly absorbed and quickly reaches the uterine blood system in high concentrations. Thereupon it dilutes out as it is absorbed into the systemic circulation. This probably explains why treatment is virtually devoid of systemic side effects.  It is important to recognize that Viagra will NOT be effective in improving endometrial thickness in all cases. In fact, about one third of women treated fail to show any improvement. This is because in certain cases of thin uterine linings, the basal endometrium will have been permanently damaged and left unresponsive to estrogen. This happens in cases of severe endometrial damage due mainly to post-pregnancy endometritis (inflammation), chronic granulomatous inflammation due to uterine tuberculosis (hardly ever seen in the United States) and following extensive surgical injury to the basal endometrium (as sometimes occurs following over-zealous D&C’s).

  • Immunologic factors: These also play a role in IVF failure (see “Immunologic factors and Implantation” …see below.

IMMUNOLOGIC IMPLANTATION DYSFUNCTION (IID)

            Currently, with few exceptions, practitioners of assisted reproduction tend to attribute “unexplained and/or repeated” IVF failure(s), almost exclusively to poor embryo quality, advocating adjusted protocols for ovarian stimulation and/or gamete and embryo preparation as a potential remedy. The idea, having failed IVF, that all it takes to ultimately succeed is to keep trying the same recipe is over-simplistic.

            The implantation process begins six or seven days after fertilization of the egg. At this time, specialized embryonic cells (i.e., trophoblasts), that later become the placenta begin growing into the uterine lining. When the trophoblast and the uterine lining meet, they, along with immune cells in the lining, become involved in a “cross talk” through mutual exchange of hormone-like substances called cytokines. Because of this complex immunologic interplay, the uterus can foster the embryo’s successful growth. Thus, from the earliest stage, the trophoblast establishes the very foundation for the nutritional, hormonal and respiratory interchange between mother and baby.  In this manner, the interactive process of implantation is not only central to survival in early pregnancy but also to the quality of life after birth.

There is an ever growing realization, recognition, and acceptance of the fact that uterine immunologic dysfunction can lead to immunologic implantation dysfunction (IID) with “unexplained” infertility, IVF failure, and recurrent pregnancy loss (RPL).

DIAGNOSIS 

Because immunologic problems may lead to implantation failure, it is important to properly evaluate women with risk factors such as:

  • Unexplained or recurrent IVF failures
  • Unexplained infertility or a family history of autoimmune diseases (e.g., rheumatoid arthritis, lupus erythematosus and hypothyroidism).
  • Recurrent Pregnancy Loss (RPL)
  • Endometriosis
  • A personal or family history of autoimmune conditions, e.g., Rheumatoid Arthritis, Lupus erythematosus, autoimmune hypothyroidism (Hashimoto’s disease) etc.

            Considering its importance, it is not surprising that the failure of a properly functioning immunologic interaction during implantation has been implicated as a cause of recurrent miscarriage, late pregnancy fetal loss, IVF failure and infertility. A partial list of immunologic factors that may be involved in these situations includes:

  • Antiphospholipid antibodies (APA)
  • Antithyroid antibodies (ATA/AMA)
  • Activated natural killer cells (NKa)

ACTIVATED NATURAL KILLER CELLS (NKa):

Following ovulation and during early pregnancy, NK cells and T-cells comprise more than 80% of the lymphocyte-immune cells that frequent the uterine lining. These lymphocytes (white blood cells) journey from the bone marrow to the uterus and under hormonal regulation, proliferate there. After exposure to progesterone (due to induced /spontaneous exogenous administration), they begin to produce TH-1 and TH-2 cytokines. TH-2 cytokines are humoral in nature and induce the trophoblast (“root system of the embryo”) to permeate the uterine lining while TH-I cytokines induce a process referred to as apoptosis (cell suicide) thereby confining placental development to the inner part of the uterus. Optimal placental development (placentation) mandates that there be a balance between TH1 and TH-2 cytokines. Most of the cytokine production originates from NK cells (rather than from cytotoxic T-cells/Lymphocytes (CTL)). Excessive production/release of TH-1 cytokines, is toxic to the trophoblast and to endometrial cells, leading to programmed death/suicide (apoptosis) and subsequently to IID.

Functional NK cells reach a maximal concentration in the endometrium by about t day 6-7 days after exposure to progesterone …. This timing corresponds with when the embryo implants into the uterine lining (endometrium).

It is important to bear in mind that measurement of the concentration of blood NK cells has little or no relevance when it comes to assessing NK cell activation (NKa). Rather, it is the NK cell activation that matters. In fact, there are certain conditions (such as with endometriosis) where the NK cell blood concentration is below normal, but NK cell activation is markedly increased.

There are several methods by which NK cell activation (cytotoxicity) can be assessed in the laboratory. Methods such as immunohistochemical assessment of uterine NK cells and/or through measurement of uterine or blood TH-1 cytokines. However, the K-562 target cell blood test still remains the gold standard. With this test, NK cells, isolated from the woman’s blood using Flow Cytometry are incubated in the presence of specific “target cells”. The percentage (%) of “target cells” killed is then quantified. More than 12% killing suggests a level of NK cell activation that usually requires treatment.

Currently, there are less than a half dozen Reproductive Immunology Reference Laboratories in the U.S.A that are capable of performing the K-562 target cell test reliably.

There exists a pervasive but blatant misconception on the part of many, that the addition of IL or IVIg to a concentration of NK cells could have an immediate down-regulatory effect on NK cell activity. Neither IVIg nor IL is capable of significantly suppressing already activated “functional NK cells”. They are believed to work through “regulating” NK cell progenitors which only thereupon will start to propagate down-regulated NK cells. Thus, testing for a therapeutic effect would require that the IL/IVIg infusion be done about 14 days prior to ovulation or progesterone administration…  in order to allow for a sufficient number of normal (non-activated) “functional” NK cell” to be present at the implantation site when the embryos are transferred.

Failure to recognize this reality has, in our opinion, established an erroneous demand by practicing IVF doctors, that Reproductive Immunology Reference Laboratories report on NK cell activity before and again, immediately following laboratory exposure to IVIg and/or IL in different concentrations. Allegedly, this is to allow the treating physician to report back to their patient(s) on whether an IL or IVIG infusion will be effective in downregulating their Nka.  But, since already activated NK cells (NKa) cannot be deactivated in the laboratory, effective NKa down-regulation can only be adequately accomplished through deactivation of NK cell “progenitors /parental” NK cells in order to allow them thereupon, to s propagate normal “functional” NK cells and his takes about 10-14 days, such practice would be of little clinical benefit. This is because even if blood were to be drawn 10 -14 days after IL/IVIg treatment it would require at least an additional 10 -14days to receive results from the laboratory, by which time it would be far too late to be of practical advantage.

ANTIPHOSPHOLIPID ANTIBODIES:

 Many women who experience “unexplained” IVF failure, women with RPL, those with a personal or family history of autoimmune diseases such as lupus erythematosus, rheumatoid arthritis, scleroderma, and dermatomyositis (etc.)  as well as women who have endometriosis (“silent” or overt) test positive for APAs. More than 30 years ago, we were the first to propose that women who test positive for APA’s be treated with a mini-dose heparin to improve IVF implantation and thus birth rates. This approach was based upon research that suggested that heparin repels APAs from the surface of the trophoblast (the embryo’s “root system) thereby reducing its ant-implantation effects.  We subsequently demonstrated that such therapy only improved IVF outcome in women whose APAs were directed against two specific IgG and/or IgM phospholipids [i.e., phosphatidylethanolamine (PE) and phosphatidylserine (PS)].  More recently low dosage heparin therapy has been supplanted using longer acting low molecular weight heparinoids such as Lovenox and Clexane.   It is very possible that APAs alone do not cause IID but that their presence might help to identify a population at risk due to concomitant activation of uterine natural killer cells (Nka) which through excessive TH-1 cytokine production causes in IID: This is supported by the following observations:

  • The presence of female APAs in cases of male factor cases appears to bear no relationship to IID.
  • Only APA positive women who also test positive for abnormal NK activity appear to benefit from selective immunotherapy with intralipid/IVIg/ steroids.
  • Most APA positive women who have increased NK cell activity also harbor IgG or IgM phosphatidylethanolamine (PE) and phosphatidylserine (PS) antibodies.

ANTITHYROID ANTIBODIES: (ATA).

 A clear relationship has been established between ATA and reproductive failure (especially recurrent miscarriage and infertility).

Between 2% and 5% of women of the childbearing age have reduced thyroid hormone activity (hypothyroidism). Women with hypothyroidism often manifest with reproductive failure i.e., infertility, unexplained (often repeated) IVF failure, or recurrent pregnancy loss (RPL). The condition is 5-10 times more common in women than in men. In most cases hypothyroidism is caused by damage to the thyroid gland resulting from of thyroid autoimmunity (Hashimoto’s disease) caused by damage done to the thyroid gland by antithyroglobulin and antimicrosomal auto-antibodies. 

The increased prevalence of hypothyroidism and thyroid autoimmunity (TAI) in women is likely the result of a combination of genetic factors, estrogen-related effects, and chromosome X abnormalities.  This having been said, there is significantly increased incidence of thyroid antibodies in non-pregnant women with a history of infertility and recurrent pregnancy loss and thyroid antibodies can be present asymptomatically in women without them manifesting with overt clinical or endocrinologic evidence of thyroid disease. In addition, these antibodies may persist in women who have suffered from hyper- or hypothyroidism even after normalization of their thyroid function by appropriate pharmacological treatment. The manifestations of reproductive dysfunction thus seem to be linked more to the presence of thyroid autoimmunity (TAI) than to clinical existence of hypothyroidism and treatment of the latter does not routinely result in a subsequent improvement in reproductive performance.

It follows, that if antithyroid autoantibodies are associated with reproductive dysfunction they may serve as useful markers for predicting poor outcome in patients undergoing assisted reproductive technologies.

Some years back, I reported on the fact that 47% of women who harbor thyroid autoantibodies, regardless of the absence or presence of clinical hypothyroidism, have activated uterine natural killer cells (NKa) cells and cytotoxic lymphocytes (CTL) and that such women often present with reproductive dysfunction. We demonstrated that appropriate immunotherapy with IVIG or intralipid (IL) and steroids, subsequently often results in a significant improvement in reproductive performance in such cases.

The fact that almost 50% of women who harbor antithyroid antibodies do not have activated CTL/NK cells suggests that it is NOT the antithyroid antibodies themselves that cause reproductive dysfunction. The activation of CTL and NK cells that occurs in half of the cases with TAI is probably an epiphenomenon with the associated reproductive dysfunction being due to CTL/NK cell activation that damages the early “root system” (trophoblast) of the implanting embryo. We have shown that treatment of those women who have thyroid antibodies + NKa/CTL using IL/steroids, improves subsequent reproductive performance while women with thyroid antibodies who do not harbor NKa/CTL do not require or benefit from such treatment

TEATMENT OF IID:

The mainstay of treatment involves the selective use of:

  • Intralipid (IL) infusion
  • IVIg therapy
  • Corticosteroids (Prednisone/dexamethasone)
  • Heparinoids (Lovenox/Clexane)

Intralipid (IL) Therapy:  IL is a suspension of soybean lipid droplets in water and is primarily used as source of parenteral nutrition. When administered intravenously, IL provides essential fatty acids, linoleic acid (LA), an omega-6 fatty acid, and alpha-linolenic acid (ALA), an omega-3 fatty acid.     

It is thought that fatty acids within the emulsion serve as ligands that activate peroxisome proliferator-activated receptors (PPARs) expressed by the NK cells. This is believed to decrease NK cell cytotoxic activity, and thereby enhance implantation A growing number of IVF programs, including ours, perform egg retrieval under conscious sedation using Propofol, a short acting hypnotic agent.

            Whatever the exact mechanism of action might be, Intralipid acts primarily to suppress NK cell over-production of TH-I cytokines. It exerts a modulating effect on certain immune cellular mechanisms largely by down-regulating cytotoxic /activated natural killer cells (NKa). This effect is enhanced through the concomitant administration of corticosteroids such as dexamethasone, prednisolone and prednisone which augment immune modulation of T cells. The combined effect of IL + steroid therapy suppresses pro-inflammatory cellular TH1 cytokines such as interferon gamma and TNF-alpha that are produced in excess by activated NK cells and cytotoxic lymphocytes/T-cells (CTL).   IL will, in about 80% of cases, successfully down-regulate activated natural killer cells (NKa) over a period of 2-3 weeks. It is likely to be just as effective as IVIg in this respect but at a fraction of the cost and with a far lower incidence of side-effects. Its effect lasts for ~ 4-6 weeks when administered in early pregnancy.

Intralipid is a suspension of soybean lipid droplets in water and is primarily used as source of parenteral nutrition. When administered intravenously, IL provides essential fatty acids, linoleic acid (LA), an omega-6 fatty acid, and alpha-linolenic acid (ALA), an omega-3 fatty acid.     

It is thought that fatty acids within the emulsion serve as ligands that activate peroxisome proliferator-activated receptors (PPARs) expressed by the NK cells. This is believed to decrease NK cell cytotoxic activity, and thereby enhance implantation A growing number of IVF programs, including ours, perform egg retrieval under conscious sedation using Propofol, a short acting hypnotic agent.

            Whatever the exact mechanism of action might be, Intralipid acts primarily to suppress NK cell over-production of TH-I cytokines. It exerts a modulating effect on certain immune cellular mechanisms largely by down-regulating cytotoxic /activated natural killer cells (NKa). This effect is enhanced through the concomitant administration of corticosteroids such as dexamethasone, prednisolone and prednisone which augment immune modulation of T cells. The combined effect of IL + steroid therapy suppresses pro-inflammatory cellular TH1 cytokines such as interferon gamma and TNF-alpha that are produced in excess by activated NK cells and cytotoxic lymphocytes/T-cells (CTL).   IL will, in about 80% of cases, successfully down-regulate activated natural killer cells (NKa) over a period of 2-3 weeks. It is likely to be just as effective as IVIg in this respect but at a fraction of the cost and with a far lower incidence of side-effects. Its effect lasts for ~ 4-6 weeks when administered in early pregnancy.

            Can laboratory testing be used to assess for an immediate effect of IL on Nka suppression?  Since the downregulation of NKa through IL (or IVIg) therapy can take several weeks to become measurable, it follows that there is really no benefit in trying to assess the potential efficacy of such treatment by retesting NKa in the laboratory after adding IL (or IVIg) to the cells being tested.

IVIg Therapy:  Until about a decade ago, the only effective and available way (in the US) to down-regulate activated NK cells was through the intravenous administration of a blood product known as immunoglobulin-G (IVIg). The fear (albeit unfounded) that the administration of this product might lead to the transmission of viral infections such as HIV and hepatitis C, plus the high cost of IVIG along with the fact that significant side effects occurred about 20% of the time, led to bad press and bad publicity for the entire field of reproductive immunology. It was easier for RE’s to simply say “I don’t believe IVIg works” and thereby avoid risk and bad publicity. But the thousands of women who had babies because of NK cell activity being downregulated through its use, attests to IVIg’s efficacy. But those of us who felt morally obligated to many desperate patients who would not conceive without receiving IVIg were facing an uphill battle. The bad press caused by fear mongering took its toll and spawned a malicious controversy. It was only through the introduction of IL less (about 15-20 years ago ), that the tide began to turn in favor of those patients who required low cost, safe and effective immunotherapy to resolve their IID.

 Corticosteroid Therapy (e.g., Prednisone, and Dexamethasone): Corticosteroid therapy has become a mainstay in the treatment of most women undergoing IVF. It is believed by most to enhance implantation due to an overall immunomodulatory effect. Corticosteroids reduce TH-1 cytokine production by CTL. When given in combination with IL or IVIG they augment the implantation process. The prednisone or dexamethasone therapy must commence (along with IL/IVIg) 10-14 days prior to egg retrieval and continue until pregnancy is discounted or until the 10th week of pregnancy.

 Heparinoid Therapy: There is compelling evidence that the subcutaneous administration of low molecular heparin (Clexane, Lovenox) once daily, (starting with the onset of ovarian stimulation) can improve IVF birthrate in women who test positive for APAs and might prevent later pregnancy loss when used to treat certain thrombophilias (e.g., homozygous MTHFR mutation)

What About Baby Aspirin? In our opinion, aspirin has little (if any) value when it comes to IID, and besides, it could even reduce the chance of success. The reason for this is that aspirin thins the blood and increases the potential to bleed. This effect can last for up to a week and could complicate an egg retrieval procedure or result in “concealed” intrauterine bleeding at the time of embryo transfer, thereby potentially compromising IVF success.

TH-1 Cytokine Blockers (Enbrel, Humira): TH-1 cytokine blockers, (Enbrel and Humira) are in our opinion relatively ineffective in the IVF setting. There has to date been no convincing data to support their use. However, these blockers could have a role in the treatment of a threatened miscarriage thought to be due to CTL/NK activation, but not for IVF. The reason is that the very initial phase of implantation requires a cellular response involving TH-1 cytokines. To block them completely (rather than simply restore a TH-1:TH-2 balance as occurs with IL therapy) so very early on could compromise rather than benefit implantation.

Leukocyte Immunization Therapy (LIT): The subcutaneous injection of the male partner’s lymphocytes to the mother is thought to enhance the ability for the mother’s decidua (uterus) to recognize the DQ alpha matching embryo as “self” or “friend” and thereby avert its rejection. LIT has been shown to up-regulate Treg cells and thus down-regulate NK cell activation thereby improving decidual TH-1:TH-2 balance. Thus, there could be a therapeutic benefit from such therapy. However, the same benefit can be achieved through the use of IL plus corticosteroids. Besides, IL is much less expensive, and the use of LIT is prohibited by law in the U.S.A.

There are two categories of immunologic implantation dysfunction (IID) linked to NK cell activation (NKa).

Autoimmune Implantation Dysfunction: Here, the woman will often have a personal or family history of autoimmune conditions such as Rheumatoid arthritis, Lupus Erythematosus, and thyroid autoimmune activity (e.g., Hashimoto’s disease) etc. Autoimmune as well as in about one third of cases of endometriosis, regardless of severity.  Autoimmune sometimes also occurs in the absence of a personal or family history of autoimmune disease.

When it comes to treating  NKa in  IVF cases complicated by autoimmune implantation dysfunction,  the combination of daily oral dexamethasone commencing with the onset of ovarian stimulation and continuing until the 10th week of pregnancy, combined with an initial infusion of IL (100ml, 20% Il dissolved in 500cc of saline solution, 10-14  days prior to PGT-normal embryo transfer and repeated once more (only), as  soon as the blood pregnancy test is positive), the anticipated chance of a viable pregnancy occurring within 2 completed IVF attempts (including fresh + frozen ET’s)  in women under 39Y (who have normal ovarian reserve)  is approximately  65%.

Alloimmune Implantation Dysfunction: Here, NK cell activation results from uterine exposure to an embryo derived through fertilization by a spermatozoon that shares certain genotypic (HLA/DQ alpha) similarities with that of the embryo recipient.

Partial DQ alpha/HLA match:  Couples who upon genotyping are shown to share only one DQ alpha/HLA gene are labeled as having a “partial match”. The detection of a “partial match” in association with NKa puts the couple at a considerable disadvantage with regard to IVF outcome. It should be emphasized however, that in the absence of associated Nka, DQ alpha/HLA matching whether “partial” or “total (see below) will NOT cause an IID. Since we presently have no way of determining which embryo carries a matching paternal DQ alpha gene, it follows that each embryo transferred will have about half the chance of propagating a viable pregnancy. Treatment of a partial DQ alpha/HLA match (+ Nka) involves the same IL, infusion as for autoimmune-Nka with one important caveat, namely that here we prescribe oral prednisone as adjunct therapy (rather than dexamethasone) and the IL infusion is repeated every 2-4 weeks following the diagnosis of pregnancy and continued until the 24th week of gestation. Additionally, (as alluded to elsewhere) in such cases we transfer a single (1) embryo at a time. This is because, the likelihood is that one out of two embryos will “match” and we are fearful that if we transfer >1 embryo, and one transferred embryos “matches” it could cause further activation of uterine NK cells and so prejudice the implantation of all transferred embryos. Here it should be emphasized that if associated with Nka, a matching embryo will still be at risk of rejection even in the presence of Intralipid (or IVIg) therapy.

Total (complete) DQ alpha Match:   Here the husband’s DQ alpha genotype matches both of that of his partner’s. While this occurs very infrequently, a total alloimmune (DQ alpha) match with accompanying Nka, means that the chance of a viable pregnancy resulting in a live birth at term, is unfortunately greatly diminished.  Several instances in our experience have required the use of a gestational surrogate.

It is indeed unfortunate that so many patients are being denied the ability to go from “infertility to family” simply because (for whatever reason) so many reproductive specialists refuse to embrace the role of immunologic factors in the genesis of intractable reproductive dysfunction. Hopefully this will change, and the sooner the better.

______________________________________________________________________

I urge you to  visit my website at  www.SherIVF.com. Then go to my Blog and access the “search bar”. Type in the titles of any/all of the articles listed below, one by one. “Click” and you will immediately be taken to those you select.  Please also take the time to post any questions or comments with the full expectation that I will (as always) respond promptly.

 

  • A Fresh Look at the Indications for IVF
  • The IVF Journey: The importance of “Planning the Trip” Before Taking the Ride”
  • Controlled Ovarian Stimulation (COS) for IVF: Selecting the ideal protocol
  • IVF: Factors Affecting Egg/Embryo “competency” during Controlled Ovarian Stimulation(COS)
  • The Fundamental Requirements For Achieving Optimal IVF Success
  • Use of GnRH Antagonists (Ganirelix/Cetrotide/Orgalutron) in IVF-Ovarian Stimulation Protocols.
  • Human Growth Hormone Administration in IVF: Does it Enhances Egg/Embryo Quality and Outcome?
  • IVF and the use of Supplementary Human Growth Hormone (HGH) : Is it Worth Trying and who needs it?
  • The BCP: Does Launching a Cycle of Controlled Ovarian Stimulation (COS). Coming off the BCP Compromise Response?
  • Blastocyst Embryo Transfers Should be the Standard of Care in IVF
  • Anti Mullerian Hormone (AMH) Measurement to Assess Ovarian Reserve and Design the Optimal Protocol for Controlled Ovarian Stimulation (COS) in IVF.
  • IVF: Approach to Selecting the Best Embryos for Transfer to the Uterus.
  • Fresh versus Frozen Embryo Transfers (FET) Enhance IVF Outcome
  • Frozen Embryo Transfer (FET): A Rational Approach to Hormonal Preparation and How new Methodology is Impacting IVF.
  • Genetically Testing Embryos for IVF
  • Staggered IVF
  • Staggered IVF with PGS- Selection of “Competent” Embryos Greatly Enhances the Utility & Efficiency of IVF.
  • Preimplantation Genetic Testing (PGS) in IVF: It should be Used Selectively and NOT be Routine.
  • IVF: Selecting the Best Quality Embryos to Transfer
  • Preimplantation Genetic Sampling (PGS) Using: Next Generation Gene Sequencing (NGS): Method of Choice.
  • PGS in IVF: Are Some Chromosomally abnormal Embryos Capable of Resulting in Normal Babies and Being Wrongly Discarded?
  • PGS and Assessment of Egg/Embryo “competency”: How Method, Timing and Methodology Could Affect Reliability
  • Endometrial Receptivity Array (ERA): Is There an actual “There, There”?
  • IVF Failure and Implantation Dysfunction:
  • Diagnosing and Treating Immunologic Implantation Dysfunction (IID)
  • The Role of Immunologic Implantation Dysfunction (IID) & Infertility (IID):PART 1-Background
  • Immunologic Implantation Dysfunction (IID) & Infertility (IID):PART 2- Making a Diagnosis
  • Immunologic Dysfunction (IID) & Infertility (IID):PART 3-Treatment
  • Thyroid autoantibodies and Immunologic Implantation Dysfunction (IID)
  • Immunologic Implantation Dysfunction: Importance of Meticulous Evaluation and Strategic Management:(Case Report
  • Intralipid and IVIG therapy: Understanding the Basis for its use in the Treatment of Immunologic Implantation Dysfunction (IID)
  • Intralipid (IL) Administration in IVF: It’s Composition; How it Works; Administration; Side-effects; Reactions and Precautions
  • Natural Killer Cell Activation (NKa) and Immunologic Implantation Dysfunction in IVF: The Controversy!
  • Endometrial Thickness, Uterine Pathology and Immunologic Factors
  • Vaginally Administered Viagra is Often a Highly Effective Treatment to Help Thicken a Thin Uterine Lining
  • A Thin Uterine Lining: Vaginal Viagra is Often the Answer (update)
  • Cervical Ureaplasma Urealyticum Infection: How can it Affect IUI/IVF Outcome?
  • The Role of Nutritional Supplements in Preparing for IVF
  • The Basic Infertility Work-Up
  • Defining and Addressing an Abnormal Luteal Phase
  • Male Factor Infertility
  • Routine Fertilization by Intracytoplasmic Sperm Injection (ICSI): An Argument in Favor
  • Hormonal Treatment of Male Infertility
  • Hormonal Treatment of Male Infertility
  • Antisperm Antibodies, Infertility and the Role of IVF with Intracytoplasmic Sperm Injection (ICSI)
  • Endometriosis and Infertily
  • Endometriosis and Immunologic Implantation Dysfunction (IID) and IVF
  • Endometriosis and Infertility: Why IVF Rather than IUI or Surgery Should be the Treatment of Choice.
  • Endometriosis and Infertility: The Influence of Age and Severity on Treatment Options
  • Early -Endometriosis-related Infertility: Ovulation Induction (with or without Intrauterine Insemination) and Reproductive Surgery  Versus IVF
  • Deciding Between Intrauterine Insemination (IUI) and In Vitro Fertilization (IVF).
  • Intrauterine Insemination (IUI): Who Needs it & who Does Not: Pro’s & Con’s!IUI-Reflecting upon its Use and Misuse: Time for a Serious “Reality Check
  • Mode of Action, Indications, Benefits, Limitations and Contraindications for its ue
  • Clomiphene Induction of Ovulation: Its Use and Misuse!

 

 

 

Name: Yvonne W

Hi Dr Sher, I wish you had never left South Africa, as that is where I am from.

I am 33 years old, we went to do ivf as my fiance is paralysed. He was placed on meds to improve sperm quality and a TESA procedure was done, they got 12 straws. First fresh transfer was on 1 March, had a missed miscarriage at 8 weeks after seeing the heartbeat – HR at 6 week scan was 104. We did genetic testing on products of conception and it came back as genetically normal female. My Dr… Dr Antonio Rodrigues from Medfem fertility sent me for the miscarriage panel of tests, everything was normal, had slightly raised protein C.

For the next FET on 2 June Dr put me on the immunoglobulin protocol with ecotrin, beriglobin injections, progesterone and prednisone. At 5 weeks I started bleeding, but it stopped after about an hour. First scan at 6 weeks showed a heartbeat of 103… Dr asked me to come back in 4 days to see if it had improved… But it was at about 80… 4 days after that at the next scan, there was no heartbeat. Genetically normal male.

I have endo stage 2/3, Dr did a hysteroscopy and laparoscopy before we started ivf in Jan this year.

We did a uterine biopsy and sent the tissues to the USA for the EMMA and ALICE tests… Results came back that I have the gardnerella bacteria in my uterus. Dr treated me with Flagyl and gyna guard vaginal probiotics. We will repeat the antibiotics and probiotics when we do the next transfer in November.

I also asked to repeat the miscarriage panel of tests and I got a low aPTT.

My concern is… My mother has SLE and RA, auto immune diseases run in my family, but all my testing is clear. I don’t doubt My Dr at all, he is amazing. But I really want to do everything we can this time.

Would it help to rather receive clexane and intra lipids vs ecotrin and beriglobin injections?

In SA there aren’t many highly specialized labs like in USA. So we are doing what we can from here. I’m currently on Stims, next scan on Thursday and then egg retrieval next week, sending the embryos for genetic testing before transfer.

Is there anything else you know that might help, or anything you can recommend.

Thank you so much.

Answer:

I really sympathize with your predicament but there is no substitute for an appropriate and comprehensive medical evaluation of your problem….especially when it comes to an evaluation of immunologic implantation dysfunction.

I urge you to download a free copy of my E-book , “Recurrent Pregnancy Loss and Unexplained IVF Failure: The Immunologic Link”,  by clicking on: :https://drive.google.com/file/d/1iYKz-EkAjMqwMa1ZcufIloRdxnAfDH8L/view

 

When it comes to reproduction, humans are the poorest performers of all mammals. In fact we are so inefficient that up to 75% of fertilized eggs do not produce live births, and up to 30% of pregnancies end up being lost within 10 weeks of conception (in the first trimester). RPL is defined as two (2) or more failed pregnancies. Less than 5% of women will experience two (2) consecutive miscarriages, and only 1% experience three or more.

Pregnancy loss can be classified by the stage of pregnancy when the loss occurs:

  • Early pregnancy loss (first trimester)
  • Late pregnancy loss (after the first trimester)
  • Occult “hidden” and not clinically recognized, (chemical) pregnancy loss (occurs prior to ultrasound confirmation of pregnancy)
  • Early pregnancy losses usually occur sporadically (are not repetitive).

 

In more than 70% of cases the loss is due to embryo aneuploidy (where there are more or less than the normal quota of 46 chromosomes). Conversely, repeated losses (RPL), with isolated exceptions where the cause is structural (e.g., unbalanced translocations), are seldom attributable to numerical chromosomal abnormalities (aneuploidy). In fact, the vast majority of cases of RPL are attributable to non-chromosomal causes such as anatomical uterine abnormalities or Immunologic Implantation Dysfunction (IID).

Since most sporadic early pregnancy losses are induced by chromosomal factors and thus are non-repetitive, having had a single miscarriage the likelihood of a second one occurring is no greater than average. However, once having had two losses the chance of a third one occurring is double (35-40%) and after having had three losses the chance of a fourth miscarriage increases to about 60%. The reason for this is that the more miscarriages a woman has, the greater is the likelihood of this being due to a non-chromosomal (repetitive) cause such as IID. It follows that if numerical chromosomal analysis (karyotyping) of embryonic/fetal products derived from a miscarriage tests karyotypically normal, then by a process of elimination, there would be a strong likelihood of a miscarriage repeating in subsequent pregnancies and one would not have to wait for the disaster to recur before taking action. This is precisely why we strongly advocate that all miscarriage specimens be karyotyped.

There is however one caveat to be taken into consideration. That is that the laboratory performing the karyotyping might unwittingly be testing the mother’s cells rather than that of the conceptus. That is why it is not possible to confidently exclude aneuploidy in cases where karyotyping of products suggests a “chromosomally normal” (euploid) female.

Late pregnancy losses (occurring after completion of the 1st trimester/12th week) occur far less frequently (1%) than early pregnancy losses. They are most commonly due to anatomical abnormalities of the uterus and/or cervix. Weakness of the neck of the cervix rendering it able to act as an effective valve that retains the pregnancy (i.e., cervical incompetence) is in fact one of the commonest causes of late pregnancy loss. So also are developmental (congenital) abnormalities of the uterus (e.g., a uterine septum) and uterine fibroid tumors. In some cases intrauterine growth retardation, premature separation of the placenta (placental abruption), premature rupture of the membranes and premature labor can also causes of late pregnancy loss.

Much progress has been made in understanding the mechanisms involved in RPL. There are two broad categories:

  1. Problems involving the uterine environment in which a normal embryo is prohibited from properly implanting and developing. Possible causes include:
  • Inadequate thickening of the uterine lining
  • Irregularity in the contour of the uterine cavity (polyps, fibroid tumors in the uterine wall, intra-uterine scarring and adenomyosis)
  • Hormonal imbalances (progesterone deficiency or luteal phase defects). This most commonly results in occult RPL.
  • Deficient blood flow to the uterine lining (thin uterine lining).
  • Immunologic implantation dysfunction (IID). A major cause of RPL. Plays a role in 75% of cases where chromosomally normal preimplantation embryos fail to implan
  • Interference of blood supply to the developing conceptus can occur due to a hereditary clotting disorder known as Thrombophili

 

  1. Genetic and/or numerical chromosomal abnormalities(aneuploidy) of the embryo are far away the commonest overall causes of miscarriages. But this only applies to sporadic pregnancy losses (which comprises the majority of all miscarriages. However, recurrent, (consecutive) pregnancy losses are much more likely due to implantation dysfunction than to embryo-related issues, where implantation dysfunction (usually anatomical or immunologic) factors usually underly the problem.
  2. Genetic or Structural chromosomal abnormalities (which only occur in about 1% of cases) can also cause RPL. This is referred to as an unbalanced translocation and they result from part of one chromosome detaching and then fusing with another chromosome. Additionally, genetic defects (unrelated to chromosomal abnormalities) can also affect embryo quality and pregnancy outcome. Damaged sperm DNA can sometimes be diagnosed using the SCSA (see before) which primarily measures the sperm DNA fragmentation index (DFI).

 

IMMUNOLOGIC IMPLANTATION DYSFUNCTIO-IID (see before)

Autoimmune IID: Here an immunologic reaction is produced by the individual to his/her body’s own cellular components. The most common antibodies that form in such situations are APA and antithyroid antibodies

Alloimmune IID, i.e., where there is an immunologic reaction to antigens derived from another member of the same species (i.e. the woman’s immune system reacts to the paternal antigens in the sperm (see above) .

*It is important to recognize that alloimmune (rather than autoimmune) IID is more commonly associated with RPL.

Autoimmune IID is often genetically transmitted. Thus, it should not be surprising to learn that it is more likely to exist in women who have a family (or personal) history of primary autoimmune diseases such as lupus erythematosus (LE), scleroderma or autoimmune hypothyroidism (Hashimoto’s disease), autoimmune hyperthyroidism (Grave’s disease), rheumatoid arthritis, etc. Reactionary (secondary) autoimmunity can occur in conjunction with any medical condition associated with widespread tissue damage. One such gynecologic condition is endometriosis. Since autoimmune IID is usually associated with activated NK and T-cells from the outset, it usually results in such very early destruction of the embryo’s root system that the patient does not even recognize that she is pregnant. Accordingly, the condition usually presents as “unexplained infertility” or “unexplained IVF failure” rather than as a miscarriage.

Alloimmune IID, on the other hand, usually starts off presenting as unexplained miscarriages (often manifesting as RPL). Over time as NK/T cell activation builds and eventually becomes permanently established the patient often goes from RPL to “infertility” due to failed implantation. RPL is more commonly the consequence of alloimmune rather than autoimmune implantation dysfunction.

However, regardless, of whether miscarriage is due to autoimmune or alloimmune implantation dysfunction the final blow to the pregnancy is the result of activated NK cells and CTL in the uterine lining that damage the developing embryo’s “root system” (trophoblast) so that it can no longer sustain the growing conceptus. This having been said, it is important to note that autoimmune IID is readily amenable to reversal through timely, appropriately administered, selective immunotherapy, and alloimmune IID is not. It is much more difficult to treat successfully, even with the use of immunotherapy. In fact, in some cases the only solution will be to revert to selective immunotherapy plus using donor sperm (provided there is no “match” between the donor’s DQ alpha/HLA profile and that of the female recipient) or alternatively to resort to gestational surrogacy.

DIAGNOSING THE CAUSE OF RPL

In the past, women who miscarried were not evaluated thoroughly until they had lost several pregnancies in a row. This was because sporadic miscarriages are most commonly the result of embryo numerical chromosomal irregularities (aneuploidy) and thus not treatable. However, a consecutive series of miscarriages points to a repetitive cause that is non-chromosomal and is potentially remediable. Since RPL is most commonly due to a uterine pathology or immunologic causes that are potentially treatable, it follows that early chromosomal evaluation of products of conception could point to a potentially treatable situation. Thus, we strongly recommend that such testing be done in most cases of miscarriage. Doing so will avoid a great deal of unnecessary heartache for many patients.

Establishing the correct diagnosis is the first step toward determining effective treatment for couples with RPL. It results from a problem within the pregnancy itself or within the uterine environment where the pregnancy implants and grows. Diagnostic tests useful in identifying individuals at greater risk for a problem within the pregnancy itself include:

 

  • Karyotyping (chromosome analysis) both prospective parents
  • Assessment of the karyotype of products of conception derived from previous miscarriage specimens
  • Ultrasound examination of the uterine cavity after sterile water is injected or sonohysterogram, fluid ultrasound, etc.
  • Hysterosalpingogram (dye X-ray test)
  • Hysteroscopic evaluation of the uterine cavity
  • Full hormonal evaluation (estrogen, progesterone, adrenal steroid hormones, thyroid hormones, FSH/LH, )
  • Immunologic testing to include:
    • Antiphospholipid antibody (APA) panel
    • Antinuclear antibody (ANA) panel
    • Antithyroid antibody panel (i.e., antithyroglobulin and antimicrosomal antibodies)
    • Reproductive immunophenotype
    • Natural killer cell activity (NKa) assay (i.e., K562 target cell test)
    • Alloimmune (DQ alpha/HLA) testing of both the male and female partners

 

TREATMENT OF RPL

Treatment for Anatomic Abnormalities of the Uterus: This involves restoration through removal of local lesions such as fibroids, scar tissue, and endometrial polyps or timely insertion of a cervical cerclage (a stitch placed around the neck of the weakened cervix) or the excision of a uterine septum when indicated.

Treatment of Thin Uterine Lining: A thin uterine lining has been shown to correlate with compromised pregnancy outcome. Often this will be associated with reduced blood flow to the endometrium.  Such decreased blood flow to the uterus can be improved through treatment with sildenafil and possibly aspirin.

Sildenafil (Viagra) Therapy (see above). Viagra has been used successfully to increase uterine blood flow. To date, we have seen significant improvement of the thickness of the uterine lining in about 70% of women treated. Successful pregnancy resulted in almost half of those women who responded to the Viagra. It should be borne in mind that most of these women had previously experienced repeated IVF failures.

Use of Aspirin: This is an anti-prostaglandin that improves blood flow to the endometrium. It is administered at a dosage of 81 mg orally, daily from the beginning of the cycle until ovulation.

Treating Immunologic Implantation Dysfunction with Selective Immunotherapy: Modalities such as IL/IVIg, heparinoids (Lovenox/Clexane), and corticosteroids can be used in select cases depending on autoimmune or alloimmune dysfunction.

The Use of IVF in the Treatment of RPL. In the following circumstances, IVF is the preferred option:

  • When in addition to a history of RPL, another standard indication for IVF (e.g., tubal factor, endometriosis, and male factor infertility) is superimposed.
  • In cases where selective immunotherapy is needed to treat an immunologic implantation dysfunction.

The reason for IVF being a preferred approach in such cases is that in order to be effective, the immunotherapy needs to be initiated well before spontaneous or induced ovulation. Given the fact that in the absence of IVF the anticipated birthrate per cycle of COS with or without IUI is at best about 15%, it follows that short of IVF, to have even a reasonable chance of a live birth, most women with immunologic causes of RPL would need to undergo immunotherapy repeatedly, over consecutive cycles. Conversely, with IVF, the chance of a successful outcome in a single cycle of treatment is several times greater and, because of the attenuated and concentrated time period required for treatment, IVF is far safer and thus represents a more practicable alternative

Since embryo aneuploidy is a common cause of miscarriage, the use of PGS/PGT-A can provide a valuable diagnostic and therapeutic advantage in cases of RPL. PGD requires IVF to provide access to embryos for testing.

There are a few cases of intractable alloimmune dysfunction due to “complete DQ alpha matching where Gestational Surrogacy or use of Donor  Sperm could represent the only viable recourse, other than abandoning treatment altogether and/or resorting to adoption. Other non-immunologic factors such as an intractably thin uterine lining or severe uterine pathology might also warrant that last resort consideration be given to gestational surrogacy.

The good news is that if a couple with RPL is open to all of the diagnostic and treatment options referred to above, a live birthrate of 70%–80% is ultimately achievable.

I strongly recommend that you visit www.SherIVF.com. Then go to my Blog and access the “search bar”. Type in the titles of any/all of the articles listed below, one by one. “Click” and you will immediately be taken to those you select.  Please also take the time to post any questions or comments with the full expectation that I will (as always) respond promptly.

  • The IVF Journey: The importance of “Planning the Trip” Before Taking the Ride”
  • Controlled Ovarian Stimulation (COS) for IVF: Selecting the ideal protocol
  • IVF: Factors Affecting Egg/Embryo “competency” during Controlled Ovarian Stimulation(COS)
  • The Fundamental Requirements For Achieving Optimal IVF Success
  • Ovarian Stimulation for IVF using GnRH Antagonists: Comparing the Agonist/Antagonist Conversion Protocol.(A/ACP) With the “Conventional” Antagonist Approach
  • Ovarian Stimulation in Women Who have Diminished Ovarian Reserve (DOR): Introducing the Agonist/Antagonist Conversion protocol
  • Anti Mullerian Hormone (AMH) Measurement to Assess Ovarian Reserve and Design the Optimal Protocol for Controlled Ovarian Stimulation (COS) in IVF.
  • Human Growth Hormone Administration in IVF: Does it Enhances Egg/Embryo Quality and Outcome?
  • The BCP: Does Launching a Cycle of Controlled Ovarian Stimulation (COS). Coming off the BCP Compromise Response?
  • Blastocyst Embryo Transfers Should be the Standard of Care in IVF
  • IVF: How Many Attempts should be considered before Stopping?
  • “Unexplained” Infertility: Often a matter of the Diagnosis Being Overlooked!
  • IVF Failure and Implantation Dysfunction:
  • The Role of Immunologic Implantation Dysfunction (IID) & Infertility (IID):PART 1-Background
  • Immunologic Implantation Dysfunction (IID) & Infertility (IID):PART 2- Making a Diagnosis
  • Immunologic Dysfunction (IID) & Infertility (IID):PART 3-Treatment
  • Thyroid autoantibodies and Immunologic Implantation Dysfunction (IID)
  • Immunologic Implantation Dysfunction: Importance of Meticulous Evaluation and Strategic Management:(Case Report
  • Intralipid and IVIG therapy: Understanding the Basis for its use in the Treatment of Immunologic Implantation Dysfunction (IID)
  • Intralipid (IL) Administration in IVF: It’s Composition; How it Works; Administration; Side-effects; Reactions and Precautions
  • Natural Killer Cell Activation (NKa) and Immunologic Implantation Dysfunction in IVF: The Controversy!
  • Endometrial Thickness, Uterine Pathology and Immunologic Factors
  • Vaginally Administered Viagra is Often a Highly Effective Treatment to Help Thicken a Thin Uterine Lining
  • Treating Out-of-State and Out-of-Country Patients at Sher-IVF in Las Vegas:
  • A personalized, stepwise approach to IVF
  • How Many Embryos should be transferred: A Critical Decision in IVF.
  • The Role of Nutritional Supplements in Preparing for IVF

 

 

______________________________________________________

ADDENDUM: PLEASE READ!!

INTRODUCING SHER FERTILITY SOLUTIONS (SFS)

Founded in April 2019, Sher Fertility Solutions (SFS) offers online (Skype/FaceTime) consultations to patients from > 40 different countries. All consultations are followed by a detailed written report presenting my personal recommendations for treatment of what often constitute complex Reproductive Issues.

 

If you wish to schedule an online consultation with me, please contact my assistant (Patti Converse) by phone (800-780-7437/702-533-2691), email (concierge@SherIVF.com) or,  enroll online on then home-page of my website (www.SherIVF.com). 

 

PLEASE SPREAD THE WORD ABOUT SFS!

 

Geoff Sher

 

 

Name: Cassidy O

Hi Dr Sher,

I hope this message finds you very well!

I’m turning 44 soon and have finished the embryo banking process (2 euploid embryos + 7 LLM embryos). Clearly I’m unlikely to retrieve any more euploid embryos at my age.

I’ve completed two FETs unsuccessfully with untested embryos, assume reason for failure was embryo competency (with hindsight should have tried the Euploids in priority).

However, I note from your articles that there are multiple reasons for failures, some of which are presumably not yet known.

From your experience – is age still a factor where euploid embryos are concerned, is the reproductive potential higher/significantly higher in younger women? (euploid embryos age 42).

Also, I’m thinking of adopting a kitchen sink approach now, I’ve asked for additional testing at an implantation clinic – it will delay transfer by a couple of months – do you think that is sensible? Noting that reason for failure is not known, but better to optimise circumstances, where possible, in my view.

Many thanks in advance!
Best wishes to you and you loved ones!

Cassidy xx

Answer:

Thank you for your communication. Although age primarily affects IVF outcome through impacting the risk of aneuploidy, it also presumably has an effect via other factors (metabolomic and epigenewtic) although these are far less relevant.

In my opinion, you would be well served to evaluation for embryo implantation dysfunction before doing another transfer.

Implantation dysfunction is unfortunately often overlooked as an important cause of IVF failure. This is especially relevant in cases of unexplained IVF failure, recurrent pregnancy loss (RPL), and in women with underlying endo-uterine surface lesions, thickness of the uterine lining (endometrium) and /or immunologic factors.

IVF success rates have been improving over the last decade. The average live birth rate per embryo transfer in the U.S.A for women under 40y using their own eggs is currently better than 1:3 women. However, there is still a wide variation from program to program for IVF live birth rates, ranging from 20% to near 50%. Based upon these statistics, most women undergoing IVF in the United States require two or more attempts to have a baby. IVF practitioners in the United States commonly attribute the wide dichotomy in IVF success rates to variability in expertise of the various embryology laboratories. This is far from accurate. In fact, other factors such as wide variations in patient selection and the failure to develop individualized protocols for ovarian stimulation or to address those infectious, anatomical, and immunologic factors that influence embryo implantation are at least equally important.

About 80% of IVF failures are due to “embryo incompetency” that is largely due to aneuploidy      usually related to advancing age of the woman and is further influenced by other factors such as the protocol selected for ovarian stimulation, diminished ovarian reserve (DOR), and severe male factor infertility. However, in about 20% of dysfunctional cases embryo implantation is the cause of failure.

This section will focus on implantation dysfunction and IVF failure due to:

ANATOMICAL ENDO-UTERINE SURFACE LESIONS

            It has long been suspected that anatomical defects of the uterus might result in infertility. While the presence of uterine fibroids, in general, are unlikely to cause infertility, an association between their presence and infertility has been observed in cases where the myomas distort the uterine cavity or protrude through the endometrial lining.  Even small fibroids that lie immediately under the endometrium (submucous fibroids) and protrude into the uterine cavity have the potential to lower embryo implantation.  Multiple fibroids in the uterine wall (intramural fibroids) that encroach upon the uterine cavity can sometimes so compromise blood flow that estrogen delivery is impaired, and the endometrium is unable to thicken properly. This can usually be diagnosed by ultrasound examination during the proliferative phase of the menstrual cycle.  It is likely that any surface lesion in the uterine cavity, whether submucous fibroids, intrauterine adhesions a small endometrial or a placental polyp, has the potential to interfere with implantation by producing a local inflammatory response, not too dissimilar in nature from that which is caused by an intrauterine contraceptive device (IUD).       

            Clearly, since even small uterine lesions have the potential to adversely affect implantation, the high cost (financial, physical, and emotional) associated with IVF and related procedures, justifies the routine performance of diagnostic procedures such as an HSG, hysterosonogram (fluid ultrasound examination), or hysteroscopy prior to initiating IVF.  Identifiable uterine lesions that have the potential of impairing implantation usually require surgical intervention.  In most cases, dilatation and curettage (D & C) or hysteroscopic resection will suffice. Some cases might require the performance of a laparotomy.  Such intervention will often result in subsequent improvement of the endometrial response.      

Sonohysterography [Fluid ultrasonography (FUS)]: Fluid ultrasonography is a procedure whereby a sterile solution of saline is injected via a catheter through the cervix and into the uterine cavity. The fluid-distended cavity is examined by vaginal ultrasound for any irregularities that might point to surface lesions such as polyps, fibroid tumors, scarring, or a uterine septum. If performed by an expert, a FUS is highly effective in recognizing even the smallest lesion and can replace hysteroscopy under such circumstances. FUS is less expensive, less traumatic, and equally as effective as hysteroscopy. The only disadvantage lies in the fact that if a lesion is detected, it may require the subsequent performance of hysteroscopy to treat the problem anyway.

Hysteroscopy: Diagnostic hysteroscopy is an office procedure that is performed under intravenous sedation, general anesthesia, or paracervical block with minimal discomfort to the patient. This procedure involves the insertion of a thin, lighted, telescope like instrument known as a hysteroscope through the vagina and cervix into the uterus to fully examine the uterine cavity. The uterus is first distended with normal saline, which is passed through a sleeve adjacent to the hysteroscope. As is the case with FUS, diagnostic hysteroscopy facilitates examination of the inside of the uterus under direct vision for defects that might interfere with implantation. We have observed that approximately one in eight candidates for IVF have lesions that require attention prior to undergoing IVF in order to optimize the chances of a successful outcome. We strongly recommend that all patients undergo therapeutic surgery (usually by hysteroscopy) to correct the pathology prior to IVF.  Depending on the severity and nature of the pathology, therapeutic hysteroscopy may require general anesthesia and, in such cases, should be performed in an outpatient surgical facility or conventional operating room where facilities are available for laparotomy, a procedure in which an incision is made in the abdomen to expose the abdominal contents for diagnosis, or for surgery should this be required.       

THICKNESS OF THE UTERINE LINING (ENDOMETRIUM):

As far back as in 1989 we first reported on the finding that ultrasound assessment of the late proliferative phase endometrium can identify those candidates who are least likely to conceive. We noted that the ideal thickness of the endometrium at the time of ovulation or egg retrieval is >8 mm and that thinner linings are associated with decreased implantation rates.

More than 30 years ago we first showed that in normal and “stimulated” cycles, pre-ovulatory endometrial thickness and ultrasound appearance is predictive of embryo implantation (pregnancy) potential following ET. With conventional IVF and with FET, endometrial lining at the time of the “trigger shot” or with the initiation of progesterone needs to preferably be at least 8 mm in sagittal thickness with a triple line (trilaminar) appearance. Anything less than an 8mm endometrial thickness       is associated with a reduction in live birth rate per ET. An 8-9mm thickness represents a transitional measurement…a “gray zone”.  Hitherto, attempts to augment endometrial growth in women with poor endometrial linings by bolstering circulating estrogen blood levels (through the administration of increased doses of fertility drugs, aspirin administration and by supplementary estrogen therapy) yielded disappointing results.

            A “poor” uterine lining is usually the result of the innermost layer of endometrium (the basal or germinal endometrium from which endometrium grows) not being able to respond to estrogen by propagating an outer, “functional” layer thick enough to support optimal embryo implantation and development of a healthy placenta (placentation). The “functional” layer ultimately comprises 2/3 of the full endometrial thickness and is the layer that sheds with menstruation if no pregnancy occurs.

The main causes of a “poor” uterine lining are:

  • Damage to the basal endometrium because of:
  • Inflammation of the endometrium (endometritis) most commonly resulting from infected products left over following abortion, miscarriage, or birth
  • Surgical trauma due to traumatic uterine scraping, (i.e. due to an over-aggressive D & C)
  • Insensitivity of the basal endometrium to estrogen due to:
  • Prolonged, over-use/misuse of clomiphene citrate
  • Prenatal exposure to diethylstilbestrol (DES). This is a drug that was given to pregnant women in the 1960’s to help prevent miscarriage
  • Over-exposure of the uterine lining to ovarian male hormones (mainly testosterone): Older women, women with diminished ovarian reserve (poor responders) and women with polycystic ovarian syndrome -PCOS tend to have raised LH biological activity. This causes the connective tissue in the ovary (stroma/theca) to overproduce testosterone. The effect may be further exaggerated when certain methods for ovarian stimulation such as “flare” protocols and high dosages of Menopur are used in such cases.
  • Reduced blood flow to the basal endometrium: Examples include.
  • Multiple uterine fibroids – especially when these are present under the endometrium (submucosal)
  • Uterine adenomyosis (excessive, abnormal invasion of the uterine muscle by endometrial glands).

Vaginal Viagra: About 35 years ago, after reporting on the benefit of administering vaginal Sildenafil (Viagra) to women who had implantation dysfunction due to thin endometrial linings we announced the birth of the world’s first “Viagra baby.”  Viagra administered vaginally, but not orally, in affected women improves uterine blood flow causing more estrogen to be delivered to the basal endometrium and increasing the endometrial thickening.  Following vaginal administration, Viagra is rapidly absorbed and quickly reaches the uterine blood system in high concentrations. Thereupon it dilutes out as it is absorbed into the systemic circulation. This probably explains why treatment is virtually devoid of systemic side effects.  It is important to recognize that Viagra will NOT be effective in improving endometrial thickness in all cases. In fact, about one third of women treated fail to show any improvement. This is because in certain cases of thin uterine linings, the basal endometrium will have been permanently damaged and left unresponsive to estrogen. This happens in cases of severe endometrial damage due mainly to post-pregnancy endometritis (inflammation), chronic granulomatous inflammation due to uterine tuberculosis (hardly ever seen in the United States) and following extensive surgical injury to the basal endometrium (as sometimes occurs following over-zealous D&C’s).

  • Immunologic factors: These also play a role in IVF failure (see “Immunologic factors and Implantation” …see below.

IMMUNOLOGIC IMPLANTATION DYSFUNCTION (IID)

            Currently, with few exceptions, practitioners of assisted reproduction tend to attribute “unexplained and/or repeated” IVF failure(s), almost exclusively to poor embryo quality, advocating adjusted protocols for ovarian stimulation and/or gamete and embryo preparation as a potential remedy. The idea, having failed IVF, that all it takes to ultimately succeed is to keep trying the same recipe is over-simplistic.

            The implantation process begins six or seven days after fertilization of the egg. At this time, specialized embryonic cells (i.e., trophoblasts), that later become the placenta begin growing into the uterine lining. When the trophoblast and the uterine lining meet, they, along with immune cells in the lining, become involved in a “cross talk” through mutual exchange of hormone-like substances called cytokines. Because of this complex immunologic interplay, the uterus can foster the embryo’s successful growth. Thus, from the earliest stage, the trophoblast establishes the very foundation for the nutritional, hormonal and respiratory interchange between mother and baby.  In this manner, the interactive process of implantation is not only central to survival in early pregnancy but also to the quality of life after birth.

There is an ever growing realization, recognition, and acceptance of the fact that uterine immunologic dysfunction can lead to immunologic implantation dysfunction (IID) with “unexplained” infertility, IVF failure, and recurrent pregnancy loss (RPL).

DIAGNOSIS 

Because immunologic problems may lead to implantation failure, it is important to properly evaluate women with risk factors such as:

  • Unexplained or recurrent IVF failures
  • Unexplained infertility or a family history of autoimmune diseases (e.g., rheumatoid arthritis, lupus erythematosus and hypothyroidism).
  • Recurrent Pregnancy Loss (RPL)
  • Endometriosis
  • A personal or family history of autoimmune conditions, e.g., Rheumatoid Arthritis, Lupus erythematosus, autoimmune hypothyroidism (Hashimoto’s disease) etc.

            Considering its importance, it is not surprising that the failure of a properly functioning immunologic interaction during implantation has been implicated as a cause of recurrent miscarriage, late pregnancy fetal loss, IVF failure and infertility. A partial list of immunologic factors that may be involved in these situations includes:

  • Antiphospholipid antibodies (APA)
  • Antithyroid antibodies (ATA/AMA)
  • Activated natural killer cells (NKa)

ACTIVATED NATURAL KILLER CELLS (NKa):

Following ovulation and during early pregnancy, NK cells and T-cells comprise more than 80% of the lymphocyte-immune cells that frequent the uterine lining. These lymphocytes (white blood cells) journey from the bone marrow to the uterus and under hormonal regulation, proliferate there. After exposure to progesterone (due to induced /spontaneous exogenous administration), they begin to produce TH-1 and TH-2 cytokines. TH-2 cytokines are humoral in nature and induce the trophoblast (“root system of the embryo”) to permeate the uterine lining while TH-I cytokines induce a process referred to as apoptosis (cell suicide) thereby confining placental development to the inner part of the uterus. Optimal placental development (placentation) mandates that there be a balance between TH1 and TH-2 cytokines. Most of the cytokine production originates from NK cells (rather than from cytotoxic T-cells/Lymphocytes (CTL)). Excessive production/release of TH-1 cytokines, is toxic to the trophoblast and to endometrial cells, leading to programmed death/suicide (apoptosis) and subsequently to IID.

Functional NK cells reach a maximal concentration in the endometrium by about t day 6-7 days after exposure to progesterone …. This timing corresponds with when the embryo implants into the uterine lining (endometrium).

It is important to bear in mind that measurement of the concentration of blood NK cells has little or no relevance when it comes to assessing NK cell activation (NKa). Rather, it is the NK cell activation that matters. In fact, there are certain conditions (such as with endometriosis) where the NK cell blood concentration is below normal, but NK cell activation is markedly increased.

There are several methods by which NK cell activation (cytotoxicity) can be assessed in the laboratory. Methods such as immunohistochemical assessment of uterine NK cells and/or through measurement of uterine or blood TH-1 cytokines. However, the K-562 target cell blood test still remains the gold standard. With this test, NK cells, isolated from the woman’s blood using Flow Cytometry are incubated in the presence of specific “target cells”. The percentage (%) of “target cells” killed is then quantified. More than 12% killing suggests a level of NK cell activation that usually requires treatment.

Currently, there are less than a half dozen Reproductive Immunology Reference Laboratories in the U.S.A that are capable of performing the K-562 target cell test reliably.

There exists a pervasive but blatant misconception on the part of many, that the addition of IL or IVIg to a concentration of NK cells could have an immediate down-regulatory effect on NK cell activity. Neither IVIg nor IL is capable of significantly suppressing already activated “functional NK cells”. They are believed to work through “regulating” NK cell progenitors which only thereupon will start to propagate down-regulated NK cells. Thus, testing for a therapeutic effect would require that the IL/IVIg infusion be done about 14 days prior to ovulation or progesterone administration…  in order to allow for a sufficient number of normal (non-activated) “functional” NK cell” to be present at the implantation site when the embryos are transferred.

Failure to recognize this reality has, in our opinion, established an erroneous demand by practicing IVF doctors, that Reproductive Immunology Reference Laboratories report on NK cell activity before and again, immediately following laboratory exposure to IVIg and/or IL in different concentrations. Allegedly, this is to allow the treating physician to report back to their patient(s) on whether an IL or IVIG infusion will be effective in downregulating their Nka.  But, since already activated NK cells (NKa) cannot be deactivated in the laboratory, effective NKa down-regulation can only be adequately accomplished through deactivation of NK cell “progenitors /parental” NK cells in order to allow them thereupon, to s propagate normal “functional” NK cells and his takes about 10-14 days, such practice would be of little clinical benefit. This is because even if blood were to be drawn 10 -14 days after IL/IVIg treatment it would require at least an additional 10 -14days to receive results from the laboratory, by which time it would be far too late to be of practical advantage.

ANTIPHOSPHOLIPID ANTIBODIES:

 Many women who experience “unexplained” IVF failure, women with RPL, those with a personal or family history of autoimmune diseases such as lupus erythematosus, rheumatoid arthritis, scleroderma, and dermatomyositis (etc.)  as well as women who have endometriosis (“silent” or overt) test positive for APAs. More than 30 years ago, we were the first to propose that women who test positive for APA’s be treated with a mini-dose heparin to improve IVF implantation and thus birth rates. This approach was based upon research that suggested that heparin repels APAs from the surface of the trophoblast (the embryo’s “root system) thereby reducing its ant-implantation effects.  We subsequently demonstrated that such therapy only improved IVF outcome in women whose APAs were directed against two specific IgG and/or IgM phospholipids [i.e., phosphatidylethanolamine (PE) and phosphatidylserine (PS)].  More recently low dosage heparin therapy has been supplanted using longer acting low molecular weight heparinoids such as Lovenox and Clexane.   It is very possible that APAs alone do not cause IID but that their presence might help to identify a population at risk due to concomitant activation of uterine natural killer cells (Nka) which through excessive TH-1 cytokine production causes in IID: This is supported by the following observations:

  • The presence of female APAs in cases of male factor cases appears to bear no relationship to IID.
  • Only APA positive women who also test positive for abnormal NK activity appear to benefit from selective immunotherapy with intralipid/IVIg/ steroids.
  • Most APA positive women who have increased NK cell activity also harbor IgG or IgM phosphatidylethanolamine (PE) and phosphatidylserine (PS) antibodies.

ANTITHYROID ANTIBODIES: (ATA).

 A clear relationship has been established between ATA and reproductive failure (especially recurrent miscarriage and infertility).

Between 2% and 5% of women of the childbearing age have reduced thyroid hormone activity (hypothyroidism). Women with hypothyroidism often manifest with reproductive failure i.e., infertility, unexplained (often repeated) IVF failure, or recurrent pregnancy loss (RPL). The condition is 5-10 times more common in women than in men. In most cases hypothyroidism is caused by damage to the thyroid gland resulting from of thyroid autoimmunity (Hashimoto’s disease) caused by damage done to the thyroid gland by antithyroglobulin and antimicrosomal auto-antibodies. 

The increased prevalence of hypothyroidism and thyroid autoimmunity (TAI) in women is likely the result of a combination of genetic factors, estrogen-related effects, and chromosome X abnormalities.  This having been said, there is significantly increased incidence of thyroid antibodies in non-pregnant women with a history of infertility and recurrent pregnancy loss and thyroid antibodies can be present asymptomatically in women without them manifesting with overt clinical or endocrinologic evidence of thyroid disease. In addition, these antibodies may persist in women who have suffered from hyper- or hypothyroidism even after normalization of their thyroid function by appropriate pharmacological treatment. The manifestations of reproductive dysfunction thus seem to be linked more to the presence of thyroid autoimmunity (TAI) than to clinical existence of hypothyroidism and treatment of the latter does not routinely result in a subsequent improvement in reproductive performance.

It follows, that if antithyroid autoantibodies are associated with reproductive dysfunction they may serve as useful markers for predicting poor outcome in patients undergoing assisted reproductive technologies.

Some years back, I reported on the fact that 47% of women who harbor thyroid autoantibodies, regardless of the absence or presence of clinical hypothyroidism, have activated uterine natural killer cells (NKa) cells and cytotoxic lymphocytes (CTL) and that such women often present with reproductive dysfunction. We demonstrated that appropriate immunotherapy with IVIG or intralipid (IL) and steroids, subsequently often results in a significant improvement in reproductive performance in such cases.

The fact that almost 50% of women who harbor antithyroid antibodies do not have activated CTL/NK cells suggests that it is NOT the antithyroid antibodies themselves that cause reproductive dysfunction. The activation of CTL and NK cells that occurs in half of the cases with TAI is probably an epiphenomenon with the associated reproductive dysfunction being due to CTL/NK cell activation that damages the early “root system” (trophoblast) of the implanting embryo. We have shown that treatment of those women who have thyroid antibodies + NKa/CTL using IL/steroids, improves subsequent reproductive performance while women with thyroid antibodies who do not harbor NKa/CTL do not require or benefit from such treatment

TEATMENT OF IID:

The mainstay of treatment involves the selective use of:

  • Intralipid (IL) infusion
  • IVIg therapy
  • Corticosteroids (Prednisone/dexamethasone)
  • Heparinoids (Lovenox/Clexane)

Intralipid (IL) Therapy:  IL is a suspension of soybean lipid droplets in water and is primarily used as source of parenteral nutrition. When administered intravenously, IL provides essential fatty acids, linoleic acid (LA), an omega-6 fatty acid, and alpha-linolenic acid (ALA), an omega-3 fatty acid.     

It is thought that fatty acids within the emulsion serve as ligands that activate peroxisome proliferator-activated receptors (PPARs) expressed by the NK cells. This is believed to decrease NK cell cytotoxic activity, and thereby enhance implantation A growing number of IVF programs, including ours, perform egg retrieval under conscious sedation using Propofol, a short acting hypnotic agent.

            Whatever the exact mechanism of action might be, Intralipid acts primarily to suppress NK cell over-production of TH-I cytokines. It exerts a modulating effect on certain immune cellular mechanisms largely by down-regulating cytotoxic /activated natural killer cells (NKa). This effect is enhanced through the concomitant administration of corticosteroids such as dexamethasone, prednisolone and prednisone which augment immune modulation of T cells. The combined effect of IL + steroid therapy suppresses pro-inflammatory cellular TH1 cytokines such as interferon gamma and TNF-alpha that are produced in excess by activated NK cells and cytotoxic lymphocytes/T-cells (CTL).   IL will, in about 80% of cases, successfully down-regulate activated natural killer cells (NKa) over a period of 2-3 weeks. It is likely to be just as effective as IVIg in this respect but at a fraction of the cost and with a far lower incidence of side-effects. Its effect lasts for ~ 4-6 weeks when administered in early pregnancy.

Intralipid is a suspension of soybean lipid droplets in water and is primarily used as source of parenteral nutrition. When administered intravenously, IL provides essential fatty acids, linoleic acid (LA), an omega-6 fatty acid, and alpha-linolenic acid (ALA), an omega-3 fatty acid.     

It is thought that fatty acids within the emulsion serve as ligands that activate peroxisome proliferator-activated receptors (PPARs) expressed by the NK cells. This is believed to decrease NK cell cytotoxic activity, and thereby enhance implantation A growing number of IVF programs, including ours, perform egg retrieval under conscious sedation using Propofol, a short acting hypnotic agent.

            Whatever the exact mechanism of action might be, Intralipid acts primarily to suppress NK cell over-production of TH-I cytokines. It exerts a modulating effect on certain immune cellular mechanisms largely by down-regulating cytotoxic /activated natural killer cells (NKa). This effect is enhanced through the concomitant administration of corticosteroids such as dexamethasone, prednisolone and prednisone which augment immune modulation of T cells. The combined effect of IL + steroid therapy suppresses pro-inflammatory cellular TH1 cytokines such as interferon gamma and TNF-alpha that are produced in excess by activated NK cells and cytotoxic lymphocytes/T-cells (CTL).   IL will, in about 80% of cases, successfully down-regulate activated natural killer cells (NKa) over a period of 2-3 weeks. It is likely to be just as effective as IVIg in this respect but at a fraction of the cost and with a far lower incidence of side-effects. Its effect lasts for ~ 4-6 weeks when administered in early pregnancy.

            Can laboratory testing be used to assess for an immediate effect of IL on Nka suppression?  Since the downregulation of NKa through IL (or IVIg) therapy can take several weeks to become measurable, it follows that there is really no benefit in trying to assess the potential efficacy of such treatment by retesting NKa in the laboratory after adding IL (or IVIg) to the cells being tested.

IVIg Therapy:  Until about a decade ago, the only effective and available way (in the US) to down-regulate activated NK cells was through the intravenous administration of a blood product known as immunoglobulin-G (IVIg). The fear (albeit unfounded) that the administration of this product might lead to the transmission of viral infections such as HIV and hepatitis C, plus the high cost of IVIG along with the fact that significant side effects occurred about 20% of the time, led to bad press and bad publicity for the entire field of reproductive immunology. It was easier for RE’s to simply say “I don’t believe IVIg works” and thereby avoid risk and bad publicity. But the thousands of women who had babies because of NK cell activity being downregulated through its use, attests to IVIg’s efficacy. But those of us who felt morally obligated to many desperate patients who would not conceive without receiving IVIg were facing an uphill battle. The bad press caused by fear mongering took its toll and spawned a malicious controversy. It was only through the introduction of IL less (about 15-20 years ago ), that the tide began to turn in favor of those patients who required low cost, safe and effective immunotherapy to resolve their IID.

 Corticosteroid Therapy (e.g., Prednisone, and Dexamethasone): Corticosteroid therapy has become a mainstay in the treatment of most women undergoing IVF. It is believed by most to enhance implantation due to an overall immunomodulatory effect. Corticosteroids reduce TH-1 cytokine production by CTL. When given in combination with IL or IVIG they augment the implantation process. The prednisone or dexamethasone therapy must commence (along with IL/IVIg) 10-14 days prior to egg retrieval and continue until pregnancy is discounted or until the 10th week of pregnancy.

 Heparinoid Therapy: There is compelling evidence that the subcutaneous administration of low molecular heparin (Clexane, Lovenox) once daily, (starting with the onset of ovarian stimulation) can improve IVF birthrate in women who test positive for APAs and might prevent later pregnancy loss when used to treat certain thrombophilias (e.g., homozygous MTHFR mutation)

What About Baby Aspirin? In our opinion, aspirin has little (if any) value when it comes to IID, and besides, it could even reduce the chance of success. The reason for this is that aspirin thins the blood and increases the potential to bleed. This effect can last for up to a week and could complicate an egg retrieval procedure or result in “concealed” intrauterine bleeding at the time of embryo transfer, thereby potentially compromising IVF success.

TH-1 Cytokine Blockers (Enbrel, Humira): TH-1 cytokine blockers, (Enbrel and Humira) are in our opinion relatively ineffective in the IVF setting. There has to date been no convincing data to support their use. However, these blockers could have a role in the treatment of a threatened miscarriage thought to be due to CTL/NK activation, but not for IVF. The reason is that the very initial phase of implantation requires a cellular response involving TH-1 cytokines. To block them completely (rather than simply restore a TH-1:TH-2 balance as occurs with IL therapy) so very early on could compromise rather than benefit implantation.

Leukocyte Immunization Therapy (LIT): The subcutaneous injection of the male partner’s lymphocytes to the mother is thought to enhance the ability for the mother’s decidua (uterus) to recognize the DQ alpha matching embryo as “self” or “friend” and thereby avert its rejection. LIT has been shown to up-regulate Treg cells and thus down-regulate NK cell activation thereby improving decidual TH-1:TH-2 balance. Thus, there could be a therapeutic benefit from such therapy. However, the same benefit can be achieved through the use of IL plus corticosteroids. Besides, IL is much less expensive, and the use of LIT is prohibited by law in the U.S.A.

There are two categories of immunologic implantation dysfunction (IID) linked to NK cell activation (NKa).

Autoimmune Implantation Dysfunction: Here, the woman will often have a personal or family history of autoimmune conditions such as Rheumatoid arthritis, Lupus Erythematosus, and thyroid autoimmune activity (e.g., Hashimoto’s disease) etc. Autoimmune as well as in about one third of cases of endometriosis, regardless of severity.  Autoimmune sometimes also occurs in the absence of a personal or family history of autoimmune disease.

When it comes to treating  NKa in  IVF cases complicated by autoimmune implantation dysfunction,  the combination of daily oral dexamethasone commencing with the onset of ovarian stimulation and continuing until the 10th week of pregnancy, combined with an initial infusion of IL (100ml, 20% Il dissolved in 500cc of saline solution, 10-14  days prior to PGT-normal embryo transfer and repeated once more (only), as  soon as the blood pregnancy test is positive), the anticipated chance of a viable pregnancy occurring within 2 completed IVF attempts (including fresh + frozen ET’s)  in women under 39Y (who have normal ovarian reserve)  is approximately  65%.

Alloimmune Implantation Dysfunction: Here, NK cell activation results from uterine exposure to an embryo derived through fertilization by a spermatozoon that shares certain genotypic (HLA/DQ alpha) similarities with that of the embryo recipient.

Partial DQ alpha/HLA match:  Couples who upon genotyping are shown to share only one DQ alpha/HLA gene are labeled as having a “partial match”. The detection of a “partial match” in association with NKa puts the couple at a considerable disadvantage with regard to IVF outcome. It should be emphasized however, that in the absence of associated Nka, DQ alpha/HLA matching whether “partial” or “total (see below) will NOT cause an IID. Since we presently have no way of determining which embryo carries a matching paternal DQ alpha gene, it follows that each embryo transferred will have about half the chance of propagating a viable pregnancy. Treatment of a partial DQ alpha/HLA match (+ Nka) involves the same IL, infusion as for autoimmune-Nka with one important caveat, namely that here we prescribe oral prednisone as adjunct therapy (rather than dexamethasone) and the IL infusion is repeated every 2-4 weeks following the diagnosis of pregnancy and continued until the 24th week of gestation. Additionally, (as alluded to elsewhere) in such cases we transfer a single (1) embryo at a time. This is because, the likelihood is that one out of two embryos will “match” and we are fearful that if we transfer >1 embryo, and one transferred embryos “matches” it could cause further activation of uterine NK cells and so prejudice the implantation of all transferred embryos. Here it should be emphasized that if associated with Nka, a matching embryo will still be at risk of rejection even in the presence of Intralipid (or IVIg) therapy.

Total (complete) DQ alpha Match:   Here the husband’s DQ alpha genotype matches both of that of his partner’s. While this occurs very infrequently, a total alloimmune (DQ alpha) match with accompanying Nka, means that the chance of a viable pregnancy resulting in a live birth at term, is unfortunately greatly diminished.  Several instances in our experience have required the use of a gestational surrogate.

It is indeed unfortunate that so many patients are being denied the ability to go from “infertility to family” simply because (for whatever reason) so many reproductive specialists refuse to embrace the role of immunologic factors in the genesis of intractable reproductive dysfunction. Hopefully this will change, and the sooner the better.

______________________________________________________________________

I urge you to  visit my website at  www.SherIVF.com. Then go to my Blog and access the “search bar”. Type in the titles of any/all of the articles listed below, one by one. “Click” and you will immediately be taken to those you select.  Please also take the time to post any questions or comments with the full expectation that I will (as always) respond promptly.

 

  • A Fresh Look at the Indications for IVF
  • The IVF Journey: The importance of “Planning the Trip” Before Taking the Ride”
  • Controlled Ovarian Stimulation (COS) for IVF: Selecting the ideal protocol
  • IVF: Factors Affecting Egg/Embryo “competency” during Controlled Ovarian Stimulation(COS)
  • The Fundamental Requirements For Achieving Optimal IVF Success
  • Use of GnRH Antagonists (Ganirelix/Cetrotide/Orgalutron) in IVF-Ovarian Stimulation Protocols.
  • Human Growth Hormone Administration in IVF: Does it Enhances Egg/Embryo Quality and Outcome?
  • IVF and the use of Supplementary Human Growth Hormone (HGH) : Is it Worth Trying and who needs it?
  • The BCP: Does Launching a Cycle of Controlled Ovarian Stimulation (COS). Coming off the BCP Compromise Response?
  • Blastocyst Embryo Transfers Should be the Standard of Care in IVF
  • Anti Mullerian Hormone (AMH) Measurement to Assess Ovarian Reserve and Design the Optimal Protocol for Controlled Ovarian Stimulation (COS) in IVF.
  • IVF: Approach to Selecting the Best Embryos for Transfer to the Uterus.
  • Fresh versus Frozen Embryo Transfers (FET) Enhance IVF Outcome
  • Frozen Embryo Transfer (FET): A Rational Approach to Hormonal Preparation and How new Methodology is Impacting IVF.
  • Genetically Testing Embryos for IVF
  • Staggered IVF
  • Staggered IVF with PGS- Selection of “Competent” Embryos Greatly Enhances the Utility & Efficiency of IVF.
  • Preimplantation Genetic Testing (PGS) in IVF: It should be Used Selectively and NOT be Routine.
  • IVF: Selecting the Best Quality Embryos to Transfer
  • Preimplantation Genetic Sampling (PGS) Using: Next Generation Gene Sequencing (NGS): Method of Choice.
  • PGS in IVF: Are Some Chromosomally abnormal Embryos Capable of Resulting in Normal Babies and Being Wrongly Discarded?
  • PGS and Assessment of Egg/Embryo “competency”: How Method, Timing and Methodology Could Affect Reliability
  • Endometrial Receptivity Array (ERA): Is There an actual “There, There”?
  • IVF Failure and Implantation Dysfunction:
  • Diagnosing and Treating Immunologic Implantation Dysfunction (IID)
  • The Role of Immunologic Implantation Dysfunction (IID) & Infertility (IID):PART 1-Background
  • Immunologic Implantation Dysfunction (IID) & Infertility (IID):PART 2- Making a Diagnosis
  • Immunologic Dysfunction (IID) & Infertility (IID):PART 3-Treatment
  • Thyroid autoantibodies and Immunologic Implantation Dysfunction (IID)
  • Immunologic Implantation Dysfunction: Importance of Meticulous Evaluation and Strategic Management:(Case Report
  • Intralipid and IVIG therapy: Understanding the Basis for its use in the Treatment of Immunologic Implantation Dysfunction (IID)
  • Intralipid (IL) Administration in IVF: It’s Composition; How it Works; Administration; Side-effects; Reactions and Precautions
  • Natural Killer Cell Activation (NKa) and Immunologic Implantation Dysfunction in IVF: The Controversy!
  • Endometrial Thickness, Uterine Pathology and Immunologic Factors
  • Vaginally Administered Viagra is Often a Highly Effective Treatment to Help Thicken a Thin Uterine Lining
  • A Thin Uterine Lining: Vaginal Viagra is Often the Answer (update)
  • Cervical Ureaplasma Urealyticum Infection: How can it Affect IUI/IVF Outcome?
  • The Role of Nutritional Supplements in Preparing for IVF
  • The Basic Infertility Work-Up
  • Defining and Addressing an Abnormal Luteal Phase
  • Male Factor Infertility
  • Routine Fertilization by Intracytoplasmic Sperm Injection (ICSI): An Argument in Favor
  • Hormonal Treatment of Male Infertility
  • Hormonal Treatment of Male Infertility
  • Antisperm Antibodies, Infertility and the Role of IVF with Intracytoplasmic Sperm Injection (ICSI)
  • Endometriosis and Infertily
  • Endometriosis and Immunologic Implantation Dysfunction (IID) and IVF
  • Endometriosis and Infertility: Why IVF Rather than IUI or Surgery Should be the Treatment of Choice.
  • Endometriosis and Infertility: The Influence of Age and Severity on Treatment Options
  • Early -Endometriosis-related Infertility: Ovulation Induction (with or without Intrauterine Insemination) and Reproductive Surgery  Versus IVF
  • Deciding Between Intrauterine Insemination (IUI) and In Vitro Fertilization (IVF).
  • Intrauterine Insemination (IUI): Who Needs it & who Does Not: Pro’s & Con’s!IUI-Reflecting upon its Use and Misuse: Time for a Serious “Reality Check
  • Mode of Action, Indications, Benefits, Limitations and Contraindications for its ue
  • Clomiphene Induction of Ovulation: Its Use and Misuse!

.

Name: Vicky A

Dear Dr Sher,

I am currently 8+2 pregnant following our final FET with our last euploid embryo. It was a natural FET. My 8 week scan showed everything to be on track with no obvious concerns.

My IVF clinic say to stop all progesterone at 10 weeks (I’m on 3 x lubion and 3 x 400mg cyclogest per day). I am also under the care under a reproductive immunologist who will keep me on 2 x 400mg cyclogest per day until 16 weeks. Is it ok to drop straight down to just 2 cyclogest at 10 weeks or should I taper off more gradually? Or should I ask to keep on full progesterone support for longer?

I would value your opinion.

Many thanks

Vicky

Answer:

I agree eith your RE!

 

Geoff Sher